• Shuffle
    Toggle On
    Toggle Off
  • Alphabetize
    Toggle On
    Toggle Off
  • Front First
    Toggle On
    Toggle Off
  • Both Sides
    Toggle On
    Toggle Off
  • Read
    Toggle On
    Toggle Off
Reading...
Front

Card Range To Study

through

image

Play button

image

Play button

image

Progress

1/486

Click to flip

Use LEFT and RIGHT arrow keys to navigate between flashcards;

Use UP and DOWN arrow keys to flip the card;

H to show hint;

A reads text to speech;

486 Cards in this Set

  • Front
  • Back
(Oct-2012 Q5) You are asked to assess a 4 year old child who is scheduled for a strabismus correction as a day case procedure. 1. What are the issues relevant to anaesthesia (70%). 2. What would prevent you from discharging this patient home after surgery? (30%).
a)
Patient Fx
- may have associated syndrome, and potentially epilepsy, cardiac anomalies, etc.
- may be associated with condition with difficult intubation (mucopolysaccharidosis)
- 4yo child, who may be anxious (with parents), need to consider premedication, and parental presence on induction, as well as IV vs Gas induction if patient cooperative
- any other risk fx for PONV? (past hx or family hx, age >3)

Anaesthetic Fx
- Remote airway access, use of LMA vs ETT in this situation
- potentially stimulating procedure, so need adequate depth, and intraop analgesia
- postop analgesia can be by surgical LA infiltration
- risk of oculocardiac reflex (bradycardia), and need for treatment
- high risk of PONV, due to type of surgery, duration of surgery, and age of child (~55% risk, as per SAMBA guidelines). Therefore antiemetic prophylaxis is vital (ie, dexamethasone 0.1mg/kg, ondansetron 0.1mg/kg), +/- TIVA and No N2O.
- potential for emergence delerium

Surgical Fx
- need for adequate conditions (depth, still)
- stimulating, PONV

b)
Pt must meet criteria for discharge from day surgery as per ANZCA professional document. These include Pt/Social Fx.
Social
- has to have responsible/mature parents/carers that understand discharge instructions
- lives within 60 mins of medical care
- has telephone

Patient
- low risk of airway, bleeding probs (in this situation)
- analgesia ok
- PONV controlled
- alimentation (has at least tolerated fluids)
- passed urine?
- mobilised (if usually applies to child, not necessarily relevant to paed practice).
(Apr-2009 Q13) Outline the steps you would take to ensure the safe introduction of elective paediatric surgery at your local private hospital.
Requires a multi-disciplinary approach, with surgical, anaesthetic, nursing and administrative involvement.
- consultation with appropriate regulatory authorities (levels of government)
- committee of above groups to commence planning, implementation, and SCOPE of practice
- write protocols for care
* day surgery as per ANZCA guidelines (ie, considering age, ASA, etc)
* other day surgery protocols
* protocols for ALS, pain, PONV, discharge for Paed patients
- consideration of facilities for paed pts / parents
* pre-admission, consultation rooms, PACU, wards
- someone to advise on and be responsible for paed equipment
* airway
*breathing circuits
* IV's, BP cuffs, fluids, drugs
- ensuring staff (surgeons, anaesthetists, nurses, anaesthetic assistants) are credentialled and appropriately trained for paeds, and trained as per ANZCA guidelines
- liason and consultation with tertiary facility, specifically for advice/transfer of patients unable to be cared for in private facility
- graded implementation, and ongoing audit/review to improve
(Oct-2008 Q7) A 6yo girl with severe spastic cerebral palsy presents for orthopaedic surgery to correct lower limb deformities. Outline the implications of cerebral palsy for anaesthesia management for this operation.
CP is a non-progressive disorder of motor and posture resulting from a cerebral insult pre/peri/post-natally or even during infancy. May be spastic, dyskinetic, ataxic or mixed.

Implications
- A
Risk of aspiration on induction due to high frequency of GORD. RSI uncommonly used however, due to difficulties with pre-induction IV access and cooperation.
- B
possible chronic aspiration, poor cough, restrictive lung disease due to chest wall spasticity / SCOLIOSIS. Means ETT likely required for airway protection, suction, and care with oversedation/opioids in postoperative period due to risk of sedation/respiratory depression
- C
Risk of blood loss from osteotomies, need to monitor blood loss and fluid resuscitate
- D
may have cognitive impairment, amking periop communication difficult (ie, induction, emergence, and postop pain assessment).
potentially painful procedure, and may not be able to use PCA if CI.
- E
"large neonate", with thin skin, and potential for periop hypothermia (which may cause poor wound healing, coagulopathy, and slow emergence/sedation). Need to keep warm with FAW, warm fluids, etc.
- F
sensitivity to opioids/sedatives
painful operation, so useful to use regional technique such as epidural (with LA +/- fentanyl +/- clonidine, run at 0.1-0.2ml/kg/hr).
G - GORD and risk aspiration mentioned. risk of PONV after surgery (esp opioids and duration).
J - risk of contractures, and pressure areas with difficult positioning/padding/IV access
(May-2008 Q8) You are asked to provide assistance to resuscitate a baby. One minute after birth the baby is apnoeic, grey/blue all over, floppy and unresponsive to stimulation, with a pulse felt at the umbilical stump of 60/min. What is this baby's APGAR score? Describe your resuscitation of the baby.
The APGAR is 1. APGAR score is calculated at 1 and 5 minutes, and uses
* Colour
* Tone
* Resp effort
* HR
* "Reflex irratibility"
To give score out of 10.

The resuscitation of this neonate should follow neonatal resusc guidelines, and follow an A, B, C approach.

1) Is baby TERM, CRYING/BREATHING, and has TONE? THen goes straight to mum. If not, needs further ax.
1st 30 secs is to stimulate, dry, etc, and ax HR/Resp effort.
*A
Airway should be cleared
* B
PPV should be instituted if no resp effort +/- HR<100.
CPAP is gasping/poor resp effort. And sats monitoring.
* C
if initial HR < 60, chest compressions should be commenced (rate 3:1 breath), with 2 fingers.

2) after 1 min, if HR < 100, then ventilation should be corrected to ensure adequacy (ie, OPA, higher pressures, even intubation).
if HR < 60, start compressions.

If starting compressions, IV access should be attempted.
- fluid 10ml/kg
- adrenaline 10mcg/kg (0.1ml of 1:10,000)
100% oxygen should be given at this stage.

If term, neonate is likely to require a 3-3.5 ETT, and is approx 3-3.5kg.

further care involves checking for pneumothorax, hypovolemia, or other abnormalities.

post- resuscitation care should continue in NICU or SCN (depending on resusc needed), and family should be informed.
(May-2008 Q11) You are the anaesthetist at a childrens' hospital. A 3yo schedules for dental restoration and extractions is found to have a systolic murmur during your preoperative assessment on the day of surgery. They have been on a waiting list for 6 months and have had a dental abscess that settled with antibiotics. Describe how you would evaluate the significance of this murmur and how this decision would affect your decision to proceed or not with surgery.
only 1% of murmurs are from CHD. The aim is to differentiate from the children with CHD that need evaluation and treatment prior to elective surgery, from those with innocent murmurs.

Evaluate by Hx, Ex, Ix and Consultation.

- Hx
"well" child, any comorbidities or syndromic, recurrent chest infections
hx of poor feeding, FTT, poor ability to keep up with peers in terms of playing/exercise
hx of blue spells, squatting
family hx of cardiac conditions (ie, HOCM)

-Ex
general appearance, cyanosis (sats)
pulses (4 limbs - coarctation)
signs of heart failure (lung creps, hepatomegaly, oedema)
Murmur
* timing (late systolic, pansystolic bad)
* quality (loud, harsh)

- Ix
ECG. look for signs of LVH or RVH
*RVH R wave V1 > 3.5 sqrs (<5yo) with upgoing TW
*LVH R wave in V5,6 > 8 sqrs!
*VE's
TTE if in tertiary centre with easy access to cardiology, especially if any concerning signs.

If patient does not have any comorbidities or concerning features on Hx/Ex/Ix, then would be suitable to go ahead with procedure, and initiate referral for follow-up with cardiologist.

If any signs of concern, and available - cardiology consult prior to surgery on DOS, or cancellation, with appropriate referral. This would have to be discussed properly with parents.

If antibiotic prophylaxis came into it - I would personally not be thinking of proceeding, as this implies you think it is some sort of CHD.
(May-2007 Q4)A 2 year old child has burns to lower body from immersion into a hot bath. Describe your assessment and management of pain and fluid requirements in the first 2 hours following injury.
- May or may not have significant burns (ie, superficial vs deep) from this injury, and may be different depending on accidental vs NAI
- Need to make an Ax of burn size and severity
* Rule of 9's vs 1% palm, vs Paed burns charts
*significant burns > 10% BSA in children require fluids (partial/full) and require t/f to specialist facility, or full thickness > 5% any age.
*signficant burns involving perineum, joints, circumferential burns on limbs/torso, inhalational injury, face/eyes/ears/hands/feet

- fluid requirements are as per % burn (parkland formula), maintenance requirements, and assessed by urine output adequacy.
- RCH formula,
*3ml/%BSA/kg crystalloid/albumin over 24 hrs since burn (1/2 in 1st 8 hrs).
*normal maintenance also added to this
*urine output > 0.75ml/kg/hr
*2nd 24 hrs usually involves about 1/2 the "resuscitation" amount.

Pain
- should ideally be observed with objective measurement tool (such as FLACC - Facial expression, Leg movement, Activity, Crying, Consolability)
- requires titration of IV opioid if significant burn injury, and care must be taken as potentially hypovolemic (could titrate 0.1-0.2mg/kg morphine in 10 ml syringe, and give 1-2ml at a time).
(Sep-2006 Q10) Discuss in detail the technique of rapid sequence induction with cricoid pressure in a child. Include the reasons for your choice of relaxant.
detail any technique that is safe. Particular attention to problems with RSI in a child. Aim is rapid airway securing, and avoidance of aspiration.

- IV access! potentially the most difficult with an uncooperative/distressed child, and would depend on indication for RSI on how long this is attempted.
- Pre-oxygenation. again depends on cooperation. May be difficult in anxious, may not get good pre-O2 if screaming!
- Dose, depends on age, size, medical stability. Would tend to use 3-5mg propofol, and suxamethonium 2mg/kg, depending on medical status/hx. atropine if < 12 months for sux bradycardia.
- discuss parental presence on induction
- use of atracurium if medical status requires
- depending on age, may elect to gently BMV with cricoid to maintain oxygenation (rapid destauration in smaller child due to physiology)
- choice of uncuffed vs cuffed tube (would choose cuffed given only want 1 go, and best airway protection).

Reasons for sux
- most rapid onset drug, best intubating conditions quickest, although rocuronium is close in high dose
- likely to wear off quickly for spontaneous ventilation
- has some risks - but benefits in true emergency outweigh these in my opinion.
(Sep-2005 Q1) What are the indications for tracheal intubation in a 3 year old who presents with "croup"? Describe your technique for intubation.
Croup is a laryngotracheobronchitis, usually caused by a virus. It presents between 6/12 and 6 years, peaking around 2-3yo. about 0.5-1% require intubation.

Indications based on clinical ax
Hx
- rapid onset
- other medical comorbidities
Ex
- resp distress, exhaustion
- lethargic
- sitting up, drooling, unable to lie flat
- cyanosis / hypoxaemia (late)

failure to respond to medical treatment.

Intubation technique
- parental presence, try not to upset child, no IV, etc
- gas induction with 100% O2 /Sevo, in position child is comfortable
- experienced paed anaesthetist in room, in contorl of airway
- ENT surgeon on standby in case of loss of airway
- IV access when deep enough (assessed by clinical and MAC)
- view of larynx should not be difficult, but passing of ETT may be, choose 0.5-1 size smaller than expected, if possible croup tube (longer) and nasal intubation (required for ICU if possible). Unlikely to be leak.
- commence sedation, discontinue anaesthetic and ensure appropriate treatment with steroids +/- antibiotics.
(May-2004 Q12) Working in a small obstetric unit you are asked to attend at the birth of a child where there is meconium stained liquor. How will you manage the infant's resuscitation?
See Q4 for neonatal resuscitation.

Main thing here is to determine whether the child is flat or whether they are thriving. If flat, then they need to have trachea suctioned (via ETT) prior to any resuscitation. If not, they do not require suctioning.

Are they TERM, TONED, and CRYING? They can go straight to mother.
(May-2004 Q13) Describe the characteristics of a ventilator suitable for neonates.
.
(Sep-2003 Q2) A 4 yo boy weighing 15kg presents for day surgery repair of a left inguinal hernia for which you plan general anaesthesia and caudal block. He has no significant past history, and is well. Justify your choice of agent(s) for caudal injection for this child.
- Caudal block involves injection of LA into epidural space (caudal) through the saccrococcygeal membrane/sacral hiatus.
- The volume of LA determines the level of the block, generally 0.25% bupivacaine or levobupivacaine is used, my standard dose is 1ml/kg (gives a good lumbosacral block).
Choice of LA has some theoretical benefits - levo/rop less CVS toxicity in case of inadvertent IV injection. Rop has less motor block than bup for same volume, but this is likely a dose potency issue.
- Clonidine
increases duration of analgesia, without the pruritis/PONV/urinary retention issues of opioids.
prolongation of motor block may not be warranted in day surgery
- adrenaline
mainly useful for test dose (5mcg/ml). May provide analgesia through alpha-receptors. risk of spinal cord ischaemia though. does not prolong action of long-acting LA.
- opioids
morphine and fentanyl may improve block and duration, but also have SE's, such as sedation and respiratory depression. For day case surgery I would avoid.

In summary - a block of levobupivacaine 0.25% 1ml/kg (15ml), with 1mcg/kg clonidine added.
(May-2003 Q2) A four week old infant presents for bilateral inguinal herniotomy at a free-standing day surgery unit with a significant paediatric caseload. This infant is to have a general anaesthetic. The parents wish to return to the country that evening. Is the use of a laryngeal mask an acceptable option for airway management? Justify your answer.
Inguinal hernia repair is the most common surgical procedure in this age group, with 1% of children requiring this. It is more common in the premature infant, with up to 30% requiring IHR. It is also commonly performed in these infants just prior to discharge (to ensure no complications from hernia incarceration after discharge home).

The main issue with surgery in this age group, is the risk of postoperative apnoeas.
An Apnoea can be central/obstructive or mixed.
It is defined as cessation of airflow for >20 secs, or less if associated with bradycardia or hypoxaemia.
- higher risk with prematurity / low PCA
- higher risk with other comorbidities such as chronic lung disease, anaemia, neuromuscular disease, previous apnoeas
- higher risk with GA vs spinal anaesthesia (no sedation).
A bilateral repair is more likely to require a GA due to time constraints of a spinal anaesthetic.
There is also the issue of how a GA effects the neurobiological development of an infant - this question is not yet answered.

LMA advantages
- simple to use
- available in size for infant (0-5kg would be size 1).
- avoids trauma to trachea and risk of post-extubation stridor (esp in day case)
- can breathe spontaneously - no need for muscle relaxation in this procedure

LMA disadvantages
- although size 1 for this age group, often difficult to get a good fit/seal, with risk of slight airway obstruction
- potentially difficult to change to ETT during case, as covered in drapes, and baby down bed towards surgeon
- unable to provide PEEP (often required for neonates/infants due to CC>FRC during normal tidal breathing, with shunt)
- still requires anaesthetic observation until awake (does not increase "turnover"), as still has risk of laryngospasm on lightening.
- risk of inflating stomach with PPV if poor seal
- highest incidence of complications with 1-1.5 LMA, epiglottis impingement and airway obstruction, and case reports of nerve palsies

I would choose an ETT (probably size 3.5) in this infant, as is a more secure airway, can provide PPV/PEEP, without much increase in risk as long as careful with sizing.
(May-2003 Q1) How would you provide post-op analgesia for this infant? Include information on dosage and routes of administration.
The options are to use systemic and regional analgesic techniques.

Systemic
- paracetamol, NSAID, Opioid

Regional
- Caudal, Bilateral Ilioinguinal blocks, LA infiltration

Ideally, given risk of POA, systemic opioids should be avoided. NSAID's probably also avoided in this age group due to immature renal function.

I would provide paracetamol (orally or IV), and a caudal block, after appropriate consent from the parent.

My caudal block would be 1ml/kg of 0.25% levobupivacaine with 1mcg/kg clonidine for better quality and prolonged duration of analgesia, and paracetamol dose would be 15mg/kg IV paracetamol intraoperatively.

Postoperatively I would advise regular paracetamol 15mg/kg QID for 2-3 days. Given immature hepatic enzymes at this age, I would avoid higher doses, and also regular doses for more than about 3 days postoperatively. (a neonate <10 days old, or ex-prem should be dosed at 7.5mg/kg).
(May-2003 Q3) On what basis would you decided if it is appropriate for this infant to return to the country that evening?
This decision involves patient/anaesthetic/surgical and social factors. It should also comply with ANZCA guidelines!.

The general criteria are as follows
- low risk of postop bleeding
- low risk of postop airway compromise
- low risk of PONV
- pain controlled by outpatient methods
- able to tolerate fluids

This procedure meets these criteria, but obviously would have to take into account the patient's state postop.

Social Fx
- responsible parent/carer who understands postop instructions
- within 60 mins of appropriate medical care
- access to telephone

Patient Fx
- The main risk for an infant, is risk of postoperative apnoea (especially following GA).
*Risk increases with lower gestational age at birth
*risk increases with lower PCA
*previous apnoea
*GA vs spinal
*comorbidities (already mentioned above).

The college guidelines state
- term baby (not specified!) should be > 6/52 old
- ex-prem should be > 52/52 PCA
- lowest risk overall is > 60/52 PCA*
- potentially "neonate" should not be performed as a day case OR at a non-specialist paediatric facility

The highest risk of apnoea is in the first 12 hours postop.

Therefore, I believe this child should not be performed as a day case, in this facility.
(Sep-2001 Q1) You discover a heart murmur, which has not been noted before, in a 3 yo child presenting for elective inguinal hernia repair. How would you assess this child at the bedside with respect to this murmur, and what findings would prompt you to refer this child to a cardiologist prior to surgery?
see 5.
(Sep-2001 Q2) A cardiological opinion is sought, and echocardiography is advised. The child becomes extremely upset, and the paediatrician and parents ask you to sedate him for the procedure. How will you manage this?
.
(Sep-2001 Q3) The echocardiogram is reported as normal and the cardiological opinion is that the murmur is physiological and of no concern. Outline and justify your plan for postoperative analgesia following the hernia repair, including after discharge.
.
(May-2001 Q7) An 8 month old, 10 kg infant presents for laparotomy following failed barium enema reduction of an intussusception. Describe and justify your perioperative fluid management.
see 2010b - Q5.
(May-2001 Q8) At the completion of surgery the haemoglobin is measured at 70g/L. Would you transfuse this patient? Justify your answer.
This would depend on any comorbidities the child had, and expected ongoiing blood loss or under-resuscitation (ie, more fluid will dilute this value further). As well as assessment of any signs of inadeqaute organ oxygenation (such as tachycardia, lactate still up).

In an otherwise healthy infat, with no ongoing losses, I would monitor the Hb and not transfuse - give there are many risks to transfusion which could be avoided.

An 8 month infant is also likely to have a lower Hb than a neonate or 1yo, due to physiologic anaemia of infancy (ie, Hb 70-100 at approx 6 months of age), due to change from foetal to adult Hb. Therefore, this may not be significantly deranged from normal.
(May-2001 Q9) In what circumstances would it be reasonable to provide continuous epidural analgesia for postoperative pain relief in this child?
.
(Aug-2000 Q9) List the anatomical differences between the neonatal and adult airway. Include the significance of each difference.
Neonatal airway
- large head / occiput (need for neutral position for intubation/ventilation)
- narrow nasal passages (nose easily obstructed, obligate nasal breathing can cause resp distress)
- large tongue (potential obstruction, and difficult laryng
Neonatal airway
- large head / occiput (need for neutral position for intubation/ventilation)
- narrow nasal passages (nose easily obstructed, obligate nasal breathing can cause resp distress)
- large tongue (potential obstruction, and difficult laryngoscopy)
- epiglottis u-shaped and long/floppy (may need different method of laryngoscopy, such as straight blade to lift up epiglottis rather than tip in vallecula)
- larynx at C3-4, rather than 5-6, anterior (may need BURP, may be more difficult to get view of larynx with traditional laryngoscopy)
- funnel-shaped trachea, with cricoid narrowest part, also small, so any oedema will result in significant increase in airway resistance (potential for subglottic oedema/stridor with poor ETT sizing
- shorter trachea (more likely endobronchial intubation, especially with cuffed tube)
- peripheral airways small and contribute 50% of AWR versus 20% in adults (bronchiolitis may cause severe RD, also non-responsive to bronchodilators due to immature smooth muscle)

Other issues
- higher VA/FRC ratio and higher oxygen consumption, so more rapid desaturation with apnoea (RSI)
- also CC>FRC with normal tidal breathing, so more likely as well
- fatigue much more likely with resp distress due to reliance on diaphragm (with more type I fibres), inefficient "non- bucket-handle" ribs
.(Apr-2000 Q10) A three year old child is being assessed for insertion of middle ear drainage tubes. On examination you discover that the child has a precordial murmur. What information would you be seeking in your assessment of this child to decide if the murmur is innocent?
see 5.
(Apr-2000 Q11) If the child is found to have a ventricular septal defect, but is otherwise well, how will this influence your anaesthetic management?
.
(Aug99) A nine year old child with spina bifida presenting for a tendon transfer procedure is said to have multiple allergies including latex and antibiotics. How would you decide whether or not the child has latex allergy?
.
(Aug99) If the child does have latex allergy, describe the precautions that should be taken peri-operatively to prevent this child developing a latex reaction?
.
(Aug99) What would you advise the parents regarding the risks that latex allergy adds to the perioperative period?
.
(Apr99) A 4 yo boy weighing 15kg presents for day surgery repair of a left inguinal hernia for which you plan GA and a caudal block. He has no significant past history and is well. Describe how you would perform a caudal injection for this child.
Caudal
Technique - informed consent from parent, IV access, appropriate drugs, monitoring, drugs and equipment. asleep with ECG monitoring, place child laterally in foetal position, aseptic technique.
With left hand
- find PSIS's with thumb/3rd finger,
Caudal
Technique - informed consent from parent, IV access, appropriate drugs, monitoring, drugs and equipment. asleep with ECG monitoring, place child laterally in foetal position, aseptic technique.
With left hand
- find PSIS's with thumb/3rd finger, and with index finger at apex of triangle, palpate the sacrual hiatus/cornua (the incomplete midline fusion of the S4/5 vertebrae). The sacral cornua are the base of another triangle pointing cephalad.
With 2 fingers over the cornua, I would place a 22g needle between them, pointing towards apex of 2nd triangle, at 60 degrees to the skin (actually fairly flat if lumbar spine flexed enough). Aiming to get into the sacral canal, and feel a "give" as the needle passes through the saccrococcygeal ligament.
- would then wait to see if blood or CSF drips out of needle, connect 10ml syringe, aspirate gently, and inject a few mls, then wait for ecg changes, disconnect again, check, reconnect and slowly incrementally dose up to 1ml/kg.
1ml/kg 0.25% levobupivacaine with 1mcg/kg clonidine & 5mcg/ml adrenaline
(Apr99) If his parents express concern about caudal analgesia, what alternative analgesia options would you offer? Include a brief comment on their particular advantages and disadvantages.
.
(Jul98) Compare propofol with sevoflurane as the sole anaesthetic general anaesthetic agent for a 3 yo child requiring insertion of drainage tubes for chronic otitis media.
.
(Apr97) A 14yo girl, 130cm tall, with idiopathic scoliosis is scheduled for corrective fixation via a thoracotomy. Controlled hypotension will be used. What are the options available for providing collapse of the right lung?
.
(Apr97) Two hours into the operation the urine output is measured as 5 ml (in two hours). How would you manage this?
.
(Apr97) At the time of skin closure her core temperature is 34.1 degrees celcius. How would you manage this?
.
(Aug96) A 3 year old child presents with respiratory distress associated with a respiratory tract infection. How would you assess the need for tracheal intubation?
.
(Aug96) Describe the facilities you require when you decide to intubate the trachea.
(Change this question into - the minimum facilities required to provide anaesthesia...)
Minimum facilities for anaesthesia require attention to
- Staffing
- Equipment
- Facilities.

Anaesthesia must be administered by
*an appropriately trained medical practitioner or trainee under the supervision of such a person
*after a pre-anaesthetic consultation
*with appropriate monitoring of physiological variables

Staffing
- an assistant for the anaesthetist (wholly & exclusive while assistanting - for induction/emergence, and at any time at short notice)
- adequate assistance for positioning/transferring the patient (minimum 3 people for this)
- adequate technical support for equipment

Equipment
( 1 or more anaesthetists to advise on choice/maintenance, and 1 or more nursing/tech to be responsible for ongoing maintenance, etc)
- accurate oxygen flow
- calibrated vaporisers
- infusion devices
- breathing systems (and sterility)
- anaesthetic machines complying with minimum safety standards

(MUST HAVES)
- separate "means of inflating the lungs with O2"
- MALES (masks, airways, LMA's, 2 laryngoscopes, ETT's, Suction)
- bougies, magill's, tapes
- tourniquets, IV equipment
- stethoscope, sphygmomanometer, scissors
- monitoring as per guidelines
- scavenging
- protection for staff (gowns, gloves, sharps bins)

(READILY AVAILABLE)
- equipment for managing difficult airway
- automatic ventilation
- direct arterial/venous pressure measurement
- rapid infusors
- defib
- interpleural drainage sets
- equipment for regional anaesthesia
- equipment to heat or cool

Facilities
- lighting (incl emergency & power supplies), telephone/intercom, fridge, thermostat, patient trolleys
- DRUGS
(just about anything! - including Dantrolene, min 24 ampoules, large hospitals have 36)

Checking equipment
- regular cleaning, sterilising, maintenance

Facilities
- PACU
- ECT, Delivery suite, dental surgeries, medical imaging all have their own requirements also (but must meet minimum standards).
(Aug96) What are the possible causes of cardiac arrest in this child one hour after intubation?
.
(Apr96) What are your views on the statement: "children having a tonsillectomy should not be prescribed narcotic analgesics post-operatively"?
- Blanket statements don't help in the management of individual patients
- more important in patients with OSA
*young (<2)
*snoring/apnoeas
*behavioural issues, hyperactivity
*signs of RHF

Tonsillectomy should not be that painful, but some people may need opioids.
(Oct-2012 Q11) You have been asked to provide anaesthesia for a lower uterine segment caesarean section in a woman at 38/40. She has a pacemaker-AICD implanted for a known cardiomyopathy. Her current echo demonstrates an EF of 35% with mild to moderate left ventricular global hypokinesis. Clinically, the patient feels very well. 1. What additional preparations with respect to her cardiovascular system would you make to ensure the safe management of this patient during her Caesarean section? 2. Outline the relative benefits and risks of a regional technique compared with general anaesthesia in this patient.
1)
Basically Ax (Hx/Ex/Ix) and COPE
- Hx
cause of cardiomyopathy, course, complications, function of PPM (ie, BiV, or PPM-dependent), Hx arrythmias, medications (incl anticoagulation?)
Ex
- confirm stability of disease, PPM-function, other comorbid disease
Ix
baseline function, ECG/TTE/Bloods

C (Consultation)
- Cardiologist / PPM tech and check - especially underlying rhythm, magnet response, battery life, and AICD function (recent shocks and when it is programmed to shock)
- Obstetricians/Midwives/Paeds
Optimisation
- may include plan for anticoagulation/DVT prophylaxis, monitoring, change in function to AICD (although not necessarily required given below umbilicus surgery)
Premd
- maybe not necessary, all usual cardiac meds
Explain
consent for anaesthetic technique

2)
Risks of Regional
- sympatholysis, drop in preload and potentially CO, risk of poor coronary perfusion
- timing with any anticoagulation
- conversion to GA may be deleterious
- no intraop TOE if required
- no HR response to drop in BP due to PPM?

Benefits
- (esp if slowly titrated CSE), can maintain haemodynamic stability, controlled
- avoids postop ventilation
- better analgesia?
- avoids risk of GA (difficult airway, aspiration, foetal depression)
- can be awake to bond with child
(Apr-2009 Q8) Outline the features and clinical management of amniotic fluid embolism.
AFE is a catastrophic obstetric emergency, which occurs when amniotic fluid or foetal cells enter the maternal circulation, causing the possible triad of hypoxia, cvs collapse and DIC.
- now called the "anaphylactoid syndrome of pregnancy" due to similarities to anaphylaxis.
- 1:8000-80,000, can occur at any time during pregnancy, but mostly peri-delivery
- high mortality and morbidity rate, foetal outcome also poor.
- potential risk factors include (none proven?)
* increased maternal age, multiparity, FDIU, polyhydramnios, maternal atopy, uterine rupture, placenta accreta, IOL & C/section
*Sx & Sx include
- agitation, restlessness, acute foetal compromise, chest pain (air embolism too), dyspnoea, cyanosis/hypoxia, hypotension/tachycardia, LOC, under GA (bronchospasm/high AW pressures, decreased ETCO2, CVS collapse, DIC)
- characterised by "biphasic" response of initial acute pulmonary HT with RV dysfunction (30 mins), then LVF

Management
- supportive, follow ALS guidelines if arrested
- remember additional things such as L lateral tilt, early intubation, perimortem C/section
- likely to need FFP/platelets and massive transfusion, possibly VIIa

DDx
- Obstetric
Eclampsia, placental abruption/PPH, peripartum cardiomyopathy
- non-obstetric
VTE, VAE, septic shock, anaphylaxis, AMI, total spinal, drug toxicity
(Oct-2008 Q8) A 25yo primigravida patient presents to the delivery suite at 38 weeks gestation complaining of a headache and difficulty with her vision. Her BP is 180/115 and she has clonus. CTG monitoring shows no indications of foetal distress. Outline your initial management of her pre-eclampsia
- This woman has SEVERE PE, with hypertension and neurological signs (headache/clonus). This is an obstetric emergency, and she should be assessed ASAP.
- Apart from Obs/Anaes Hx, particularly focussing on hypertensive disorders of pregnancy, the aims of initial management are to:-
* Control the BP ASAP (using labetalol/hydralazine, with aim of SBP 140-150)
*seizure prophylaxis with MgSO4 (4g load 15/60, then 1g/hr for 24 hrs post delivery
*Careful fluid Mx given risk of APO, IDC, small bolus fluid if oliguric, limit fluids, CVC if oliguric and creatinine rising, and not responding to 2-3 small boluses of fluids
*Plan for delivery of the baby (ideally using a regional technique).
*This woman needs appropriate monitoring of her and her foetus, in a HDU setting or equivalent

Pre-Eclampsia
- defined as BP>140/90 and proteinuria (>300mg/24 hrs) for mild, and for severe, one of BP >160/110, severe proteinuria, neurological sx, LFT abn, platelets < 100, IUGR, etc... after 20/40 gestation, and resolving within 3/12 post-partum.
- Sx/Sx on assessment should include airway (oedema), CVS changes (APO), neurological (clonus/hyperreflexia), RUQ tenderness
- Ix should include FBE (Hb and plt count, haemolysis), EUC (creatinine), LFT (raised ALT/AST, BR), coags, G&H, urine dipstick, protein/creatinine ratio.
- Plan should be for regional anaesthesia, unless evidence of coagulopathy/HELLP
- Main risk to patient includes Eclamptic fit, ICH (especially under GA), APO, renal failure, and possibility of difficult airway (GA), as well as HELLP syndrome (and possible hepatic rupture!).
- She needs multidisciplinary Rx by senior staff in Obstetrics, Anaesthesia & Paediatrics.
(May-2008 Q7) 34yo woman presents at 36 weeks gestation with an anterior placenta previa and a caesarean section is scheduled. She has no intercurrent health problems. She has a history of two caesarean sections under regional anaesthesia. Describe and justify the changes this history would make to your pre-operative and intra-operative plan for CS.
- Placenta praevia is when part of the placenta covers the cervical os in the lower uterine segment. There is risk of placental adherence and bleeding (as well as uterine hypotonia).
- The history of ANTERIOR PP, and previous c/sections, means that this woman is actually at high risk of placenta ACCRETA (RWH protocol says to treat PP patients with 2 or more previous C/sections as the same).

Preop
- usual Anaesthetic/Obs history, particularly focussing on risk of bleeding, airway difficulties previously. Treatment of any preop anaemia.
- Further Ax or her PP by a senior Obstetrician. May need further U/S or MRI to ascertain possibility of PA.
- a full MDT approach, and should be admitted for this.
- plan for elective c/section in-hours, with support from blood bank, paediatricians, interventional radiology, and possibly other surgeons (ie, Gynae-Onc for laparotomy, prep for C/S hysterectomy).\]- Will likely be "early" (34-36/40 if no bleeding, or > 28/40 if bleeding), so pre-delivery steroids for foetal
- elective HDU referral for post-op care
- full discussion of risks with patient/partner, including hysterectomy, future fertility-saving procedures (potentially use of angio, or even conservative mx leaving placenta in situ)
- discussion re: anaesthesia plan (ie, invasive monitoring, large bore IV access, RA vs GA).

Intraop
- Plan for GA (esp if PA), otherwise potentially a CSE for PP only. Therefore premed, difficult airway trolley, BIS, etc.
- potentially 2nd anaesthetist
- invasive monitoring incl IAL, at least 2x large bore IV (+/- CVC)
- potential cell saver
- x-matched blood in OT
- pre-warming and temp monitoring
- use of drugs such as ergometrine, prostin, PGF2a
- postop disposition depending on surgery
- surgical changes include potential placement of ureteric stents, midline incision, possible need for elective c/s hysterectomy & interventional radiology
- may require prolonged postop stay in hospital if placenta left in situ
(Sep-2007 Q8) You are asked to provide epidural pain relief for a woman in labour. She is primigravida, and is 3cm dilated. Describe and justify both your choice of drugs for and the mode of administration of epidural analgesia in this situation.
- Would choose "low-dose" infusion or PCEA, with dilute LA (ie, ropivacaine 0.2%), and fentanyl (with initial loading dose of 10-20ml of LA and 100mcg fentanyl).
- Ropivacaine 0.2%, low dose to avoid motor block, less instrumental delivery, less hypotension. less cardiotoxic than bupivacaine if given IV, but also probably less potent (in terms of motor block).
- fentanyl, short-acting opioid which has probable supraspinal (bolus) and systemic effects (infusion). better quality block/analgesia (less patchy), and also better perineal/sacral analgesia before LA has had time to spread, enables usage of lower dose LA as above. short-acting and lipid-soluble so less risk of delayed respiratory depression, pruritis, N/V than morphine (for eg).
- LDI/PCEA have been shown to be superior to "traditional" top-ups (of 0.25% bupivacaine), with less LA used, less motor block, less intrumental delivery, hypotension. LDI vs PCEA - more lA/opioid used, higher neonatal resusc, and more "rescue top-ups required" in LDI grp.
CSE?
- thought potentially better in later part of 1st stage due to rapid analgesia. Also minimal motor block, so "walking epidural" - not commonly practiced here. no better analgesia/maternal satisfaction, but higher pruritis (cochrane). Risks include PDPH (deliberate dural puncture), risk of LA spread to intrathecally (total spinal), untested epi catheter (in case of emergency).
Mode & drugs would depend on institutional practice/protocols (and also on anesthetic cover overnight!).
(Sep-2006 Q7)While performing an epidural for labour analgesia in an otherwise healthy primigravida in first stage you inadvertently cause a dural puncture with the Touhy needle. Describe and justify your management of this complication.
- A dural puncture with an 18g touhey needle will cause a PDPH approx 50-80% of the time.
- Therefore, consideration needs to be given to immediate mx of labour analgesia, any immediate prophylactic measures, and surveillance for PDPH and appropriate treatment if it occurs.
- Immediate Mx
Choices are to resite epidural at different level (possible risks include another DP, or even "leak" of LA into intrathecal space and total spinal) or leave IT catheter in situ (good analgesia, but may be difficult depending on institution/protocols, as infusions may be dangerous, midwife top-ups should not be allowed, so will require anaesthetist-only top-ups, which may be time-consuming and difficult to do in practice, especially if hours of labour still to go. Potential other benefit is prophylactic.
- Prophylactic Mx
Little evidence that anything really reduces risk of PDPH. Many things tried, including epidural saline infusion, dextrans, epidural/IT morphine (probably just analgeses). Some evidence for
* leaving IT catheter in situ for 24 hrs
- level III evidence. Thought due to "inflammtory" response in dura, which seals up CSF leak. Risks include inadvertent use of IT catheter, forgetting to remove (!), and infection (may lead to meningitis, epidural abscess).
- Prophylactic EBP.
Probably the only thing shown truly to decrease rate of PDPH, but still not as effective as a TREATMENT EBP (After 24 hrs at least). Has some risks (ie, infection, further PDPH), and some women will not actually get a PDPH from the initial DP.
- Surveillance
If PDPH develops after 24 hrs, mx is either conservative (ie, bed rest, analgesia, fluids, weak evidence for caffeine) or invasive (EDBP).
Will depend on debilitation associated with PDPH, and also obviously the wishes of the patient.

I would, depending on the institutional protocols and staffing, leave an intrathecal catheter in for the duration of labour, with labelling "anaesthetist-only" for top-ups.
I would discuss with the patient/partner re: likely PDPH, and potential treatment options, and apologise for the complication.
Would ensure appropriate follow-up of patient, and adequate treatment if PDPH occurs.
Would also ensure reporting to departmental QA/MM committee.
(May-2006 Q15) Discuss the elements you consider important when obtaining consent for epidural analgesia in labor.
Discussion of technique, potential risks/benefits and alternatives (ie, PCA), ideally prior to procedure, but acknowledge that sometimes this is extremely difficult. The aim is to respect the patient's autonomy and to "do good".
- failure (20%), hypotension, itch, n/v
- PDPH 1/100
- temp N damage (1/5000) permanent (1/150,000)
- infection/bleeding which may cause the above
- LA toxicity
- increased risk of instrumental delivery, overall increase in duration of labour (minimal, measured in minutes only) no increase in c/section rate (although may increase for foetal distress, no change in neonatal apgars, and LESS naloxone usage than parenteral opioids). Increased hypotension, pruritis, motor block, urinary retention cf opioids. better analgesia. No change in long-term back pain.
- The procedure involves, positioning/monitoring/IDC, antiseptic and needle in back, potential for difficulties.
- Any questions?
(May-2004 Q14) Discuss the contra-indications to spinal anaesthesia for caesarean section.
Contraindications to spinal anaesthesia for C/section can be relative and absolute, and need to take into account the alternative of a GA.

Absolute
- severe coagulopathy/thrombocytopenia (ie, platelets < 50, INR > 2)
- maternal refusal
- Severe aortic/mitral stenosis
- Severe Hypovolemia
- raised ICP / intracranial lesion
- tethered spinal cord syndrome

Relative
- surgery may take longer than block (ie, abnormal placentation)
- coagulopathy/thrombocytopenia rel severe
- fixed cardiac output state as above
- Need for rapid anaesthesia due to foetal distress
- maternal sepsis / corioamnionitis (depending on how unwell)
- previous back surgery

Some are relative - coagulopathy/thrombocytopenia may well be due to pre-eclampsia, and still may be preferable to GA (given risk of airway oedema, ICH with induction of GA). Maternal sepsis may also be contraindicated, but not many studies support meningitis/spinal abscess with use in chorioamnionitis.
(Sep-2003 Q4) A 28 yo woman with a past history of two caesarian sections is at 34 weeks gestation with placenta praevia demonstrated by ultrasound. She is Jehovah's witness and will not accept blood products under any circumstances. She requires casesarian section. Do you consider regional anaesthesia a reasonable first option in this case? Give reasons.
See card 41 for placenta praevia.

In this case RA may be inappropriate. This woman is at HIGH risk of bleeding (due to placenta praevia and previous c/sections, may have P accreta, although unknown whether anterio/posterior). Given this likelihood, and no transfusion, she is at high risk of hypovolemia/hypotension, which will be aggravated by a regional technique. She is highly likely to require a GA at some stage, and is a risk for caesarian hysterectomy to stop bleeding.
Given the likelihood for HD instability, I would control the airway, so I could focus on HD management of this patient. She may also develop coagulopathy from bleeding, which will risk epidural haematoma.
(May-2003 Q7) A primiparous patient in active labour at 3 cm dilatation requests epidural analgesia. Examination reveals she has a temperature of 39.5 degrees. What impact does this fever have on your decision to provide epidural analgesia?
- Main risk of epidural analgesia in this situation is development of epidural abscess.
- secondary consideration is sympatholytic effect of epidural block, in someone who may be developing septic shock.
- Need to make an assessment of how systemically "unwell" the patient is, and also what the likely cause is - and this needs further investigation.
*If feels well, other vitals normal, and no obvious source of infection, could be given stat dose of antibiotics, given epidural (given long labour to go, and possible need for c/section at some stage now anyway), and then closely monitored in peripartum period for signs of neuraxial infection.
- if unwell... ie, tachycardic, hypotensive, flushed, feels unwell, rigors, etc, then may be safer to NOT perform epidural, and leave indwelling catheter.
- risk benefit of spinal vs epidural (especially if then imminent operative delivery) could be taken into account
- increased maternal fever means a higher likelihood of c/section, also a higher likelihood of neonate requiring Ix for sepsis. May also effect neonatal neuro development if continued high fever.
- would discuss with obstetric team to ensure they were assessing situation
- would discuss all risks/benefits with patient regarding possibility of spinal infection.
(May-2003 Q8) Which complications would you discuss with this patient when obtaining consent for an epidural? Include your estimates of the incidence of these complications.
See card 44.

from 47 though - particularly epidural abscess needs to be discussed.
(May-2003 Q9) An epidural is placed. Two days later she complains of back pain, urinary incontinence and a weak sensation in her right leg. How would you manage this problem?
This woman potentially has a serious complication (ie, epidural abscess with potential spinal cord compression or cauda equina syndrome), and so needs assessment ASAP.

Classical triad is fever, back pain and neurology, but uncommon to have all 3 present. Radicular pain may also be present. She may also have epidural haematoma.

Hx - pain, radiation, fevers, neurological (motor/sensory loss - UMN lesion suggests spinal cord compression, and LMN lesion could be either nerve root or periperhal nerve (peripheral nerve is more likely obstetric nerve palsy rather than neuraxial cause).

Ex- as above.

Ix - needs urgent imaging, MRI if available, otherwise at least CT. Bloods to look at signs of infection. She then needs urgent referral to a neurosurgeon depending on results. Within 6/24 is the timeframe for a good outcome if decompressed within this time.
(Sep-2001 Q7) After 10 hours labour, a healthy 28 yo primiparous woman at term requests epidural analgesia. Her cervix is 8cm dilated. Discuss the assertion that "a combined spinal epidural technique is a better choice of analgesia for this woman".
see card 42.

Advantages include quicker onset of analgesia (given late stage in 1st stage labour), good quality analgesia, high maternal satisfaction, and potentially better mobility ("the walking epidural").

Disadvantages include having an untested epidural catheter, potential for LA spread intrathecally, higher pruritis rate, similar maternal satisfaction to standard epidural, risk of PDPH (?higher?), possibly higher risk of foetal HR abnormalities (with no evidence of more worse outcomes in neonates).
(Sep-2001 Q8) She reaches full dilatation and delivers before any block is performed. Following delivery she has a retained placenta. Justify your choice of anaesthetic technique for manual removal of placenta.
A retained placenta can potentially be associated with significant "hidden" blood loss inside the uterus, as the placenta interferes with normal uterine involution. It can sometimes be the presentation of a placenta accreta.

Given there may not be any signs of significant blood loss <20% in a pregnant woman, this must be considered when anaesthetising for a retained placenta. If any suspicion of major blood loss, a GA with RSI should be performed.

A regional (spinal or epidural top-up) may still be preferred if patient appears well with minimal blood loss. This still means avoidance of a GA in a patient population with higher incidence of difficult intubation, and risk of aspiration. It also means a quicker return to bonding with their baby.
It may also be preferred in women who have other reasons not to give a GA (ie, severe asthma, pre-eclampsia, etc).

A GA should be performed (with RSI/ETT) if any concerns about blood loss, as this means better haemodynamic stability and also the airway is secured early (rather than worry about someone losing consciousness from hypotension midway through the procedure). The volatile agent may help uterine relaxation to deliver the placenta initially, and can be switched to 50% nitrous to aid in uterine tone afterwards.

More generalised things should include large bore IV access, bloods sent for FBE/Coags/Xmatch, left lateral tilt, aspiration prophylaxis,e tc.
(Apr-2000 Q4) You are asked to provide pain relief for a woman in labour. She is a primigravida, has twins and is 5cm dilated. You provide epidural analgesia. Describe and give reasons for your choice and method of delivery of drug(s).
See epidural questions previously.

a standard low dose epidural (via infusion or PCEA) is fine, as long as titration of the block is done slowly. A parturient with twins may have an exaggerated height of block for a given dose of LA, due to the extra compression of the epidural space by distended epidural veins.

The other thing is that this epidural should be followed up carefully to ensure it is WORKING - as this woman may need a rapid top-up for instrumental delivery of the 2nd twin, or a caesarian section. A CSE may therefore not be a good technique, as it is difficult to assess whether the epidural catheter is working if the spinal has already been performed.
(Apr-2000 Q5) The patient is now ready for vaginal delivery, but perineal analgesia is inadequate. Discuss the method you would recommend to remedy this.
This is a common problem as the LA has to descend into the sacral segments to provide analgesia for the perineum.

The options are
- epidural opioid (bolus fentanyl), effective.
- top-up with small bolus of concentrated LA (ie, 5ml 2% Lig) with patient sitting up. Effective, but more likely to get motor block, and potentially require instrumental delivery due to lack of motor power and sensation.
- i would discuss these 2 epidural options with the patient, and tend to use fentanyl first line, and strong LA second.
(Aug99) A 31 yo primigravida at 36 weeks gestation presents with a blood pressure of 170/110 mmHg, proteinuria, persistent headache and hyperreflexia. She requires delivery by caesarian section within 3 hours. How would you manage her blood pressure in the time before surgery?
These sx/sx describe severe PE which is an obstetric emergency. A BP of 170/110 or greater requires urgent treatment, ideally with IV agents.

1st line labetalol
- 100mg in 20ml NEAT. 20mg bolus x 2, then infusion 20mg/hr titrated. May need arterial line for monitoring.
2nd line hydralazine
- 20mg ampoule, 10mg bolus x 2 after 10-15 mins each. Then start infusion at 1-10mg/hr
- may need careful fluid bolus to ensure hypotension does not occur.
- aim for SBP 140-50, and no lower, due to risk of uterine hypoperfusion.

She also requires MgSO4 infusion for seizure prophylaxis, although this is not adequate for BP management.

If she is in labour, and there were no contraindications, an epidural placed before C/section may also be appropriate (may help BP control, uterine perfusion, as well as being mode of anaesthesia for c/section).
(Aug99) Justify your choice of anaesthesia for caesarian section.
As above.

- a regional technique is better.
* lower maternal mortality compared to GA
* better neonatal outcome compared to GA

In this particular case, a GA in a woman risks
- difficult intubation due to airway oedema, and possible bleeding on instrumentation
- severe hypertension on laryngoscopy and possible ICH (most common cause of death in severe PE)
- a titrated epidural would be ideal, could also be left in for postop analgesia if BP still difficult to manage. This would depend on no contrainidcations to neuraxial anaesthesia (such as low platelets or coagulopathy, other CVS disease, or maternal refusal).
(Aug99) If she had an uneventful general anaesthetic, but started convulsing in the recovery ward two hours post-operatively, how would you manage this?
This is likely an eclamptic fit, which is an emergency.
She needs attention to A, B, C firstly, then cessation of seizure (usually occurs spontaneously), then treatment (4g IV magnesium).

She may need intubation/ventilation if does not recover consciousness quickly, as risk of aspiration. she should be given 100% oxygen, and have appropraite monitoring attached.
Further mx may involve a CT scan if she doesn't recover consciousness (r/o ICH)
Also needs appropriate postop ward (ie, HDU).
(Jul98) A 34 yo woman requires repeat lower section caesarean section. Last time she had a Caesarean section her post operative course was complicated by dural puncture headache (following dural puncture with a 16G needle), as well as a deep venous thrombosis. She won't have general anaesthesia. How would you minimise the problem of post dural puncture headache on this occasion.
- This woman is having an elective c/section, so therefore my primary anaesthetic method would be a spinal anaesthetic. This has a lower incidence of PDPH than an epidural technique.

I would minimise the problem by
- technique.
Spinal vs Epidural/CSE. lower rate of PDPH.
- needle size/design
I would choose an "atraumatic" needle design such as a pencil-point Whitacre, and 27g would be the smallest needle gauge to use.
- procedure
I would try and align the bevel perpendicular to the dural fibres, although this is controversial.
I would ensure a senior anaesthetist performs the block, as experience is also a factor. A single spinal is better than several stabs!
- There have been several other methods to prevent PDPH or reduce its severity, without a lot of evidence for any of them. These include
*Caffeine (IV may be effective in this scenario)
*IV hydrocortisone for 48 hrs (reduces severity but not incidence after spinal)
* sumitriptan, spinal opioids, etc.

This woman also requires careful follow-up so that if she does get a PDPH, that prompt treatment (>24hrs though) is provided in the form of a EBP.
(Jul98) Describe and justify the regional anaesthesia technique you would choose for this woman.
.
(Jul98) Describe and justify your prophylaxis against deep venous thrombosis for her. (begin your answer by stating in just a word or two the regional anaesthesia technique you have chosen).
Spinal anaesthesia.

Neuraxial blockade has been shown to reduce risk of DVT in early studies (non-obstetric population).

She also requires chemical prophylaxis, and potentially mechanical modes also (at least TEDS, but calf compression devices may be more appropriate fro someone at high risk like her). She should have LMWH prescribed while she is in hospital (given at least 6/24 after the spinal block), and potentially for when she goes home, although I am unaware of any studies in the obstetric population regarding this (so would discuss with a Haematologist).

Risk (as per RANZCOG?)
*Low
- elective C/section with no risk factors, can be just mobilised early, and have hydration
*Moderate
- Age > 35, BMI>30, Para>3, varicose veins, current infection, Pre-Eclampsia, other majo systemic illness, emergency C/section = LMWH
*High
- 3 or more risk factors above, extended surgery, personal/family hx DVT, antiphospholipid syndrome = LMWH +/- TEDS

Apparently the only RCT on this issue used dalteparin 5000u (before surgery - although not done due to spinal), between 4-24hrs after surgery, for minimum of 4 days
(Guideline RWH).
(Apr96) You are asked to attend a patient who cannot move her legs twelve hours after a vaginal delivery. The last epidural top-up used 0.25% bupivacaine and occurred shortly before delivery. How would you manage her leg weakness?
Needs assessment ASAP, as risk is epidural haematoma.

Hx/Ex/Ix
- severity of weakness, and distriubtion, sensory changes?
- leg weakness present during labour?
- review anaesthetic chart for doses, adjuvants (ie, clonidine may prolong block)
- risk of intrathecal spread?
- other risk factors for haematoma (ie, anticoagulant use, low platelets) or infection (fevers, leucocytosis)
- may be dosing error? 0.25% more likely to get motor block, but not expected for 12 hrs afterwards with complete block

- Need to exclude haematoma/abscess with MRI (ideally) or CT (if MR not available).
- discuss with neurosurgeon's early if high suspicion, and begin to organise transfer
- need to report to epiduralist

may just be a matter of waiting/watching depending on findings, but a serious cause should be excluded if suspected.
(Apr96) Comment on the choice of 0.25% bupivacaine for her analgesia.
.
(Apr96) Discuss the methods which may identify an inadvertant epidural venous cannulation during labour.
- Blood seen
see blood in needle, or down catheter when try to thread in epidural space
may also be aspirated (although risk of epidural vein collapsing due to negative pressure)
if epidural catheter below level of insertion, then venous pressure may cause CSF (or blood) to track down the catheter, this can be tested before loading the catheter.
- NO Blood seen
Use of a test dose may pick this up. The traditional test dose of 3ml 2% LA with adrenaline was used, as would be enough to cause motor block if intrathecal, and would potentially cause an increase in HR (>10-15) due to adrenaline (1:200,000, or 5mcg/ml) and BP if given intravenously. Reported high false positive rate though, so not commonly used these days.
After a normal loading dose, there may be no block, in which case the epidural is either intravenous or outside the epidural space (not intrathecally though). Can then be resited.
Some people use fentanyl as a "test dose", as 100mcg fentanyl will give a sudden "rush", and analgesia quite quickly. not really studied as far as I am aware.
A higher dose LA (such as for c/section) may be given, which might cause LA toxicity if given intravenously - with signs of CNS toxicity (perioral numbness/tingling, tinnitus, seizures, LOC), or even cardiac toxicity (ventricular arrythmias, hypotension, CVS collapse).

epidural venous cannulation with the catheter can be decreased by injection with 5-10ml saline on finding the epidural space (through the needle).
(Oct-2012 Q3) 1. What are the prerequisites for separation from standard cardiopulmonary bypass after uneventful coronary artery bypass surgery? (50%). 2. What are the likely causes of hypotension in the immediate post-separation period? (50%)
1) separation from CPB involves restoration of the native circulation and ventilation.
Pre-requisities
- A, airway. usually still secure! But connect circtui if disconnected.
- B, re-start ventilator, incl alarms. Will initially need recruitment (care with inflation and LIMA graft - under vision from surgeon)
- C, circulation, the big one!
* Rate/Rhythm.
either a return to sinus rhythm with adequate HR (>70-80), or via pacing.
*Preload
- adequate payback, and partial clamping of venous line. ensure no ventricular distension
*Contractility
myocardium might be "stunned". expect if low pre-op function, or significant ischaemia/CBP time. May need inotropes, period of "resting" the myocardium prior to coming off, +/- IABP
*Afterload
- vasopressors ready as required
- for MAP > 70 or as per surgeon's goals

- D
Ensure adeaute anaesthesia/analgesia, check BIS, cgeck volatile back on.

- E.
rewarmed appropriately.

Need to check Hb, acid-base-electrolytes, coagulation/bleeding, and re-check haemodynamic stability before separation.

2) Causes of Hypotension
- Pump failure
Rate/rhythm disturbance, pacing problems, low contractility (long CPB time, ischaemia, graft occlusion, poor preop function, poor myocardial protection, reperfusion injury), valvular dysfunction
- Problems with preload
not enough filling
haemorrhage
cardiac tamponade
- Problems with afterload
vasoplegia (from CPB)
protamine reaction
anaphylaxis from anything
(Sep -2007 Q9) "It's no longer justifiable to use aprotinin during cardiac surgical procedures". Discuss.
- aprotinin a serine protease inhibitor, inhibits fibrinolysis, trypsin, kallikrein.
- reduces bleeding and reoperation for bleeding in many surgeries (cardiac, orthopaedic), and this is not challenged.
- earlier reviews suggested a LOWER rate of CVA, arrythmias, need for inotropes, but safety data started to accumulate.
- others say it is no longer justifiable due to increased morbidity & mortality (Mangano's 2006 study, and the BART study, 2008, both NEJM).
- Mangano, 4000 pts with prospective observational design, found 2x risk of ARF (dialysis), 55% increase in MI/CCF, 180% increase in CVA/encephalopathy, with a similar reduction in blood loss to lysine analogues.
- BART, only LARGE (2300) RCT, looked at aprotinin, EACA, TXA, in high risk cardiac pts. Showed lower risk of bleeding, but a higher mortality (when EACA & TXA combined, but not when each compared separately). Increased creatinine, but not RRT, and similar stroke/CVA.
- interestingly, 2012 saw the European medicines agency reverse their decision to ban Aprotinin, due to "problems" with the BART study (use of heparin, and measurement of anticoagulation - not stated when this occurred). Still not licensed in Aus.
(May-2007 Q14) An otherwise fit 30 yr old man is having microvascular reimplantation of his forearm. Describe methods available to optimise the perfusion of the perfusion of the reimplanted limb in the post-operative period.
.
(Sep-2006 Q3) Describe the cardiovascular changes which occur during clamping and unclamping of the supra-renal aorta during repair of an abdominal aortic aneurysm in a patient with normal ventricular function and outline your strategies to maintain critical organ perfusion during these times.
Aortic clamping produces many changes, but this depends on
* level of clamp (ie, infrarenal, suprarenal, supracoeliac)
*baseline splanchnic venomotor tone
*aortic disease (aneurysmal or occlusive - more likely to have collaterals)
*baseline cardiac function

Changes on clamping
- increase in aortic impedance, and LV wall tension
- increase in preload, with "passive recoil" of lower body venous system, and redistribution of blood to central blood volume
- usually an increased MAP due to this (approx 5%), but this will depend on splanchnic venous bed (will either accept blood and dampen redistributive changes or not) and cardiac function
*contractility and coronary flow may increase or decrease depending on cardiac function, which will effect the eventual CO. Ejection fraction usually falls by 10%.
*there may be regional wall motion abnormalities in up to 30%
*perfusion to lower limbs, kidneys, lower spinal cord will decrease markedly (RBF by 90%), with onset of anaerobic metabolism - this may also trigger a sympathetic reflex and hormonal response.

To maintain perfusion
- Coronary perfusion
may need to decrease afterload and preload with vasodilators such as GTN, ensure HR low-normal
- Renal / lower body perfusion
maintain normal MAP
may need filling if using vasodilators
non-pharm methods such as distal aortic perfusion techniques, or CSF drain if worried about spinal cord perfusion, minimise clamp time

Unclamping
- sudden decrease in afterload, and preload with redistribution of blood volume to lower body
- reperfusion of ischaemic legs results in anaerobic metabolites (K, H, adenosine, lactate) causing myocardial depression and vasodilation, exacerbating hypotension

perfusion
- filling and cessation of vasodilators prior to removing of clamp
- use of vasopressors to maintain MAP
- SLOW unclamping or intermittent unclamping
(May-2006 Q11) Discuss the principles underlying the management of a general anaesthetic for carotid endarterectomy.
Carotid endarterectomy is performed to prevent stroke from ipsilateral carotid stenosis (best results in the symptomatic patient with > 70% stenosis).

The general aims are
- to maintain haemodynamic stability to minimise risk of myocardial or cerebral ischaemia
- to maintain cerebral perfusion during cross-clamping, and facilitate cerebral function monitoring if appropriate
- to allow rapid emergence and assessment of neurological function

Preop
* Standard preop Ax (Hx, Ex, Ix and consultation), particularly focusing on cvs disease and neurological symptoms. The surgery is "intermediate" risk as per AHA guidelines for cardiac morbidity/mortality, but many patients will have significant IHD.
Ideal time for operation is within 2/52 of sx, so not much time for preop optimisation and assessment.
Other comorbidities (ie, HT, DM, CRI, etc) need to be assessed and optimised.
Airway assessment is important also as usual.

Intraop
- Prepare OT
particularly neuro monitoring available, blood x-matched
- Resuscitation drugs
need uppers and downers
- IV access
large bore due to risk of bleeding, carotid dissection
- Monitoring
standard, AND 5-lead ECG, A-line, cerebral function monitors if in use, BIS
- Equipment
FAW.
Induction/maintenance/emergence
- need "smooth" induction/emergence and HD stable maintenance, to decrease risk of cardiac/cerebral ischaemia. Control HR and BP on induction/emergence (may need adequate dose opioid, vasopressor, esmolol, lignocaine). Need to suppress stress response.
Maintain MAP at normal or 10-20% above baseline (Esp x-clamp phase)
use of remifentanil may help HD stability and extubation
use of desflurane may ensure rapid awakening for neuro exam, but beware rapid changes in concentation causing sympathetic stimulation.

Postop
- adequate BP control (risk of cerebral hyperperfusion syndrome, as well as bleeding)
- adequate monitoring on ward (airway, cardiac)
- analgesia to prevent stress response.
(May-2005 Q6) Compare the use of a pulmonary artery catheter and transoesophageal echo in evaluating cardiac function intraoperatively.
.
(May-2004 Q11) Discuss the strategies you would consider in order to protect renal function during a laparotomy for an abdominal aneurysm repair.
ARF complicates AAA surgery in about 3-13% of cases, depending on the level of the reconstruction and cross-clamp.

Preop
- identify those at high risk (or even those who may benefit from postponing and optimising better)
*elderly, pre-existing renal impairment, DM, use of contrast perioperatively, liver disease, heart failure
*level of aneurysm/planned clamp
*avoid nephrotoxins, such as diuretics, ACE I, NSAIDS, aminoglycosides
*minimise dehydration / fasting preop

Intraop
- "optimal systemic haemodynamics including maintenance of IV volume, is the most effective means of renal protection" (Miller's)
- maintain MAP and IV volume (especially during clamping/unclamping)
- inotropes if necessary to maintain CO
- minimise clamp time
- avoid nephrotoxins
- No evidence for use of dopamine, mannitol, frusemide (although these increase renal blood flow and urine output there is no data to suggest better outcome)

Postop
- avoid nephrotoxins again (esp NSAIDS)
- avoid hypotension postoperatively
- watch for other complications such as sepsis or abdominal compartment syndrome
- rapid assessment and diagnosis if oliguria or rising creatinine
(Sep-2003 Q14) All patients who present for coronary bypass surgery should be classified as ASA status 4 or 5 - discuss this statement.
.
(May-2002 Q1) A 68 yo man presents for repair of a rapidly expanding AAA. He has been a heavy smoker (80 pack years) until 5 months previously when he had a myocardial infarction. He has been treated for hypertension for the last 13 years and is currently taking atenolol, nitroglycerine and diltiazem. What clinically significant information can be obtained if a pulmonary artery catheter is placed for his operation and how would it influence the anaesthetic management?
PAC can measure
- Pulmonary artery pressures / PAOP
May give an indication of diastolic dysfunction and myocardial ischaemia - therefore can treat, especially on cross-clamping, may be able to direct vasodilator/venodilator use.
- CO / CI / mixed venous oxygen sats
all show adequacy of CO and oxygen extraction by body. May guide inotrope use, blood transfusion, etc.
- CVP another measure of R heart preload. May be useful to guide volume and venodilator therapy.
- in conjunction with MAP and CO, can calculate SVR, which will give an idea about need for volume/intropes/vasopressors to correct hypotension during op.
(May-2002 Q2) Compare the use of a pulmonary artery catheter and transoesophageal echo in evaluating cardiac function intraoperatively in this patient.
.
(May-2002 Q3) Describe the management of a rise in CVP from 15 to 23 mmHg two minutes after aortic cross clamping
Management of the increased CVP should include both diagnosis and treatment.

The aortic cross-clamp produces an acute rise in both afterload and preload (latter measured partially by CVP - preload to RV is anyway). This rise in CVP occurs, because blood is redistributed from the lower body venous system into the central blood volume. This rise depends on level of the clamp. Often, if clamp infra-coeliac, this blood is redistributed into the splanchnic venous bed (depending on pre-clamping tone), which will minimise the rise in CVP.

An acute rise of this magnitude is therefore likely to be abnormal, and potentially related to ventricular dysfunction (RV, but possibly both depending on MAP).

Treatment may involve unclamping (prior to any major incision on aorta), and slower clamping, with treatment with veno/vasodilators to reduce preload on clamping.
GTN/SNP may be required to reduce the consequences of raised afterload/preload on the heart (acute increase in myocardial oxygen demand, which may cause failure if cannot supply).
(Apr-2000 Q1) A patient is to have surgery for resection of a AAA. what advantages does TOE have over ECG monitoring for intra-operative myocardial ischaemia?
Intraoperative myocardial ischaemia may be seen by

ECG
- ST segment changes
- T wave changes
- conduction abn, arrythmias, HR changes
ST segment criteria
*horizontal/down-sloping ST >1mm
*up-sloping ST depression > 1.5mm
*elevation>1mm all 60-80msec after j-point

ECG lead sensitivity for ischaemia
- v5 most sensitive (75%), V5+V4 (90%), II, V4, 5 (98%)
- lead II and V5 only 80%
- ST elevation concurs with CA territory, but not ST D.

- TOE
*will see regional wall motion abnormalities (in segment of CA territory)
* MR, global dysfunction, diastolic dysfunction


Advantages of TOE
- most sensitive monitor of myocardial ischaemia (cf PAC, ECG 2, 5, 12 lead)
- SWMA's occur within secs and sometimes minutes before ECG changes
- SWMA's give idea of which coronary territory is involved
- also can give information about EF, valvular function, diastolic function, volume status
- useful in patients where ECG monitoring is difficult to interpret (ie, LBBB, pacemaker, etc)

Disadvantages
- costly
- requires expert training, and interpretation
- may cause oesophageal trauma
- SMWA's may not increase prediction of postoperative cardiac complications compared to ECG monitoring only (Miller)
(Apr-2000 Q2) Justify the measures you would use to minimise the risk of acute tubular necrosis if the surgeon is to clamp the supra-renal aorta.
Most responses included information about pharmacological as well as physiological methods. An explanation of the reasons for choosing particular strategies was required, including both before clamping and after cross clamp removal.
(Apr-2000 Q3) What are the relative merits of sodium nitroprusside vs glyceryl trinitrate for control of hypertension when the aorta is cross clamped?
.
(Jul98) A patient was scheduled for elective repair of an abdominal aortic aneurysm. During preparation for anaesthesia, the 8.5 french gauge introducer sheath intended for the internal jugular vein was inserted into the carotid artery. How could the risk of this incident occurring be minimised?
- correct learning of the anatomy
- Landmark techniques
*always staying lateral to pulse
*using seeker needle
*using cannula/needle - can then use cannula for checking pressure trace, and blood gas if necessary, before wiring, and importantly, before DILATING
- Use of Ultrasound to identify structures (and using within real time - seeing wire in vein before dilator used).
(Jul98) When the problem has been recognised, what is the appropriate management?
Depends on the urgency of the procedure - in this case, an elective procedure, could potentially be cancelled if necessary.
Also depends on any evidence of airway compromise (may need intubation if large haematoma develops).
Options
- removal with immediate pressure. This is best done in awake patient, so that it is known whether pressure too excessive (ie, causing cerebral hypoperfusion). It may be that a swan sheath will NOT close over with simple pressure.
- may need vascular repair by surgeon after induction of anaesthesia
(Jul98) Discuss the choice of the internal jugular vein as the initial site to attempt central vein cannulation in this patient.
Advantages
- ease of access to anaesthetist
- long straight vein
- can use ultrasound techniques easily
- low risk of infection / pneumothorax

Disadvantages
- stil has some risk of haematoma in neck (Esp carotid injury) with possible airway compromise
- uncomfortable for pt
- risk of pneumothorax cf femoral
- risk of damage to other structures (brachial plexus, vertebral artery, phrenic nerve, etc)
(Apr-2009 Q11) A 40 yo otherwise healthy male presents following a sub-arachnoid haemorrhage. He is scheduled for clipping of a middle cerebral artery aneurysm. Outline the major issues in providing anaesthesia for this patient and describe how you would address them.
This could be answered via Anesth/Surg/Pt fx, or Pt/Procedure/path or preop/intraop/postop.

Preop
- assessment of preop state, especially grade SAH, current conscious level, need for ICU, current BP
- Vasospasm propylaxis, seizure propylaxis started?
- Cx's of SAH
* CVS complications, hydrocephalus, seizures, Na imbalance

Intraop
- Risk of re-bleeding/aneurysm rupture on induction (laryngoscopy), and head pinning, determined by transmural pressure gradient, and balancing between this and maintaing CPP. Use of reminfentanil infusion to manage this, as well as pre-induction IABP
- Risk of cerebral hypoperfusion due to raised ICP
use of TIVA, or "low-dose" volatile, head up tilt, avoid neck ties, may require mannitol (usually after dura opened otherwise TMP gradient increases).
- Need for temporary clipping (decreases risk of aneurysm rebleed). Will require BP down just before temp clip, and raised just after to maintain collateral circulation
- Cerebral protection
Basically normoxia, normocarbia and normothermia. Also normoglycaemia, and maintain CPP. Volatiles may be protective but unproven. mild hyperventilation may help "relax" brain, as may mannitol (for surgical exposure). Avoid hypothermia (IHAST).
- Provide adequate surgical conditions
brain "relaxation" with mannitol. head up tilt. temp clipping as above. Keep patient still (paralysed!).

Postop
- vasospasm propylaxis
may need CVC for nimodipine if not already
hypervolemia / hypertension
- allow for rapid neurological assessment
use of sevo/des/propofol and remifentanil ideal for rapid awakening OR ICU for delayed emergence if high grade SAH.
(Oct-2008 Q10) Discuss the management of cerebral vasospasm following the coiling of a cerebral aneurysm.
Vasospasm occurs after SAH in up to 70% of patients (angiographically), but probably only 30% clinically. It can lead to "delayed neurological deficit", thought to be by vasospasm and cerebral ischaemia or infarction.
Interestingly the examiners report wanted a discussion of vasospasm and SAH despite the latter certainly not being asked.

Mx
- General
Confirm diagnosis
*may present as focal neurological deficit or decreased GCS, need to rule out other reversible causes such as hydrocephalus (CT), or seizures. Can be confirmed by tests such as TCD or Cerebral DSA.
*supportive measures such as intubation if required, ventilation to normalise CO2, support of circulation
*other ICU care such as DVT prophylaxis, PUD prophylaxis, correct electrolyte disturbance

Specific therapies
*HHH therapy
No good evidence for prophylaxis or treatment via Haemodilution, Hypervolemia & Hypertension therapy. Hypertension alone probably is effective. Either way side effects can include CVS complications and APO. Aims are to increase systolic BP to >160 (in occluded aneurysm) as blood flow is now directly related to driving pressure. CVP should be 8-12 (or some use PCWP >15), and hct about 30% for optimal microvasculature blood flow. can use normal saline to achieve this.
* Nimodipine
Evidence is for prophylactic therapy as opposed to treatment. given for 21 days, it improves neurological outcome, but not angiographic evidence of vasospasm (meaning that it probably works in other ways to its cerebral vasodilatory effect). No actual evidence for IV use (evidence is for oral 60mg 4/24), but commonly given this way by CVC.
*Invasive therapies
Cerebral angiography can be diagnostic and therapeutic. Balloon angioplasty +/- intrarterial vasodilators (papaverine or verapamil) are used.
*Novel therapies
AS yet unproven are magnesium therapy (low Mg predictive of poor outcome though), statins and endothelin antagonists.
(May-2008 Q12) Outline the issues involved in the pre-operative assessment of the patient presenting for transphenoidal surgery for acromegaly.
The issues to assess are related to the acromegaly (excess secretion of Growth Hormone by pituitary adenoma), as well as to the intracranial effect of the tumour itself (raised ICP? optic chiasm compression, causing decreased secretion of other pituitary cells - ie, hypothyroidism, hypoadrenalism, diabetes insipidus).

Acromegaly
- Airway
May have difficult BMV due to large jaw, macroglossia, presence of OSA, large nose. Difficult laryngoscopy due to macroglossia and growth of epiglottis. Difficult intubation due to overgrowth of vocal cords and possible subglottic stenosis.
- breathing
may have OSA.
-CVS
HT, LVH, IHD accelerated. Assessment of CVS status as per AHA guidelines, may require TTE, stress testing prior to surgery.
- Neurological
raised ICP due to mass effect discussed.
may have peripheral nerve compression, so documentation and positioning/padding important
- Endocrine
may have electrolyte disturbance, glucose intolerance, addison's disease, hypothyroidism. Needs testing and supplementation where appropriate.

Potentially requires perioperative treatment with octreotide to improve survival?
(Sep-2006 Q8) Describe the principles of cerebral protection in a patient with an isolated closed head injury.
The principles of cerebral protection are to maintain cerebral blood flow & OXYGENATION in the face of a raised ICP and decreased CPP, and by doing so, preventing SECONDARY INJURY. This is done by manipulating MAP, cerebral blood flow and cerebral metabolic rate.

Maintenance of Cerebral blood flow & oxygenation
- maintain arterial oxygen content
Prevent hypoxia (intubate and ventilate if needed)
Prevent or treat anaemia
- decrease cerebral oxygen demand
prevent hyperthermia
prevent seizures
decrease CMRO2 (anaesthetic agents).
normoglycaemia
- maintain CPP
CPP is the MAP - ICP/CVP (the greater of the 2), therefore involves both manipulation of the MAP and the ICP.
*Increase MAP using fluids/vasopressors as required, for a CPP >50-70, will likely need to monitor ICP directly to achieve this. CPP > 70 associated with ALI.
*Decrease ICP
ICP has 3 possible components, brain, CSF and blood (venous/arterial).
- Brain
surgical removal of tissue
manipulation of cell osmolality, using mannitol or hypertonic saline to decrease cell volume (ie, aim normal/high sodium 150).
- CSF
drain via EVD or removal by surgeon's intraoperatively
also partially determined by CBF
-Blood
*Venous
Head up tilt decreases cerebral venous pressure, as does care with neck ties, no PEEP, sedation/paralysis to prevent straining
*arterial
CBF depends on CO2 mostly, so maintain low-normal paCO2. Hyperventilation can be used in an emergency, but this may cause cerebral ischaemia if maintained too long
Also depends on CMRO2, so decreasing this will decrease cerebral blood volume and ICP.
- Skull
Can also be removed to decrease ICP. Prophylactic decompression associated with poorer long-term neurological outcome (DECRA trial).
(May-2005 Q1) Discuss the perioperative use of nimodpine for a patient undergoing clipping of a cerebral aneurysm.
NImodipine is a centrally-acting calcium antagonist used in the prevention/treatment of delayed cerebral ischaemia following SAH (ie, would not necessarily be used in clipping of an unruptured aneurysm).

Vasospasm is thought to occur due to the irritant effects of Hb breakdown products in the subarachnoid space, as volume of blood on CT correlates with the likelihood of vasospasm occuring. Vasospasm leads to cerebral ischaemia and poor neurological outcome. It occurs in about 30% of patients clinically (and up to 70% angiographically). It usually occurs within 3-14 days post-SAH (peaking 7-10 days).

Although it is thought it preferentially acts on intracerebral vessels to cause vasodilation to reverse vasospasm, it does not treat angiographically proven vasospasm, so may work by another unknown mechanism.

Should be commenced within 3-4 days of SAH (therefore may not be used preoperatively if operated on early), and continued for 21 days. The only large trial proving its reduction in poor neuro outcome used oral nimodipine 60mg 4/24 (no actual evidence for IV nimodipine).

May cause hypotension (particularly if given IV), so must be balanced against need for maintenance of adequate CPP.
(Sep-2004 Q7) Describe the pathophysiology and diagnosis of diabetes insipidus following head injury.
ADH is a nonapeptide hormone released from the posterior pituitary in response to either hypovolaemia or hyperosmolality of the plasma.

The synthesis and release of ADH might be disrupted in the setting of TBI, from causes such as direct trauma to pituitary, cerebral oedema (causing occlusion of vascular supply), cerebral ischaemia. A lack of ADH causes diabetes insipidus, due to loss of H2O retention in the collecting duct of the renal tubules.

- Diagnosis of DI can be made by Hx/Ex/ and Ix.
*Hx
- head trauma, and raised ICP, or known trauma to pituitary due to loss of pther pituitary hormones
- increased urinary frequecy, polydipsia (awake patient), or large urine volumes (clear) in sedated patient
*Ex
- large urine ouput of dilute urine as above
- signs of hypovolemia if fluid not replaced
*Ix
- measurement of serum and urine osmolality (serum will be high, and urine inappropriately low in face of high urine loss)
- Hypernatremia on EUC/ABG
- administration of ddAVP may confirm the diagnosis of "neurogenic" DI (from head trauma) versus nephrogenic DI (as will not respond to ddAVP).
_ imaging of brain (CT/MRI) may demonstrate pituitary infarction, or generalised cerebral oedema and lack of blood flow to brain (Cerebral DSA).
(May-2004 Q5) Discuss the methods you would use to reduce cerebral swelling during craniotomy for removal of tumour in an adult
Reduction of cerebral swelling and oedema can be achieved by modifying the plasma osmolality (to get changes in brain osmolality), or by changes in cerebral blood flow (manipulating CMRO2, paCO2).

Brain tissue
- can be reduced by reducing brain cell size using hypertonic solutions (which cause movement of water from brain cells into plasma, thereby shrinking the brain tissue). Most commonly used is mannitol (0.5g-1g/kg) or hypertonic saline.
- preoperative treatment with dexamethasone may help to reduce swelling from cerebral tumour or abscess, and is commonly used for this reason. high dose dexamethasone is also often requested by Neurosurgeon for any craniotomy, this may not have an immediate intraop effect, but may have an effect if the procedure is prolonged
- decreasing CMRO2/CBF
*mild hyperventilation will reduce CBF and CBV
*decreasing CMRO2 via anaesthesia, prevention of hyperpyrexia and seizures will reduce CMRO2. CBF will be matched in patients with autoregulation intact (as long as volatile agent < 1 MAC).
*decrease in cerebral venous congestion by head up positioning, lowering intrathoracic pressure (no PEEP), no neck ties, and maintaining adequate muscle relaxation (no straining).
removal of CSF
- may also be required, usually performed by surgeon via EVD or lumbar drain.
(Aug-2000 Q11) His peroperative blood pressure is 150/90 and the surgeon requests induced hypotension. Justify the level you would lower the blood pressure to and the method chosen.
. Within 20% of preop BP. Unlikely to cause ischaemia at this level (may even drop to this level at home or asleep, may have white coat hypertension).
- some level of autoregulation still occuring, would still be MAP > 70
- cerebral/coronary oxygen demand would hopefully be low due to anaesthesia/analgesia, therefore lower perfusion pressure may not cause ischaemia
- would monitor EG for signs of ischaemia and put BP up necessary
- lower BP will help to reduce bleeding, and bleeding/anaemia/hypovolemia may well have more adverse effects on patients coronary perfusion than slight hypotension.
(Aug-2000 Q12) Half way during the first-level fusion, the electronic anaesthetic machine diagnoses that it has an internal fault and without warning shuts all functions off including gas delivery, ventilation and monitoring. How will you manage this situation?
This is obviously a serious problem, with possible morbid consequences for the patient.

Firstly
- alert surgeons and anaesthetic assistant to problems, get help to manage this situation
- need to quickly switch to manual mode of ventilation with external oxygen source
- need to switch to IV anaesthesia via infusion pump if possible
- surgery should stop (unless active bleeding or something else needs control), while problem is sorted
- monitoring needs to be done, preferably on a portable monitor from recovery/similar
- Communication
Need clear communication within and outside of OT
- may need to switch to spont vent GA for safety
- may need to defer surgery unless problem fixed soon, but this is unlikely given halfway through fusion.
(Jul97) What are the advantages and disadvantages of the use of nitrous oxide in general anaesthesia for intracranial surgery?
- previously nitrous oxide thought to be useful in neurosurgery, due to rapid wake up and ability to assess neurology quickly.
- has some risk/cons
*risk of worsening pneumocephalus form open procedures, or enlarging gas bubbles in arterial circulation in neurointerventional procedures
*risk of PONV increased
*interferes with DNA synthesis, may cause macrocytic anaemia, prolonged administration may cause SCD of the spinal cord, poor wound healing
*potential for increased myocardial ischaemia due to increased homocysteine levels
*environmental effects (greenhouse gas).
* probably increases CBF/ICP in a synergistic way with other volatiles, therefore realtively contraindicated in patients with raised ICP

benefits
- rapid wakening for neurological ax (although rapid awakening is possible with des/remi these days)
- reduces volatile required
- analgesic effect, with maintenance of haemodynamic stability
(Apr96) A previously well 38 yo man presents for urgent clipping of a middle cerebral artery aneurysm. He has photophobia, and a blood pressure of 150/90. Give an account of how you would minimise the risk of arterial hypertension at tracheal intubation.
- Likely that this patient has had a SAH, or at least a "warning leak". The risk on induction of anaesthesia (particularly on laryngoscopy) is rupture of the aneurysm, which can have a much higher mortality if this occurs on induction.
- There is a need to balance the risk of aneurysm rupture versus the risk of cerebral ischaemia from lowering CPP. SAH causes an increase in ICP (especially higher grades), therefore CPP needs to be maintained to reduce cerebral ischaemia.
- Ideally BP is maintained a preoperative levels (or within 20% lower) on induction to reduce the risks of each problem
- There are many ways to reduce the hypertensive response to intubation
*adequate anaesthesia (dose of induction agent adequate for patient and comorbidities - may need vasopressor support)
*adequate opioid (I would use a remifentanil infusion, and titrate up to between 0.2-0.3mcg.kg.min for intubation, then be able to titrate lower between stimulation). Need adequate time for remifentanil to get into IV line and peak (takes about 90 secs after bolus dose).
- adeqate muscle relaxation. Sux if full stomach, as long as adequate anaesthetic agent (otherwise risks increasing ICP), otherwise high dose NDMR as long as intubation/ventilation not thought to be difficult
- ventilate to normocarbia or slightly lower
- Other possibilities include use of short-acting beta blocker (esmolol), or IV lignocaine just prior to intubation.
- a slick/smooth intubation performed by an experienced laryngoscopist is also valuable
(Oct-2012 Q15) 1. Classify the possible causes for patient awareness under general anaesthesia (70%). 2. Evaluate the evidence for the use of Bispectral Index monitoring in reducing the risk of awareness (30%).
1)
Awareness involves explicit recall of events under anaesthesia, and is about 1/1000 overall.

Patient factors
- unable to tolerate anaesthesia due to haemodynamic instability (ie, cardiac surgery, major trauma, caesarian under GA)
- increased requirements due to genetic factors (ie, patients with red hair have higher MAC)
- increased requirements due to pharmacological factors
*hyperthyroidism
*use of drugs chronically such as alcohol, smoking, or acute use of amphetamines
*preop anticonvulsants

Anaesthetic factors
- human error
*distraction (ie, forget to turn on volatile after a difficult rapid sequence induction)
*other, ie, being unfamiliar with machine/monitoring/infusion pump

Equipment malfunction
- infusion pump malfunctions during TIVA
- accidental disconnection/IV tissuing
- poorly seated/leaking/empty vaporiser, etc

2)
Evidence FOR
- the major trial is the B-Aware trial, a large multicentre RCT > 2000 patients, with high risk for awareness
- showed 80% reduction in awareness
- had structured interviews, multiple times
- BIS better than "standard care"

Evidence AGAINST
- B-Unaware trial
- arguably a lower risk population than the B-aware trial, and therefore not powered correctly
- single centre, large RCT
- BIS vs ETAG protocol (0.7-1.3)
- no difference

BAG-RECALL
- largest study (>6000 patients)
- multicentre, RCT
- BIS vs ETAG protocol
- no difference

Cochrane review
- evidence suggests BIS is better than "standard care" for high risk cases
- evidence suggests BIS or ETAG protocol are the same in high risk cases
- ETAG protocol is quite specific, with alarms set if MAC falls outside 0.7-1.3 range

Other
- BIS reduces anaesthetic recovery time, and anaesthetic usage
- more recent retrospective evidence suggests that patients who have BIS < 45 for a given length of time have a higher mortality (also "triple-low).

Other observational studies have shown higher awareness in TIVA, and use of NMBA's, suggesting BIS may be utilised in these groups.
(May-2010 Q1) List the complications associated with the use of limb tourniquets during surgery. (60%)
How can these complications be minimised? (40%)
a)
Local Effects
- Skin injury (poor padding, skin integrity, alcoholic prep)
- muscle injury ("tourniquet syndrome")
- Neurological injury (direct pressure, ischaemia - also increases with duration ~ 3x risk if 2.5 hrs TT)
- Arterial injury (rare, but high incidence of amputation if ocurs)

Systemic Effects
- Haemodynamic effects
*increased central BV / afterload, risk of embolism
*opposite on deflation, along with cardiac arrythmias, myocardial depression, increased CBF/ICP with increased ETCO2
-Metabolic effects
- Hypothermia
- Tourniquet pain / Hypertension

failure of tourniquet!

Patients at risk = PVD, CCF, peripheral neuropathy, known DVT, sickle cell

b) Prevention
- patient selection
- padding, cuff size, prep
- minimum pressure (SBP +100 (arm) / + 150 (leg)), or measuring "limb occlusion pressure"
- 90-120 mins TT, with reperfusion if needed again
- care with limb exsanguination/inflation/deflation
- monitoring for at risk poatients
- pre-emptive HD Mx (ie, fluid bolus/vasopressor, etc)
- tourniquet pain (clonidine, GBP, ketamine).
(Apr-2009 Q1) What do the terms decontamination, disinfection and sterilisation mean? (30%)
What measures should be in place to minimise the risk of transmission of infection to the respiratory tract of patients via anesthetic equipment (70%)
a)
Decontamination = removal of microorganisms/unwanted matter from contaminated materials or living tissue

Disinfection = inactivation of non-sporicidal bacteria/viruses by chemical or thermal means

Sterilisation = complete destruction of all microorganisms including spores

Asepsis = prevention of contamination of living tissues or sterile materials

b) Risk of transmission of infection to patient's respiratory tract by INSTRUMENTATION, the circuit and by health-care providers (should use standard precautions).

Equipment
- Instrumentation
* use single use items
*devices sited in upper airway need to be kept in sterile packaging prior to use
*FM's, laryngoscopes need to be disinfected, LMA's sterilised (laryngoscopes often sterilised).

Circuit
- protected by a new filter for each patient, otherwise needs sterilisation
- anything between filter and patient should be disinfected or single-use
- sampling lines ok as one-way flow
- ventilator bellows dsinfected
- bronchs need high level disinfection at minimum
(Apr-2009 Q2) What are the essential safety requirements for delivery of gases via anaesthetic machines and their associated breathing circuits in use in Australia and New Zealand? (do not include ventilators of scavenging in your answers).
The main safety aims are
- prevention of delivery of hypoxic gas mixtures
- prevention of barotrauma

*Gas supply to machine
- ensure no pipeline crossover (gas analysis)
- gas cylinders pin indexed
- reserve oxygen supply
- supply pressures (pipeline/cylinder)

*Machine
- O2 supply failure alarm
- oxygen last gas to enter common gas manifold (with largest knob and fluted design)
- anti-hypoxia device (either mechanical or pneumatic linkage with nitrous)
- vaporiser interlock
- pressure regulators
- O2 flush between CGO and check valve (ie, 100% O2 only)
- low pressure leak test
- working APL valve, insp/exp valves

*Breathing Circuit
- CGO 22/15mm connectors
- high pressure relief valve
- test circuit for leaks
- gas monitoring / gas analysis calibrated

Recommended
- alarm for sustained high or low airway pressures
- ON/Off switch and O2 Flush button should be protected from accidental activation

NOT PART OF ANSWER TO THIS QUESTION
Anaesthetic Machine Checks
- Level 1
biomedical engineering - appropriate gas analysis/no crossover
very detailed, required on any new system, and after servicing
- Gas delivery (minimising leaks, exclusion of crossover, gas analysis, oxygen failure alarms)
- Inhalational (no leaks, calibrated, thermostat, interlock)
- Infusion devices

Level 2
- performed at start of each session, responsibility of anaesthetist (but can be delegated)
- service label, high pressure system (pipeline/cyclinder, oxygen failure alarm, gas analysis)
- Low pressure (flow meters, anti-hypoxia device, vaporisers, precircuit leaks
- breathing systems (PP leak test, insp/exp valves, CO2 absorber)
- ventilation system
- scavenging
- emergency BMV
- other (suction, airway equipment, monitoring)

Level III
- before each anaesthetic IF
* vaporiser changed
*breathing circuit changed
*infusion device
(Oct-2008 Q1) Outline the operating principles and safety features of a modern variable bypass out of circuit vaporiser.
Out-of-circle vaporiser can be drawover or plenum (most common).

Operating Principles
- controlled splitting of FGF into bypass and vaporiser chambers (high resistance)
- full saturation of vaporiser flow is achieved due to large surface area (due to
Out-of-circle vaporiser can be drawover or plenum (most common).

Operating Principles
- controlled splitting of FGF into bypass and vaporiser chambers (high resistance)
- full saturation of vaporiser flow is achieved due to large surface area (due to wicks, etc)
- temp compensation occurs to change the splitting ratio (as vaporisation of the liquid volatile cools the liquid, lowering SVP, a bimetallic strip then changes the splitting ratio)
- stabilised for a range of low/high gas flows
- fully saturated gas then joins gas from bypass chamber to form the dialled concentration

Safety Fx
- Must be agent specific
- Interlock device
- locking spindle with o-rings
- anti-tilt measures
- visible agent level
- one-way check valve between O2 flush and vaporiser
- modern Aladin cassettes have virtually eliminated risks of tilting, as bypass chamber is within machine itself (also have colour-coded/magnetic coding)
- modern vaporisers cannot be filled during use (special guard in place)
(May-2008 Q1) Outline how oxygen is stored at the hospital and delivered to operating theatres up to and including the wall outlet. In your answer include features that ensure the safety of the system.
Oxygen is stored in a VIE
- holds up to 1500L liquid oxygen
- is main supply, but hospital often has back-up cylinders
- stored at -160 degrees (vacuum helps with this)
- pressure relief valves for pressure > 1700kPa
- differential pressure transducers / weigh scale under VIE help to determine when nearly empty
- pressure regulators for 400kPa
- often secondary VIE too and reserve cylinder bank (2 day supply)

Pipeline
- pressure regulators
- high quality copper to prevent rupture
- checked prior to use with gas analyser
- sleeve indexing system to avoid crossover

wall outlet
- sleeve index
- free of contamination

whole system has to be commissioned prior to use as epr australasian standards.
(Sep-2007 Q1) Explain the features of the electrical power supply to operating theatres that protect patients from marcoshock.
Macroshock refers to disturbance of neural, muscular or cardiac function due to application to the body of high current or voltage.
This may cause the following
0.5-1mA = level of perception
5mA = highest "comfortable" current
15mA = unable to let go
50-100mA = VF

versus microshock which is potential for VF to occur at 50-100micro amps delivered directly to heart.

Safety Fx preventing MACROshock
- Mains power earthed
leakage currents will flow to earth (in metal outer casing equipment)
- RCD
sense difference in active/neutral wires of 10mA, will shut off circuit within 40msecs
used in body-protected and cardiac-protected areas, but only when it is safe to cut power supply
- LIM
"floating circuit"
transformer is used to isolate active/neutral wires from earth
touching both will still cause electrocution, but either will not
only another leakage fault will cause current flow to earth
LIM monitors possible leakage current by testing for isolation (by intermittently injecting current and comparing resistance between 2 cicuits)
will alarm at >5mA leakage, but will not cut off power supply
also needs to be appropriate maintenance of electrical equipment as per australian standards
(May-2007 Q2) How does soda lime work? List the hazards associated with its use.
Chemical reactions
- 80% Ca(OH)2, 15% H2O, 4% NaOH (Catalyst)

1. CO2 + H2O = H2CO3
2. H2CO3 + 2NaOH = Na2CO3 + 2H2O + HEAT
3. Na2CO3 + Ca(OH)2 = CaCO3 + 2NaOH + ethyl violet indicator (turns purple as gets more acidic)

newer CO2 absorber is Ca(OH)2 + CaCl, and has minimal reactivity with volatiles (virtually eliminates CO and Compound A production), but less absorptive capacity and higher cost.

Hazards
- General
circuit leak, rebreathing if exhausted, increased resistance to breathing
- Interaction with volatiles
CO production (low flows, dessicated baralyme, desflurane)
Compound A (low flows, baralyme, fresh soda lime?)
Fire (dessicated baralyme, increased CO2 production - not reported with soda lime)
(May-2007 Q6) List the patterns of peripheral nerve stimulation that may be used to monitor non-depolarising neuromuscular blockade during anaesthesia and describe how each is used in clinical practice.
.
(May-2007 Q9) The T-Piece is obsolete in modern anaesthesia practice. Discuss.
Not in kids!
(May-2005 Q6) Compare the use of a pulmonary artery catheter and transoesophageal echocardiography in evaluating cardiac function intraoperatively.
.
(May-2005 Q8) Draw a circle breathing system and give reasons for the location of the components.
a semi-closed circular system, with 7 components
3 rules to prevent rebreathing
1. unidirectional valve between pt and reservoir bag both limbs
2. FGF AFTER exp valve
3. APL valve BEFORE insp valve

- Y-piece, inspiratory/expiratory limbs with valve
a semi-closed circular system, with 7 components
3 rules to prevent rebreathing
1. unidirectional valve between pt and reservoir bag both limbs
2. FGF AFTER exp valve
3. APL valve BEFORE insp valve

- Y-piece, inspiratory/expiratory limbs with valves, FGF inlet, CO2 absorber, Bag & APL valve
- prevents CO2 rebreathing by absorption, and allows low gas flows

*Unidirectional valves
most efficient if placed at y-connector, but too bulky, needs to allow inspection of valve function
*FGF inlet
after APL valve, and CO2 absorber to maximise efficiency and minimise venting, and before Insp valve
*Y-piece
as close to patient as possible to minimise dead space
*limbs
minimal compliance to avoid loss of delivered TV, and clear for condensation
*APL
just downstream of exp valve, and before absorber
*bag, same side of exp valve as APL valve
*CO2 absorber
after APL so expired gases removed beforehand
before FGF inlet
this increases efficiency of CO2 absorber
(May-2005 Q15) Discuss the advantages and disadvantages of intra-operative blood salvage
Indications for Cell Salvage
- anticipated blood loss > 1L or > 20% BV, or if > 10% of patients require transfusion for a specific procedure
- pt fx such as preop anaemia, rare blood group, JW

Advantages
- reduces need for allogenic blood transfusion up to 40% (cardiac, vascular, major JOINT, paediatric craniosynostosis/cardiac/scoliosis)
- cost effective for these surgeries
- useful in patients with rare blood groups, JW, etc
- controversial, but now thought ok to be used in Obstetrics, Cancer (specifically prostatectomy/cystectomy/hysterectomy) - need LDF
- oxygen transport by autologous blood better

Disadvantages
- relative CI's (cancer recurrence, AFE, sepsis)
- risk of AE
- low grade DIC (microemboli)
- coagulopathy (red cells only)
- need special training to do, and special equipment, initial cost outlay
- ALI, AKI
(Sep-2004 Q6) What are the considerations in setting the fresh gas flow rate when anaesthetising an adult with sevoflurane and nitrous oxide using a circle absorber system?
Considerations
- need for rapid change in depth of anaesthesia (esp induction./emergence = higher flows)
- also depends on alveolar ventilation (wash in / wash out
- depends on concentration of anaesthetics dialled up (ie, overpressure for induction will require less FGF)
- stable depth can decrease FGF significantly (for decreased costs, environmental effects due to less pollution)
- lower FGF allows better humidifcation of gases (via HME)
- CO2 absorber will have effect (can have low flows if fresh, otherwise need higher flows to prevent rebreathing)
- if leak present, higher FGF will be necessary (ie, in children, or poorly fitting LMA)
- Sevo manufacturers suggest min flow of >2L to avoid production of compound A with CO2 absorber (more likely with low FGF, higher temp, higher MV, dessicated CO2 absorbent)
- FGF should also depend on oxygen uptake by patient (need at least to = uptake to prevent hypoxic gas mixture, ie, > 250ml/min). There will also be significant uptake of nitrous and 3% metabolism of sevo
- ETAG monitoring will also allow lower FGF (otherwise potentially unsafe).
(Sep-2004 Q15) Describe the function of the anaesthetic circuit shown in the diagram (Mapleson F - Jackson-Reece modification of the Ayers t piece)
The T-piece is a modified Mapleson F circuit.
- valveless
- low resistance
- light weight and portable
- simple, good for <20kg

Need high FGF to prevent re-breathing
- 2-3 x MV for SV
- ~ 2 x MV for IPPV
- tubing and bag needs to be greater than
The T-piece is a modified Mapleson F circuit.
- valveless
- low resistance
- light weight and portable
- simple, good for <20kg

Need high FGF to prevent re-breathing
- 2-3 x MV for SV
- ~ 2 x MV for IPPV
- tubing and bag needs to be greater than 1/3 of TV (acts as a reservoir for oxygen, but too large and will cause rebreathing)
- bag allows visualisation of ventilation, as well as addition of CPAP and PPV

- inefficient in >25kg, HIGH FGF required = high cost, pollution, difficult to scavenge
(May-2004 Q15) "Use of bis or other similar monitor should be the standard of care during total intravenous anaesthesia". Discuss this statement.
DOA monitoring during TIVA makes logical sense - if it is believed that they are accurate methods of measuring DOA.

TIVA has an inherent risk of patient awareness, due to the possibility of not delivering drug to the patient and not realising this - ie, IV tissues/falls out, drug refluxes up IV line - will not necessarily alarm, whereas traditional volatile anaesthesia - if the circuit disconnects there will be an alarm (may not alarm if volatile just not turned on, etc).

In one large observational study, 1.1% of TIVA pts vs 0.6% of traditional pts had AWR.

Advantages
- an "additional" monitor to ensure drug delivery, as long as alarms are set!
- gives some idea of "effect-site" drug
- has been shown to reduce amount of drug (volatile or propofol) needed
- reduces risk of awareness in "high risk" patients versus "standard care" (not necessarily TIVA, but other cases such as GA C/s, Rigid Bronch, Trauma, etc).

Disadvantages
- BIS number does not always correlate with medications given (no correlation with ketamine, nitrous, relaxants)
- TIVA not included as high risk group in major RCT's, so the question is unanswered, particularly for awareness
- The standard of care should be a vigiliant anaesthetist ensuring adequate drug delivery, as the main risk is not delivering the drug in TIVA
- other large RCT's have not shown a reduction in awareness when comparing BIS to ETAG monitoring, but this does not really apply to TIVA.
- some small RCT's suggest a reduction in awareness with BIS and TIVA, but not strong enough to make it standard of care.
(Sep-2003 Q5) What is micro-shock? Outline methods designed to prevent it occurring in the operating theatre.
Microshock, also called microelectrocution is caused by current delivered directly to the heart (ie, via a CVC or pacemaker wires).
- caused by current as low as 50-100 micro AMPS (vs ~ 50 MILLI AMPS for macroshock
- requires greater level of protection than macroshock
- cardiac-protected areas, require Equipotential Earthing (as well as either LIM or RCD).
*equipotential earth employs a large diamter cooper cable link to all earth points in the area, which eliminates potential differences between the points

Methods to prevent microshock
- perform in cardiac-protected area with EE
- care with extension cords and junction boxes (As can defeat EE)
- use inly maintained equipment
- wear gloves when touching cardiac/CVC wires
- monitor ECG
- don;t touch a piece of electrical equipment and CVC/wire at same time.
(May-2003 Q15) Describe the features of an ideal disconnection alarm.
.
(May-2001 Q4) Discuss the advantages and disadvantages of: Directed blood donation (i.e. from a known donor to a specific recipient).
still reports of errors with donation, and therefore incompatability
directed donors less likely to answer "risk" form correctly, so potential higher risk of infection transmission
still allogenic blood transfusion with associated risks
costly
(May-2001 Q5) Acute normovolaemic haemodilution.
.
(May-2001 Q6) Intra-operative blood salvage.
.
(Apr99) Describe the use of a nerve stimulator to monitor neuromuscular blockade during general anaesthesia for intracranial surgery.
- idea here is NO MOVEMENT at all, as any straining/coughing/movement may have severe consequences (especially microvascular aneurysm surgery).
- therefore need a mode that monitors "deep" blockade, rather than light.
- Post-tetanic count would be most appropriate, as this monitors deep NMB (when there is not TOF count). A TOF count (at adductor pollicis) may mean there is recovery of diaphragm (which recovers earlier), so this is too light.
- Post-tetanic count uses a tetanic stimulus (50Hz) for 5 secs, then single twitch at 1hz for 15 beats. It works by increasing calcium at the motor end-plate (nerve terminal), for increased release Ach release.
- the 1st count for TOF is approached when PTC approaches 8-10.
(Jul97) Describe the principles of measurement by which oxygen concentration in inspired gas cen be measured, where mass spectrometry is not available.
.
(Jul97) Contrast a bourdon gauge with a variable orifice flowmeter for the measurement of gas flow.
.
(Apr97) How does pulse oximetry differ from laboratory co-oximetry performed on a blood specimen?
.
(Aug96) Describe the principles of measurement used in a volatile anaesthetic agent monitor in an operating theatre where mass spectrometry is not available.
.
(Aug96) Describe the safety features which may be incorporated in the flowmeter bank of a modern anaesthetic machine.
.
(Aug96) Explain why end-tidal CO2 tension may differ from arterial blood carbon dioxide tension.
.
(Aug96) How should a laryngeal mask airway be processed to avoid cross infection between patients.
.
(Apr96) How would you check a circle breathing system with carbon dioxide absorber, but without a mechanical ventilator, from the common gas outlet to the patient connection?
.
(Oct-2012 Q12) Describe the anatomy of the transversus abdominis plane relevant to regional anaesthesia (70%). List the complications associted with TAP block (30%).
(Oct-2012 Q12) Describe the anatomy of the transversus abdominis plane relevant to regional anaesthesia (70%). List the complications associted with TAP block (30%).
a) T8-9 - T12-L1.
Blocks segmental nerves hopefully prior to lateral branches, but certainly prior to anterior branches.  TAPlane.
b)
Failure, LA toxicity (relatively common due to high vascular uptake with this block), trauma to structures such as blo
a) T8-9 - T12-L1.
Blocks segmental nerves hopefully prior to lateral branches, but certainly prior to anterior branches. TAPlane.
b)
Failure, LA toxicity (relatively common due to high vascular uptake with this block), trauma to structures such as blood vessels, peritoneum, bowel
(Apr-2009 Q10) Draw a diagram illustrating the bronchial anatomy to the level of the lobar bronchi (50%) and describe how you would use a fibreoptic bronchoscope to correctly position a R sided DLT (50%)
a)
Bronchial Anatomy
- Trachea approx 15cm in length, from cricoid at C6, to carina at approx T4-5.
- RMB at 25degree angle, 2.5cm long, divides into 3 lobes, RUL/RML/RLL (with 10 segments, APA LM AMAPL)
*RUL comes off first, and variably, and is the
a)
Bronchial Anatomy
- Trachea approx 15cm in length, from cricoid at C6, to carina at approx T4-5.
- RMB at 25degree angle, 2.5cm long, divides into 3 lobes, RUL/RML/RLL (with 10 segments, APA LM AMAPL)
*RUL comes off first, and variably, and is the only lobe with 3 segments ("trifurcation"). The RMB continues as bronchus intermedius, before dividing into RML/RLL.
- LMB is at 45-50 degrees, and 5cm long. divides into LUL/LLL, LUL has lingular lobe (LUL = APA, Ling = SI, LLL = ApmAL

b)
- place the R DLT as usual
- ventilate, then inflate tracheal cuff until no leak
- clamp tracheal lumen/open
- inflate bronchial cuff until no leak through tracheal lumen
- would then FOB down tracheal lumen, to hopefully see carina, and blue bronchial cuff rim just below in RMB
- FOB down bronchial lumen, aiming to see RUL trifurcation through RUL murphy eye
- if not seen, would go down to secondary carina (RLL/RML), this should be aligned to where the RUL branches off the RMB. Would then deflate cuffs, and rotate until my RUL lumen was in line with this carina - hopefully will come into view!
- otherwise, may have to withdraw tube, and go down bronchial lumen to ensure normal positioning of RUL before replacing
(Sep-2007 Q10) Describe the blood supply to the spinal cord. Explain the determinants of spinal cord perfusion.
(Sep-2007 Q10) Describe the blood supply to the spinal cord. Explain the determinants of spinal cord perfusion.
Arterial Supply
- Ant Spinal Artery (formed via 2 branches either side from vertebral arteries)
- runs in anterior median Fissure
- supplies 2/3-75% of the anterior spinal cord (Millers, A&A, Examiners Report WRONG)
- serially reinforced by spinal bra
Arterial Supply
- Ant Spinal Artery (formed via 2 branches either side from vertebral arteries)
- runs in anterior median Fissure
- supplies 2/3-75% of the anterior spinal cord (Millers, A&A, Examiners Report WRONG)
- serially reinforced by spinal branches of vertabral, deep cervical, intercostal/radicular branches, lumbar
- main branch (radicular) is the Artery of Adamkiewicz, usually arises T10-L1 on L but variable
*probably supplies much of the lower 2/3 of the spinal cord

Posterior Spinal Arteries
- 2-4, arise from PICA's
- supply the posterior columns (25%-1/3 posterior cord)
- reinforced by similar segmental branches, and AoAkwz

There is no anastmaoses between posterior/anterior blood supply, so thrombosis can cause infarction of large segments of cord (ie, ASA syndrome).

The determinants of spinal cord perfusion are similar to cerebral perfusion
- SCPP = MAP - CSF pressure/CVP

MAP
- function of CO and SVR
- may also particularly be changed in aortic surgery where a cross clamp reduces perfusion below the level of the clamp (particularly thoracic lesions)
- MAP may also fall with anaesthesia (GA/RA), sepsis, hypovolemia, etc. Cord infarction has been described in major blood loss and hypovolemia.
- perfusion also regulated by CO2/O2 like cerebral perfusion

- CSF pressure
increased by cord ischaemia/oedema
cord injury
epidural haematoma
can be reduced by decreasing metabolic requirements of spinal cord (local or systemic cooling, anaesthesia - ie, thiopentone)
decreasing CSF pressure via CSF drainage

venous pressure/CVP
- higher in PEEP, poor positioning (ie, prone), straining (not relaxed)
(May-2008 Q3) Describe the anatomy of the brachial plexus relevant to performing and interscalene block under ultrasound guidance. Include a drawing of the real or sono-anatomy you would see in a transverse view of the brachial plexus at the point of needle insertion.
The interscalene block targets the part of the brachial plexus where the roots form the upper, middle and lower trunks between the anterior and middle scalene muscles.
- lateral/inferior to posterior border sternocleidomastoid
- lateral/posterior to IJV
The interscalene block targets the part of the brachial plexus where the roots form the upper, middle and lower trunks between the anterior and middle scalene muscles.
- lateral/inferior to posterior border sternocleidomastoid
- lateral/posterior to IJV/Carotid (and vagus within carotid sheath)
- phrenic nerve CLOSE (medial to anterior scalene, and anterior to cervical sympathetic chain), and is almost 100% blocked according to some articles (along with 100% Horner's)
- superficial to vertebral artery, TP of cervical vertebrae
- level of C6 (cricoid), and scan laterally, may need to scan inferiorly to get supraclavicular "bunch of grapes", then follow superiorly to get "seagull sign"
- good for shoulder/upper arm surgery, as blocks suprascapular N, but may miss lower trunk, and not get hand adequately
(Sep-2005 Q4) Describe the anatomy of the trigeminal nerve relevant to local anaesthesia for dental extraction.
.
.
(May-2005 Q3) Outline the anatomy of the right internal jugular vein as it is relevant to your preferred method of percutaneous cannulation.
.
.
(Sep-2004 Q8) Describe the anatomy relevant to providing an ankle block for surgery on the big toe.
(Sep-2004 Q8) Describe the anatomy relevant to providing an ankle block for surgery on the big toe.
Note this question is specifically asking about nerves in an ankle block that supply the great toe.

Tibial N (eventually plantar nerve)
- behind medial malleolus
- posterior to PTA/PTV, FDL and TP
- anterior to FHL
- supplies most of sole

Superi
Note this question is specifically asking about nerves in an ankle block that supply the great toe.

Tibial N (eventually plantar nerve)
- behind medial malleolus
- posterior to PTA/PTV, FDL and TP
- anterior to FHL
- supplies most of sole

Superifical Peroneal
- most of dorsum of foot (except small area between great toe/2nd toe = deep peroneal)
- between anterior edge tibia and lateral malleolus

Deep Peroneal
- small area of skin between great toe/2nd toe
- lateral to ATA or DP, and between EDL and EHL

Sural
- lateral edge foot, little toe
- blocked between achilles tendon and lateral malleolus, very variable

Saphenous
- branch of femoral, usually supplies medial ankle, some foot, variably great toe
- between anterior tibial edge and medial malleolus, posterior to vein (but "neural network")
(May-2004 Q10) Describe the anatomy relevant to a digital nerve block of the ring (4th) finger
.
apart from picture shown, what must be remembered is the nerve supply of the 4th finger, medial side being ulnar and lateral being median/radial

- main nerves are palmar side, and extend to supply dorsal part of distal phalanx, smaller nerves dorsally only extend proximally
(May-2003 Q4) Describe the anatomy of the ulnar nerve, relevant to supplementation of an interscalene brachial plexus block which is inadequate for an operation on the hand.
shown are relevant blocks at the elbow
shown are relevant blocks at the elbow
(May-2002 Q15) Describe the anatomy of the epidural space.
.
.
(May-2001 Q12) Describe the cartilages of the larynx.
(May-2001 Q12) Describe the cartilages of the larynx.
.
.
(May-2001 Q15) (Sep-2003 Q6) Describe the anatomy of the penis relevant to providing regional anaesthesia for circumcision in an adult.
(May-2001 Q15) (Sep-2003 Q6) Describe the anatomy of the penis relevant to providing regional anaesthesia for circumcision in an adult.
.
.
(Apr99) Describe the anatomy of the brachial plexus relevant to risks associated with the supraclavicular block.
As pic shows, supraclavicular block targets the trunks of the brachial plexus (although as this pic shows, ulnar nerve sometimes comes of early and is missed).  The divisions start at the lateral border of the 1st rib).
As pic shows, supraclavicular block targets the trunks of the brachial plexus (although as this pic shows, ulnar nerve sometimes comes of early and is missed). The divisions start at the lateral border of the 1st rib).
(Jul97) Describe the anatomy of the 6th intercostal space at the angle of the rib, relevant to an intercostal nerve block.
.
(Apr-2009 Q6) Describe the physiological effects of pneumoperitoneum with CO2 for laparoscopic surgery.
Pneumoperitoneum is caused by pumping gas into the peritoneum (allows surgical access and vision during laparoscopy). Gas is Co2. Effects are from pressure and CO2 absorption.

CVS
- low pressure (<10mmhg) increase VR/CO
- higher pressures decrease VR/CO, and increase SVR. BP initially maintained. pressure > 20 will result in fall in BP too.
- CO2 absorption may stimulate SNS further

Resp
- decreased FRC with decreased lung compliance, increased AWR/PVR
- respiratory acidosis if MV not increased

Decrease in splanchnic and renal blood flow (potential for reduced urine output).
increased renin-AT-aldosterone-ADH, to icnrease SVR/fluid reabsorption, decreased UO.

increased CBF with increased CO2, increased ICP

- apparently there is an acute phase reaction, but release of proinflammatory cytokines are lower with CO2 than air/helium.
(Oct-2008 Q5) An otherwise well 60-year-old man is ahving a radical prostatectomy. list abd briefly evaluate strategies to prevent peri-operative thromboembolism.
.
(May-2005 Q2) Discuss ways in which the risk of deep venous thrombosis can be minimised in adult patients having intra-abdominal surgery.
.
(May-2004 Q4) An 85 year old female presents with a 3 day history of bowel obstruction. The duty surgeon wants to perform a laparotomy as soon as possible. Discuss the main factors determining the optimal time for anaesthesia and surgery.
.
(May-2002 Q4) A morbidly obese patient (160kg, 165cm) with sleep apnoea requiring the use of CPAP mask develops actue cholecystitis. He is febrile and sweaty, with warm peripheries. Blood pressure is 110/70. Heart rate is 110/min. He is scheduled for urgent cholecystectomy. His saturation on arrival in the operating theatre is 93% on O2 (nasal prongs 2l/min). Discuss the advantages and disadvantages of an awake intubation in this patient.
.
(May-2002 Q5) Outline the implications of this operation being performed as an open rather than a laparoscopic procedure in this patient.
.
(May-2002 Q6) Justify your plan for postoperative respiratory care of this patient following an open procedure.
.
(Aug-2000 Q13) A patient is referred to see you six weeks before her scheduled cholecystectomy. The significant features of her history and exam are: Age 42, weight 140kg, height 171cm. Medication: oral contraceptive pill. Allergies: peanuts (severe asthma); some cosmetics; penicillin (itchy rash). Smokes 60 cigs per day for 20 years. Discuss the anaesthetic related risks associated with laparoscopic as compared to open surgery in this patient.
.
(Aug-2000 Q14) What will you advise her regarding the risks of continuing to smoke prior to the operation?
.
(Aug-2000 Q15) Discuss premedication in this patient.
.
(Apr99) A 52 yo man, height 1.75m weighing 130kg presents for laparoscopic cholecystectomy under GA. History and examination reveal no other abnormality. How does this patients obesity influence your anaesthetic management up to the time of the first incision?
.
(Apr99) Explain the physiological effects of a carbon dioxide pneumoperitoneum in this man.
.
(Apr99) Open cholecystectomy becomes necessary. The surgeon requests that you place an epidural catheter for postoperative analgesia. Outline the issues which should be considered in responding to this request.
.
(Aug96) An otherwise healthy 25 year old woman undergoes hysteroscopic endometrial ablation. After an uneventful general anaesthetic she is slow to wake in the recovery ward and becomes restless and confused. How would you diagnose the cause of this delayed recovery?
.
(Aug96) While you are assessing her, she begins to convulse. Describe your management.
.
(Aug96) Compare the pharmacology of diazepam and thiopentone, with respect to the management of the patient's convulsions.
.
(Oct-2012 Q13) Discuss the key areas of concern in your preoperative assessment of a patient for excision of a large tonsillar mass.
requires a targeted Hx/Ex/Ix to determine threat to arway patency and determining risk of perioperative airway obstruction.

Hx
- acute or chronic history
- "threats" = dyspnoea, stridor, voice change, hoarseness
- functional capacity
- how urgent is this?
- will depend on whether patient is child or adult what the intervention may involve
- anaesthetic hx, medications, allergies, recent chemo/radiotherapy

Ex
- stridor, tachypnoea
- neck swelling, airway deviation
- usual and unusual airway assessment

Ix
- pathology? ie, cancer?
- nasendoscopy - define extent of lesion, and difficulty in getting around it or with standard airway mx
- CT scan - may further define airway obstruction or extent of mass

Would need to consult with surgeon's prior to ay anaesthetic to determine intervention, as well as their thoughts on airway difficulty, etc.
(Apr-2009 Q12) Describe your management of a patient who has had a total thyroidectomy who develops respiratory distress in the recovery room.
.
(May-2008 Q9) A 25 yo man is to have laser surgery for a vocal cord papilloma. What are the hazards associated with the use of laser in this situation and how can they be minimsed?
.
(Sep-2004 Q10) Justify the use of a laryngeal mask airway in a 25 yo, 80kg man having general anaesthesia for removal of 4 molar teeth.
.
(Sep-2003 Q13) How can recurrent laryngeal nerve function be assessed in the postoperative period?
.
(Sep-2002 Q13) Discuss the issues specific to GA for myringoplasty
.
(Aug99) A healthy 25 you, 80kg man had general anaesthesia for the elective removal of 4 molar teeth. The anaesthetist uses a laryngeal mask airway (LMA). Justify the use of a LMA for this procedure.
.
(Aug99) Halfway through surgery, the capnograph trace becomes flat. Describe your management.
.
(Aug99) Discuss methods of postoperative analgesia for this surgery.
.
(Apr99) A 55 yo adult is to have nasal polypectomy under general anaesthesia. The patient has nocturnal oesophageal reflux and extensive fixed uppper dental prostheses. How would you reduce the risk of perioperative dental damage?
.
(Apr99) How can problems associated with the use of vasoconstrictors in nasal surgery be prevented?
.
(Apr99) Describe the management of systemic toxicity resulting from the use of vasoconstrictors in nasal surgery.
.
(Apr98) An obese 40 yo man with a history of snoring presents for septoplasty and cautery of turbinates. At the pre-anaesthetic consultation, what clinical features on history and examination would suggest to you that he may have sleep apnoea?
- partially as per STOP-BANG

Hx - loud snoring, observed apnoeas, daytime somnolence
- comorbidities, HT/GORD/IHD/CCF/PHT
- Sx of PHT, fatigue, Dyspnoea, syncope

Ex - BMI > 35, neck circumference > 40cm, HT > 140/90
decreased OP space/increased soft tissues, swollen uvula, plethoric facies, hypoxaemix
- sx of RVF, RVH, loud P2, raised JVP, ascites/oedema
(Apr98) Discuss your plan for intra-operative airway management for this patient.
Plan for potential difficult airway. Difficult airway most likely in OSA/big neck circumference.

- Prep.
Difficult airway trolley, assistance.
- Plan for likely difficult BMV.
- positioning, monitoring, pre-O2/head up, RAMP
- rapid onset MR
- early guedel, cricoid if risk of reflux
- would use a videolaryngoscope if thought necessary.
(Apr98) How would sleep apnoea influence your post operative management including provision of analgesia?
Aim is minimisation of sedatives/opioids due to increased sensitivity, and monitoring for resp cx. If known OSA and CPAP - using normal CPAP means there is no extra risk from the general population(?)
- no premed
- short-acting opioids/anaesthetic agents (ie desflurane/propofol)
- maximise non-opioid multimodal analgesia (ie, paracetamol, tramadol, NSAIDS, LA by surgeon).
- monitoring on appropriate ward, according to comorbidities, etc. Overnight pulse oximetry may be warranted, with closer nursing observation.
(Oct-2012 Q10) A trauma patient presents thirty minutes after a significant crush injury, with an estimated 40% blood loss. He was previously well. 1. Explain the coagulation abnormalities you would expect to see in this patient at this stage (60%). 2. Discuss the current evidence for treatment of these abnormalities (40%).
1)
Coagulation abnormalities are due to acute coagulopathy of trauma - due to initial tissue damage, and not necessarily dilutional coagulopathy, acidosis and hypothermia as previously thought (although these will aggravate the situation).
- activation of coagulation pathways via tissue damage and release of tissue factor (leading to a DIC-like coagulopathy), and tissue damage/hypoperfusion leading to activation of the thombomodulin/activated protein C / anticoagulation system
- therefore there is "consumption" of clotting factors in one theory, and inhibition in the other
- there will also be ongoing inhibition of coagulation due to acidosis and hypothermia (which will also affect platelet function)
- activation of fibrinolysis (hyperfibrinolysis) also occurs in a subset of patients
- patients with ATIC have increased mortality
- prolonged INR, APTT, low fibrinogen potentially, thrombocytopenia

2)
Current evidence
- prevent further bleeding
damage control measures, along with use of permissive hypotension prior to definitive control may help prevent further blood loss. damage control surgery and resuscitation also allows "catching up" with correction of coagulopathy/acidosis/hypothermia treatment before more definitive surgery
- blood products
shift away from aggressive use of crystalloids which may worsen coagulopathy by dilution
shift towards mimicking "whole blood" transfusion, with 1:1:1 ratios of PRBC/Platelets/FFP having better outcomes. can also monitor coagulation tests to inform management (ie, INR, fibrinogen)
use of FVIIa where bleeding ongoing (non-surgical), and other factors corrected - not to be used routinely
- antifibrinolytics
TXA has been shown to improve survival in major trauma requiring massive transfusion, especially if given in early stages (<3 hr after injury), as per CRASH-2
-massive transfusion protocols
improve use of blood products, as well as delivery of oxygen-carrying red cells to patients early
improve ratios of blood components
(Apr-2009 Q7) Outline the coagulation changes you would expect in a patient with a ruptured liver from blunt abdominal trauma requiring massive transfusion (50%) and describe how you would minimise then (50%).
.
(May-2008 Q4) Describe the clinical features and treatment of Fat Embolism Syndrome.
.
(Sep-2007 Q3) A 40-year-old woman presents having been trampled on by a horse. She has a compound fracture of her arm requiring surgery and bruising over the centre of the chest with a fractured sternum. List the injuries to the heart that may be caused by this blunt trauma. If she had no signs or symptoms of cardiac injury list and justify any screening investigations for cardiac injury you would perform prior to anaesthesia.
.
(Sep-2006 Q14)Critically evaluate the role of recombinant factor VIIa in blood loss requiring massive transfusion in the trauma patient.
.
(Sep-2001 Q10) A 72 yo male has been bought to your emergency room after being removed from a motor vehicle, which has been involved in a high speed collision. He is moaning, his eyes are closed and he withdraws to pain. What are the priorities in managing this patient on arrival in the emergency department?
This is an elderly man who has been involved in a high speed trauma.
He should be managed as per ATLS guidelines, via Primary survey, secondary survey, including a handover from the paramedics re: trauma, and and past hx about the patient (AMPLE hx).
- Primary surery is
A & C-spine (apply oxygen)
B (sats, RR, auscultate)
C (BP/HR/IV access and bloods) & external haemorrhage control
D - GCS & pupils
E - expose/exposure

Given his conscious state (moaning, eyes closed, withdrawing to pain, although not clear, he probably has GCS of ~ 8), he needs his airway secured, with ILMS of C-spine. Management of BP/oxygenation may improve conscious state first, but otherwise he requires intubation and ventilation immediately.

He has a TBI until proven otherwise, which means his ICP is ~>20, therefore MAP should be kept about 80-90mmhg. After initial primary survey as described he needs prompt imaging and liason with neurosurgery
(Sep-2001 Q11) What is the place for early intubation and controlled ventilation for this patient?
Definitely a place for this. He has a TBI (with GCS~8 = severe TBI), and so needs oxygenation and ventilation (ctronol of paCO2) to control his ICP, as well as to protect his airway (at risk of aspiration).

He is also requiring imaging, which would be otherwise dangerous without securing airwy first.
(Sep-2001 Q12) On transfer to the CT scanner his left pupil dilates. Describe your management.
A unilateral pupil dilatation in a TBI patient signifies likely transtentorial herniation of the brain causing compression of the 3rd nerve leading to pupillary dilatation. This means other brain stem fucntions are at risk if measures are not taken immediately to reduce ICP.

- Neurosurgeons need to be informed, this patient needs to be prepared for urgent craniotomy/craniectomy
- immediate hyperventilation, head up positioning
- fluid resuscitation/vasopressors to increase MAP
- adminstration of hypertonic saline of mannitol (either 100ml 3% saline, or 0.5-1g/kg mannitol 20%) to reduce brain
- also needs to look at adequate sedation/muscle paralysis to decrease cerebral oxygen demand (ie, bolus midazolam, propofol - as long as haemodynamics stable).
(May-2001 Q10) A 17 yo trail bike rider was struck on the neck by a low branch and thrown from his bike. He presents to your casualty with a hoarse voice, stridor and subcutaneous emphysema of the neck. Discuss your plan to secure this patient's airway.
.
(May-2001 Q11) Justify the steps you would take to assess the cervical spine of this patient.
.
(Jul97) A 65 yo active man with a 40 yr pack history of smoking presents with 6 broken ribs and a small flail segment, soon after a fall from a ladder. He has no other significant injury. How would you manage his respiratory care in the first four days?
.
(Jul97) Six hours after the injury, he develops ventricular ectopic beats. How would you diagnose the cause of these ectopic beats?
.
(Sep-2007 Q2) A 60 year old man develops a large haemo/pneumothorax following attempted insertion of a haemodialysis catheter via the left subclavian route.
Describe your technique of chest tube insertion to drain this and the features of the pleural drainage system you would connect to it.
.
(May-2007 Q7) A 65 year old man with a 40 pack a year history of smoking is scheduled for right pneumonectomy for carcinoma. Describe your preoperative evaluation of his respiratory system to decide his capacity to undergo this operation.
Evaluation involves Hx/Ex/Ix
Need to determine
- resectability (staging of tumour, etc)
- operability (can the patient survive the op?)

Need to look at
- Current status of disease
*COAD & optimisation
*Smoking and cessation
*Cancer (the 4 M's = Mass, Metabolic, Medications, Metastasis)
*Functional capacity (?>4 Mets) (<1 flight stairs BAD, > 2 flights okish, > 5 flights = VO2max > 20)
*Cormorbidities that may have an effect on survival (ie, CV disease, arrythmias, PHT, AGE, Alcohol)
- Anaesthetic Hx/Meds/Allergies

Ex
- Airway
- Mass effect (ie, SVC obstruction)
- Cardiorespiratory
- Sats (esp on 6MWT, if sats < 4% BAD)

Ix looks at lung mechanics, parenchymal function and cardiopulmonary reserve. The single most valid test is the ppoFEV1/
Spirometry
- FEV1 > 2L or ppoFEV1 > 40% OK
- DLCO > 40% OK
CPX
- VO2 max is best predictor here
>20ml/kg/min is GOOD, < 10 is BAD, 100% mortality.
>15ml/kg/min for lobectomy.
6MWT
>600m is GOOD.

Ix - also need to look at ease of DLT placement / masses (ie, CT or CXR).
ECG (RVH)
TTE (especially in elderly, and pneumonectomy, PHT is BAD)
Cardiac testing as per AHA guidelines.


Slinger
- Preop, stop smoking, calculate ppoFEV1, ex tolerance, discuss analgesia
- ppoFEV1<40%, assess DLCO, VQ scan, VO2 max
- Ca, assess 4 M's
- COPD, ABG, physio, bronchodilators, antibiotics?
- Renal function
- Cardiac
as per AHA
TTE for elderly / pneumonectomy.
(Sep-2003 Q15) Evaluate the methods available to confirm correct placement of a double lumen endobronchial tube.
- Clinical, Fibreoptic Bronchoscopy & chest radiography. ?others
- Clinical
*advantages
simple, no special equipment required apart from stethoscope
ventilate as normal, assessing visual inspection of chest rise, and air entry to auscultation, then isolate lung and repeat
inexpensive
*Disadvantages
approx 40% of the time despite "adequate" placement clinically, the DLT is malplaced, which may affect surgical access, or cause morbidity to patient.

FOB
- "gold standard" now
advantages
- able to visualise exact positioning in relation to main bronchi and next generation bronchi
- able to visualise cuff herniation or other malpositioning problems
*Dis
specialised equipment which needs appropriate maintenance, cleaning and training to use
expensive, and easily damaged

Xray
advantages
- radio-opaque lines should make positioning easy to see
- also radiographic evidence of lung infation or collapse (ie, adequacy)
Dis
rarely performed intraop, and often DLT not used postop
xray often taken long after the fact of isolation, o problems with malpositioning are not corrected sooner
(Aug-2000 Q1) A 57 yo man with a primary lung tumour is scheduled to have a thoracotomy for a left pneumonectomy. Justify your choice of airway device for this surgery and describe how it is placed.
What ever device is chosen, it must be certain that it is not included in the staple line (ie, L DLT or bronchial blocker).

Options
- DLT
L or R (harder to place, but no need to pull back for anastamosis)
easier to place (more familiar), also if no bronchoscope available (though arguable that this should be done!)
can suction easily down lumen, as well as provide CPAP/ O2 insufflation if necessary (until clamping)

*size selection more difficult, not great for difficult airways, not great for postop ventilation, potential for airway trauma

Bronchial Blockers
- Arndt, Fuji, Univent
- can be used with normal SLT / difficult airways
- means no ETT change for postop ventilation
- selective lobar isolation, CPAP/suction possible (but port very small)

*more time needed for positioning and lung collapse
*need bronchoscope, difficult to alternate OLV, suction, scope

Normal ETT down a main bronchus or endobronchial tube
- easier for difficult situations/airways, but no ability to scope/suction/CPAP isolated lung.

I would place a R DLT if anatomy looked reasonable (ie, could bronch the patient first through an LMA), otherwise L DLT and withdraw before stapling anastamosis

Lung isolation is required for pneumonectomy, mainly for improving surgical access for dividing vessels, etc. (Arguably a relative CI for open thoracotomy).
- R DLT
- would intubate with ETT turned 90 degrees anticlockwise, endobronchial lumen pointing anteriorly, rotate clockwise 90 degrees once through cords and remove stylette
- initial ventilation 2 lungs, and inflate tracheal cuff until no leak (as with any ETT)
- then clamp tracheal lumen (short side), open to air, ventilate R lung, and inflate bronchial cuff until no more leak through tracheal lumen (usually < 3 ml).
- would then check OLV clinically
- then check with bronchoscope
*tracheal lumen - check at carina, check bronchial cuff just visible and not herniating
*R bronchial lumen - check for RUL opening
if not visible, go down to secondary carina, and check which way this is "pointing", may need to rotate tube to attempt to align RUL opening with RUL.
(Aug-2000 Q2) Discuss the advantages and disadvantages of using a bronchoscope to check the position of the device.
advantages
- ~40% DLT's malpositioned (significantly) when using clinical grounds alone
- guides exact positioning and diagnosis of intraoperative problems (such as cuff herniation or mucus plugging)
- able to direct DLT correctly by rail-roading over FOB
- especially R DLT, can check that RUL properly ventilated
- can be used for therapeutic purposes (suction - although not great for paed bronchoscope)
- unfortunately need to use a paed FOB, which is more easily damaged and poor suction capabilities.

Disad
- needs training
- specialised/expensive equipment, needs maintenance and cleaning
- extra time for positioning
(Aug-2000 Q3) Outline your management of an oxygen saturation of 82% during one lung ventilation.
Is it rapid or gradual? Is it real?
May need to call for help and alert surgeon.
Cause could be due to OLV or SOMETHING ELSE.
Mx also depends on gradual or severe/rapididity
- if severe or rapid - need to resume 2LV

otherwise....
- 100% O2 / switch to BMV
- check position with FOB
- maintain CO (volatile < 1 MAC)
- PEEP to dependent lung (if small TV and likely atelectasis - likely need recruitment)
- CPAP to non-dependent lung
- intermittent 2LV
- partial ventilation
- clamping of PA
- Pulmonary vasodilators
- ECMO

Other causes must be thought of, such as
- delivery of oxygen ok (check flowmeter and ETO2)
- check monitors (ETCO2, volatile, BP/HR?sats trace)
- check circuit, incl filter
- may need to rapidly deflate both cuffs in case of herniation after malpositioning
(Oct-2008 Q9) Describe a technique of peribulbar block for cataract surgery. Describe how you would minimise complications of this block.
.
(Apr-2009 Q4) Draw a cross section of the arm at the level of the axilla illustrating the anatomy relevant to performing a brachial plexus block for surgery on the forearm (50%).
List the advantages and disadvantages of a block at this level compared to a suparclavicular block (50%).
This picture more distal than axillary level BUT
- need to show x-section of humerus with muscles, branches of brachial plexus (median, musculocutaneous, radial and ulnar nerves inside the axillary sheath with Artery)
- says should show intercostobrachi
This picture more distal than axillary level BUT
- need to show x-section of humerus with muscles, branches of brachial plexus (median, musculocutaneous, radial and ulnar nerves inside the axillary sheath with Artery)
- says should show intercostobrachial n, medial cutaneous nerves of forearm and arm subcutaneously

Advantages
- No risk pneumothorax
- Bleeding compressible
- better distal arm blockade
- no risk of phrenic/horner's

Disadvantages
- Risk of missing MCT nerve
- arm needs to be abducted
- IV injection / toxicity
- difficult to place catheter
- less good for proximal surgery
(Oct-2008 Q2) Describe the innvervation of the lower abdominal wall from the umbilicus to the pubis symphesis. Describe a technique of peripheral nerve block (not wound infiltration) to provide post-operative analgesia for a low transverse abdominal incision.
.
(Sep-2007 Q3) Outline guidelines you think should be in place for reducing both the incidence and the morbidity of epidural space infections as a complication of epidural analgesia.
.
(Sep-2007 Q13) Describe a technique of neural blockade in the popliteal fossa for surgery on the foot and ankle including a description of the relevant anatomy.
(Sep-2007 Q13) Describe a technique of neural blockade in the popliteal fossa for surgery on the foot and ankle including a description of the relevant anatomy.
From bottom of u/s screen up (anterior to posterior)
- femur
- popliteal artery and vein (vein is middle), artery is medial, and nerve is lateral - "mickey mouse sign" (vein = head, and others = ears).
- Tibial is medial, and CP is lateral
- medial mu
From bottom of u/s screen up (anterior to posterior)
- femur
- popliteal artery and vein (vein is middle), artery is medial, and nerve is lateral - "mickey mouse sign" (vein = head, and others = ears).
- Tibial is medial, and CP is lateral
- medial muscles are semitendinosus/semimembranosus, and lateral is biceps femoris
(May-2007 Q2) Describe the relevant anatomy and technique for field block for inguinal hernia repair.
(May-2007 Q2) Describe the relevant anatomy and technique for field block for inguinal hernia repair.
Inguinal regions supplied by:
­	Iliohypogastric nerve – from L1
•	Supplies skin over the inguinal region
­	Ilioinguinal nerve – from L1
•	Supplies skin on the superomedial aspect of the thigh
­	Genitofemoral nerve – from L1 and L2
•	Genital branch:
Inguinal regions supplied by:
­ Iliohypogastric nerve – from L1
• Supplies skin over the inguinal region
­ Ilioinguinal nerve – from L1
• Supplies skin on the superomedial aspect of the thigh
­ Genitofemoral nerve – from L1 and L2
• Genital branch: supplies motor fibers to the cremaster muscle and sensory fibers to the skin over the scrotum in men and mons pubis/labia majora in women
• Femoral branch: supplies skin over the femoral triangle
­ Subcostal nerve – lateral cutaneous branch of T12


Block

Landmark
­ 2cm medial and 2cm superior to the ASIS
• Some texts say 2cm medial and 2cm inferior to ASIS – however this is more likely to anaesthetise lateral femoral cutaneous nerve and miss ilioinguinal & iliohypogastric

Technique
­ First pop needle lies between external and internal oblique
• Blocks the iliohypogastric nerve
­ Second pop needle lies between internal oblique and transversus abdominis
• Blocks the ilioinguinal nerve

According to NYSORA, repeat at 45 degrees medial and lateral:
­ Puncture skin with a sharp needle, then insert blunt needs 2cm medial and 2cm superior (in an adult) to the ASIS
­ Advance perpendicular until 1st pop through external oblique fascia
• Aspirate and inject 2ml
­ Advance through 2nd pop (internal oblique fascia)
• Aspirate and inject 2ml
­ Withdraw to skin and redirect 45 degrees medially
• Advance and again inject 2ml after each pop
­ Withdraw to skin and redirect 45 degrees laterally
• Advance and again inject 2ml after each pop
(Sep-2006 Q2) Describe your technique for performing a continuous paravertebral block in a 50 year old man with fractured 5th – 10th left ribs. Include possible complications and relevant anatomy.
(Sep-2006 Q2) Describe your technique for performing a continuous paravertebral block in a 50 year old man with fractured 5th – 10th left ribs. Include possible complications and relevant anatomy.
.
.
(Sep-2006 Q5) Describe and justify an appropriate strategy for the use of low molecular weight heparin in a patient undergoing knee replacement surgery with an epidural block.
firstly need to determine patient's risk of VTE (or ATE), as it is a balance between risk of clot versus risk of bleeding (including epidural haematoma).

What is the LMWH for ? ie, DVT prophylaxis or anticoagulation for some reason (bridging therapy).

Consider Pt Fx (ie, age, malignancy, obesity, estrogen usage)

Consider Surgical Fx (Major joint, pelvic ops, prolonged Abdominal surgery, etc)

Initial experience with LMWH (enoxaparin) in the US had a 30mg BD dosage, and was associated with a much higher rate of epidural haematoma (probably < 1/3000). The risk is lower with a single daily prophylactic dose.

Practice Points
- Anti-Xa levels are not particularly useful, and should not be routinely done
- risk of epidural haematoma is probably increased in patients taking other medications which may affect clotting (ie, antiplatelets, NSAIDs, dextrans intraop) and this should be considered
- patient factors such as liver or renal disease should also be considered (pre-existing coagulopathy, or inability to excrete drug properly)

As per ASRA guidelines (expert consensus)
VTE prophylaxis
*avoid neuraxial technique if preop dose given within 2 hrs
*if LMWH given preop, should wait 10-12 hrs before epi needle
*should wait >4/24 after epi before next dose
*wait 10-12 hrs after dose for catheter removal, and >2 hrs until next dose afterwards

If therapeutic anticoagulation
- should wait 24 hrs after last dose, before epi needle
- next dose > 4/24 after
- catheter removed 24 hrs after last dose, and next dose > 2 hrs after.

*if using twice-daily dosing, catheter should be removed 2/24 before LMWH starts postoperatively

Other points
- if bloody tap, recommendation is to wait 24 hrs before next dose LMWH
- if using other antiplatelets (even nsNSAIDs), a neuraxial technique is NOT recommended with LMWH (although COX-2 inhibitors are thought to be ok).

Postop monitoring
- need regular monitoring/observation for any signs of epi haematoma (ie, motor block, back pain, etc)
- a low dose epidural infusion should be used to minimise motor block, and if it develops, it should be switched off until recovery is demonstrated (otherwise needs investigation).
(May-2006 Q10)Describe the anatomy of the orbit relevant to a peribulbar eye block.
(May-2006 Q10)Describe the anatomy of the orbit relevant to a peribulbar eye block.
.
.
(Sep-2004 Q4) Outline the diagnostic criteria for an epidural abscess
Hx
Risk factors
- Compromised immunity (DM, steroids/immunosuppresants, malignancy, pregnancy, HIV, alcoholism)
- disruption of spinal column (trauma, surgery, degenerative disease, neuraxial block)
- indwelling vascular catheters, IVDU, infection from another source

- back pain, fevers, "unwell", neruaxial block

Ex
- fever, meningism, pain on palpation of back
- neurological features such as weakness, sensory loss, reflexes absent (signs are late)

Ix
- leucocytosis, ESR/CRP
- CT (quick, usually easier to get, and will often find the abscess), but may need MRI
(Sep-2004 Q8) Describe the anatomy relevant to providing an ankle block for surgery on the big toe.
.
.
(Sep-2004 Q9) Give reasons for your choice of local anaesthetic agent to provide intravenous regional anaesthesia for a reduction of a Colle's fracture in an 80 year old woman weighing 95kg.
Bier's Block
- 0.5% prilocaine or lignocaine approx 40ml necessary
- RCH says 0.3ml/kg of 1% lignocaine, and equal volume of saline to make 0.5%
- basically need to leave for at least 20 mins before tourniquet down (needs to be > 50mmhg above systolic pressure).
- no use of bupivacaine/ropivacaine!!
(Sep-2003 Q7) At the end of an open cholecystectomy, intercostal nerve blocks with a total of 20ml bupivacaine 0.5% are placed at two levels while the patient is still under general endotracheal anaesthesia. The patient develops ventricular fibrillation within 3 minutes. Describe your management of this situation.
Anaesthetic Emergency
- press arrest button
- Attention to A, B, C and follow ALS guidelines for VF arrest, including....
- need to guide OT team through ALS protocol

*CPR, performed at ~100/min, ratio 30:2, although could just ventilate this patient at 8-10 breaths/min
*Defibrillate ASAP, as soon as pads are on, 150-200J x 1, then continue CPR.
*I would ask for the intralipid to be brought into theatre, as given the time course, this is likely to be due to LA toxicity from rapid intravascular absorption or IV injection
*Rhythm check and shock every 2 mins
*Adrenaline 1mg after 2nd shock
*Amiodarone 5mg/kg after 3rd shock
*Lipid rescue protocol is 1.5ml/kg bolus of 20% intralipid, and start infusion at 15ml/kg/hr
- can have 2 further boluses, and double rate of infusion, not to exceed > 12ml/kg

- continue as per ALS protocols (ie shockable or non-shockable rhythms)
- consider 5H's / 5T's
Hypovolemia
Hypo/Hyperkalemia
Hydrogen Ion
Hypothermia
Hypoxia

Tamponade
Tension Pneumothorax
Thrombosis - coronary
Thrombosis - Pulmonary
Toxins

Consider CPB (for LA toxicity)

Need to then consider post-arrest care (ie, cooling, sedation), and disposition (ICU).
(Sep-2002 Q7) A 60 yo woman presents for elective right bunion surgery. She is hypertensive, controlled on medication, and would like the procedure to be performed under spinal anaesthesia. The surgeon requests this procedure be done under tourniquet control. How would you apply the tourniquet, what pressures would you choose, and what precautions would you take?
see 91.
(Sep-2002 Q8) Three weeks following the surgery, the patient contacts you as she has persistent numbness in her big toe. Describe your management of this situation.
.
(Sep-2002 Q9) Describe in detail how you would perform and ankle block for this patient if spinal anaesthesia was contra-indicated and a tourniquet was not required.
.
(May-2002 Q7) A 75 yo heavy smoker presents for a total knee replacement. He has a mechanical mitral valve prosthesis and is warfarinised. Discuss your strategy for perioperative anticoagulation.
Preop
Need to balance risks of thromboembolism (venous and arterial) versus risks of bleeding.

Patient risks
- smoker, mechanical mitral valve
- other Hx? - previous VTE, thrombophilia, other anti-platelet agents, malignancy, CCF

Surgical risks
- Major joint replacement

Need to consult patient's cardiologist, or haematology and discuss with surgeon - but I think this patient is high risk and needs bridging therapy.
- protocol depends on institution
- cease warfarin 5-7 days before, monitor INR
- daily enoxaparin when subtherapeutic (1.5mg/kg)
- stop 24 hrs preop
- take into account other anticoagulants/antiplatelets
- discussion re anaesthetic technique (epidural prob not a good idea)

Needs mechanical prophylaxis also

Needs to resume postop anticoagulation as soon as surgeon happy, may need initially high "prophylactic" doses, then full again. restart warfarin, and enoxaparin until therapeutic.
- cease smoking!
(May-2002 Q8) What is the place of a spinal anaesthetic in this patient?
.
(May-2002 Q9) Discuss your plan for postoperative analgesia given that the surgeon plans for the patient to use a continuous passive motion device.
.
(Sep-2001 Q9) Describe the symptoms, natural history and causes of "transient neurological symptoms" following spinal anaesthesia.
.
(Aug-2000 Q4) A 46 yo male presents to a day procedure unit for a right knee arthoscopy. He wished to avoid general anaesthesia as he has experienced significant nausea in the past. In performing a spinal anaesthetic for this procedure, what factors would you consider in choosing the local anaesthetic agent?
.
(Aug-2000 Q5) The following day the surgeon calls you because the patient is complaining of pain in his right thigh. What possible causes would you consider and how would you respond?
.
(Aug-2000 Q6) When you contact the patient that day, he tells you that he has also been suffering from moderately severe headache since arriving home. What information would you seek and what advice would you give him?
.
(Aug99) An 83 year old woman slips and sustains a left Colles' fracture after a birthday lunch, and is booked for manipulation and plaster of her fracture. Discuss the benefits and drawbacks of intravenous regional anaesthesia in this patient.
.
(Aug99) Give reasons for your choice and dose of local anaesthetic agent for intravenous regional anaesthesia.
.
(Aug99) What hazards does the use of a tourniquet present in this situation, and how might they be minimised.
see 91
(Apr98) The patient becomes unconscious after injection of a total of 10 mls of bupivacaine 5mg/ml for penile block. Describe your initial assessment.
.
(Apr98) The patient is found to be in ventricular fibrillation. Describe your management of this situation.
.
(Jul97) A 50 year old apparently well man presents for inguinal hernia repair under spinal anaesthesia. Discuss the factors which would influence your choice of subarachnoid drug(s) for this surgery.
.
(Jul97) Another anaesthetist administers 4 ml plain bupivacaine 0.5% into the subarachnoid space for his anaesthesia, and you are called to assist with resuscitation of the patient when he becomes asystolic 10 minutes after surgery commences. Discuss the possible causes of the asystole.
.
(Jul97) Describe your management of this critical event.
.
(Apr97) A 75 yo man requiring TURP takes a diuretic and beta blocker for hypertension. Spinal anaesthesia with 0.5% bupivacaine (plain) results in a sensory level of T10 and a fall in blood pressure from 170/95 to 130/80. Outline the considerations in the selection of spinal as compared with general anaesthesia for this man.
.
(Apr97) How would you respond to a fall in blood pressure to 80/50, occurring after 30 minutes of surgery?
.
(Apr97) After 70 minutes of surgery the patient becomes restless. Explain your management.
.
(Oct-2012 Q8) OUtline the key steps in gaining informed consent for anaesthesia in a competent ASA 1 adult undergoing minor elective surgery.
- informed consent must be given VOLUNTARILY & without COERCION
- person must have capacity to give consent (Assumed)
- a patient has the right to self-determination
- must involve information, about risks/benefits/alternatives, specifically risks that the patient may place special significance on
- discussion ideally before surgery (time) and before OT (place)
- ideally some sort of documentation (ie, signing consent form) is warranted, but does not automatically mean pt has given proper informed consent

Is necessary because
- an individual has the right to self-determination (autonomy)
- medico-legally, avoids the committal of assault, as well as that of negligence where a procedure has been performed competently but risk discussed
(Oct-2012 Q6) You are the consultant who has been tasked with introduction of the WHO Surgical Safety Checklist to your hospital. 1. What are the principles behind the checklist that enhance patient safety, with reference to each component (70%). 2. What do you expect the barriers to its effective implementation to be? (30%).
The WHO SS checklist has been shown to reduce mortality and morbidity in a generalised population.

The components are
- sign in
- time out
- sign out

The principles are those of
- improved team work and communication
- empowerment of people in the team to speak up
- systems approach to minimise neglect of important perioperative interventions and avoidance of risks
- focus on patient safety
- can be adapted to local practices

Focus on common perioperative outcomes such as
* antibiotic prophylaxis and wound infection, DVT prophylaxis, airway difficulties, wrong side surgery, anaphylaxis, blood loss, discussion of postop care

Barriers to implementation
- protocol fatigue
- unwillingness to change traditional practice (needs buy in from all staff)
- if some components ignored, then may not be as effective
- not having effective training for all staff (including casual and part-time)
- may be initially time-consuming
(Oct-2008 Q14) What are the signs that may make you suspect opioid abuse in a colleage? If you had suspicions of opioid abuse in a colleague outline the principles that should guide intervention?
a) signs are said to be "major" (immediately reportable) and minor.
- signing out increasing opioids, patients in pain despite this, poor records
- intoxication/withdrawal, injection marks, needle in vein(!), direct observation of diversion
- clothing, out-of-hours, personality, health issues, relationship issues, breaks

Principles guiding Intervention
- Patient safety is paramount
- Safety of the individual
- Need to maintain confidentiality and gather evidence
- Reporting to senior staff / AHPRA
- Intervention, voluntary/involuntary treatment
- Longer term plans (Change profession?)
(Oct-2008 Q4) In what circumstances is it permissible to permanently handover responsibility for a colleague and how would you ensure that this handover occurs safely?
Permanent Handover
- anaesthetist fatigue
- illness
- Suitable & willing colleague
- any other legitimate commitment
- Patient safety is priority

Safe Handover
- primary anaesthetist should be satisfied as to competence of relieving anaesthetist
- must be clinically stable
- must inform surgical team
Info
- Patient
Medical/surgical/anaesthetic Hx, meds, allergies
- Surgical
nature and stage of surgery
- Anaesthetic
Airway, ventilation, lines, fluids, drugs, events, and foreseeable probs
Chart must be UTD
plan for intraop and postop mx and disposition
reliever satisfied?
contact details
YAY
(May-2007 Q11) Why is consent for a medical procedure necessary? What makes consent for a medical procedure valid?
see 230
(Sep-2005 Q9) Discuss the purpose of a postoperative visit.
- Could refer to postop ward, PACU or even after hospital discharge
- to direct certain parts of patient's postop care (ie, pain, PONV, oxygen/fluid therapy)
- to check for complications and manage them
- to follow-up known complications (for treatment/mx)
Also, for personal QA
- pt satisfaction with technique
- did regional work?
- any complications?
(Sep-2005 Q14) You see a patient in the preanaesthetic clinic who asks you to administer an "alternative medicine" as part of their anaesthetic for total hip replacement. How would you respond to this?
- need to balance respecting autonomy blindly, versus non-malificence. Also have a duty to provide best known care, and of an expected standard (?adminstering unlicensed medication - not professional, let alone, not covered by indemnity if complications resulted).
- medicine may have known or unknown adverse effects or interactions with anaesthetic agents/analgesics.
(May-2005 Q14) A recovery charge nurse approaches you as Supervisor of Training because she is concerned at the amount of opiates one of your trainees has been signed out for patients. What will be your priorities in addressing the nurses concern?
- need to satisfy reporter that you will take the concerns seriously and investigate
- priority is patient safety
- individual safety also thought of
- need to ask for discretion from reporter while investigation ongoing
- if there is evidence of notifiable behaviour, then mandatory reporting must be followed
- any "major" features may warrant immediate reporting and intervention.
(Sep-2004 Q12) What are your obligations if you suspect a colleague to be chronically impaired?
Obligations to
- Patient!
- General public safety
- Colleague
- Department/other staff
- any person reporting

If any "serious" notifiable issue is suspected or known, then under mandatory reporting laws, the colleague must be reported to the medical board AHPRA.
- The patient (especially under immediate care of the colleague) requires protection ASAP.
- The colleague is at high risk of suicide if confronted, so intervention must be planned with appropriate people, and someone in place to help the colleague prior to any detoxification procedure.
(May-2003 Q6) Discuss the importance of a routine post-anaesthetic visit by the anaesthetist
.
(Sep-2002 Q1) A colleague seeks your advice. A patient to whom they gave an anaesthetic 2 days ago for removal of wisdom teeth as a day case, claims that she was awake during the operation. The patient remembers hearing someone refering to her as a "fat old cow". Your colleague has been notified that a formal complaint about this matter is to be investigated by the relevant legal authority. Discuss the factors which may have contributed to awareness in this patient.
.
(Sep-2002 Q2) Give your recommendations for management of this patient's complaint.
.
(Sep-2002 Q3) What strategies may be used to assist your colleague with the stress they may feel about the forthcoming investigation?
.
(Sep-2001 Q4) A recovery charge nurse approaches you expressing concern at the amount of opiates one of your consultant colleagues is booking out for minor procedures. Today this colleague is working as the sole anaesthetist in the cardiac theatre. What will your be priorities in addressing the nurse's concerns?
.
(Sep-2001 Q5) If opiate abuse were found to be the problem, what would the important elements of rehabilitation for this anaesthetist?
.
(Apr-2000 Q7) You are asked to provide anaesthesia for an appendicectomy late at night. When you arrive, you smell alcohol on the surgeons breath. How would you respond to this situation?
.
(Apr-2000 Q8) What are your obligations if you suspect a colleague may be chronically impaired?
.
(Apr-2000 Q9) What are the signs of alcohol abuse in a colleague?
.
(May-2010 Q2) a. List the hazards to the patient associated with the prone position under general anaesthesia. (60%) b. How can these hazards be minimised? (40%)
a)
Potential neck/spinal cord injury from excessive flexion or extension.  Need to take care to move head with body all at once so no torsion on neck on moving patient.  Need to view patient from side to ensure adequate neck position.
- Risk of CVS inst
a)
Potential neck/spinal cord injury from excessive flexion or extension. Need to take care to move head with body all at once so no torsion on neck on moving patient. Need to view patient from side to ensure adequate neck position.
- Risk of CVS instability
decreased CI, venous return (depending on position and also abdominal compression), hypotension
difficult position to perform CPR/chest compressions in
Fluid loading, monitoring, keep bed nearby in case of need for emergency transfer back to bed.
- Resp probs
potential for better VQ matching, but thoracic compression may limit ventilation
loss of airway a potential hazard
Need to secure carefully, and position for best ventilation.
- increased bleeding due to raised epidural venous pressure
for back surgery, poor positioning and abdominal compression may lead to increased bleeding.
Special frames leaving abdomen hanging free can be used.
-risk of peripheral nerve or eye injuries
either arms by sides, or "superman" style (need shoulders abducted < 90, minimal flexion, elbows > 90), need to watch pressure areas of breasts/genitalia, pelvis, knees/toes. Eyes should be clear of head cushion or horseshoe frames. There is still potential for POVL in spinal surgery (with blood loss, anaemia, fluids ++ - most likely from ischaemic optic neuropathy).
- risk of carotid/vertebral artery dissection with rotation of neck
- loss of monitoring/IV lines on positioning
(Oct-2009 Q3) a. 49-year-old woman has just arrived in the Recovery Room following a total abdominal hysterectomy under general anaesthesia. She is agitated and complaining of difficulty breathing. 1. List your differential diagnoses. (40%) 2. How would you determine if this was caused by residual neuromuscular blockade? (40%) 3. What is the role of sugammadex in the treatment of residual neuromuscular blockade? (20%)
An organised approach to the differential diagnosis of postoperative agitation and difficulty breathing eg

o upper airway obstruction
o lower airway obstruction
o lung disease
o chest wall or diaphragmatic dysfunction including neuromuscular dysfunction
o abnormalities of circulation or oxygen delivery (eg anaemia)
o neurological dysfunction, including delirium or anxiety

Making the diagnosis with a relevant history (especially timing and dosing of any recently administered drugs, concurrent drug administration (eg gentamicin) etc), examination (especially assessment of airway, ventilation, muscle power and the use of train-of-four monitoring)

A definition of “adequate reversal” (eg TOF 0.9 using accelerometry), a description of the type of monitor used (eg mechanomyography) and a correlation between the clinical assessment of muscle power with the findings of neuromuscular monitoring

Sugammadex is a cyclodextran that binds to aminosteroid neuromuscular blocking agents and therefore has no effect on other muscle relaxants
It has a rapid onset of effect suitable for use when neuromuscular blockade is life-threatening or symptomatic, particularly when neostigmine has reached its “ceiling” effect or is unlikely to work (eg with profound blockade)
It is expensive and allergic reactions have been reported, therefore restricting the indications for its use
(Oct-2008 Q3) What would make you suspect venous gas emnbolism during a surgical procedure? Briefly outline the principles of management of venous gas embolism causing haemodynamic compromise.
.
(May-2008 Q2) Why is the radial artery a common site for arterial cannulation? What complications may occur from radial artery cannulation and how may they be minimised?
.
(May-2007 Q10) A 56 year old diabetic is scheduled for laparoscopic nephrectomy. This is his pre-operative 12 lead ECG. (See Examiners Reports.)
Ten minutes into the procedure his BP is 70/30 and his ECG lead 2 monitor looks like this. (Shows CHB). What does ECG 1 show? What Does ECG2 show? Outline your management of the situation associated with ECG 2.
.
(Sep-2006 Q13) List the risks associated with the placement of a central venous catheter? Discuss the ways in which these risks may be modified.
.
(May-2006 Q1) List the predisposing factors for aspiration of gastric contents in a patient undergoing general anaesthesia. Discuss the measures you would take to prevent this complication.
.
(May-2006 Q2) Describe the factors that contribute to intravenous drug errors in anaesthesia practice. Discuss the methods available to reduce the incidents of such errors.
.
(May-2006 Q7) A seventy five year old man having a transurethral resection of the prostate under spinal anaesthesia which has been uneventful, becomes restless 70 minutes into the procedure. He had 2 milligrams of midazolam at the start of the case and no further sedation. Describe your assessment and management of this problem.
Ax
- Need to check A, B, C, Patient, Surgeon and monitors.
- Causes of agitation could be airway obstruction, hypoxia, hypotension/hypertension, pain/discomfort, or something unrelated to surgery entirely (ie, stroke).
- Given patient is having TURP, and op has gone > 60 mins, would have to be thinking TURP syndrome.

Risks
- >60 mins
- pressure of infusion bag too high, or > 70cm
- patient hypotension (venous pressure low)
- large prostate (>50g)
- large blood loss (may cause anaemia, causing problems in itself, but may be an indication of many open veins and therefore fluid absorption).
- may result in fluid overload/LVF, confusion/agitation, seizures, blindness (glycine toxicity), hypotension

- need to discuss with patient/surgeon
- treat pain
- surgery may need to be finished quickly
- GA may need to be given for control of situation

Urgent Ix - VBG (Na), electrolytes, Hb

May need treatment with anticonvulsants or hypertonic saline (if Na < 120 or symptomatic)

Treat fluid overload with frusemide

3% saline to give?
- 2 x the TBW in mls/hr will make the Na rise by 1mmol/hr.
- ie, 70kg male, TBW = 42 x 2 = 84mls/hr.

Needs further Mx in HDU
(Sep-2005 Q8) A 35 year old female is found to have a small pneumothorax following removal of a breast lump under local anaesthesia in a day surgery facility. How would you manage this?
- pneumothorax is "air in the pleural space".
- small pthx may not need any treatment, and just careful observation, and potentially discharge from the day unit.
- assessment of exact size, and also any patient compromise of comorbidities will impact decision.
- conservative, or aspiration, or ICC/transfer as inpt
(Sep-2005 Q12) Describe the symptoms and signs of commonly seen perioperative nerve injuries in the upper limb. List the causes and possible strategies for prevention. Do not include injuries due to neural blockade or direct surgical trauma.
.
(May-2005 Q7) List the possible causes of failure to emerge from general anaesthesia and describe how you would differentiate them
Patient Fx
A- airway obstruction and hypoventilation (ie, OSA)
B - hypoventilation (OSA with increased sensitivity to opioids, sedatives, COAD/neuromuscular weakness
C- hypotension, cerebral ischaemia
D - CVA, Epilepsy/post-ictal, psychiatric, neuromuscular disease
E - electrolytes (hypoNA)
F - sensitivity to drugs (ie, liver/renal failure)
G - Hypoglycaemia / HONC

Surgical Fx
- Carotid (CVA)
- Neuro (seizures, raised ICP)
- sitting position (Cerebral ischaemia)

Anaesthetic Fx
- Volatiles (off?)
- opioids
- benzos
- residual NMB (and causes including interactions with NDMR and sux apnoea)
- drug errors
- central anticholinergic syndrome
- hypothermia

Differentiate
- requires brief Hx (look through notes for past Hx), ex (Airway, breathing, circulation, GCS/pupils, and temp, check TOF/ET-CO2/AA) and Ix (ABG/VBG for electrolytes, glucose, may need others such as doppler or CT brain
(Sep-2004 Q2) Outline the possible causes of postoperative loss of vision
Ocular or non-Ocular
- Ocular
*corneal abrasion
trauma, painful
*Ischaemic optic neuropathy (AION or PION)
most common cause of true POVL
may occur in lengthy surgeries, large blood loss, prone spinal surgery (PION), cardiac surgery (AION), hypotension, large crystalloid load, anaemia, Hx vascular disease
*Central Retinal Artery Occlusion
external compression (ie, prone, and poorly positioned)
*Acute glaucoma
headache, red eye, hx?
*Damage to globe
eye block?

Non-Ocular causes
- Cortical blindness
Cardiac, thoracovascular surgery

Metabolic
- hypo/hyperNa, glucose, TURP syndrome
(Sep-2003 Q11) A 68 y.o. man with pancreatic carcinoma is undergoing sedation for ERCP. His is in a semi-prone position. Soon after commencing ERCP you notice the oximeter reading is 73%. Describe your management.
.
(Sep-2003 Q12) Discuss the options for management of a cold white hand with poor capillary refill ten minutes after placement of a radial artery cannula.
.
(May-2003 Q10) A 25 yo woman has undergone a dianostic laparoscopy. In the PACU she complains of nausea and vomits repeatedly. Discuss the factors that make this patient more prone to post op nausea and vomiting.
.
(May-2003 Q11) Discuss your management options in the PACU
.
(May-2003 Q12) You are to anaesthetise this patient for repeat laparoscopy. Justify the strategies you would use to minimise PONV
.
(May-2003 Q13) On removal of the drapes after a two hour mastectomy on a 20 yo patient, the endotracheal tube is found to be disconnected from the breathing circuit. How would you estimate the probably time interval from disconnection to detection?
.
(May-2003 Q14) Two hours later, the patient has still failed to regain consciousness. Describe how you would explain this to the relatives.
.
(May-2001 Q13) How would you try to decrease the likelihood of post-operative nausea and vomiting in a 40 yo woman who gives a history of severe distress from this, and now requires a laparoscopic cholecystectomy.
.
(Aug96) Permanent loss of vision may occur following surgery unrelated to the eye. How may the likelihood of loss of vision following general anaesthesia for laminectomy be minimized?
.
(Aug96) An otherwise healthy 25 year old woman undergoes hysteroscopic endometrial ablation. After an uneventful general anaesthetic she is slow to wake in the recovery ward and becomes restless and confused. How would you diagnose the cause of this delayed recovery?
.
(Aug96) While you are assessing her, she begins to convulse. Describe your management.
.
(Aug96) Compare the pharmacology of diazepam and thiopentone, with respect to the management of the patient's convulsions.
.
(May-2007 Q12) How do you assess an otherwise well patient with regard to difficulty of intubation at the bedside? How accurate is such an assessment?
.
(Sep-2006 Q1) Discuss the risks and benefits associated with intermittent positive pressure ventilation through proseal ® laryngeal mask airway for a patient undergoing laparoscopic cholecystectomy.
.
(May-2006 Q6) Describe your immediate assessment and management of the airway in a patient with smoke inhalation injury.
.
(May-2006 Q13) What is the physiological basis of preoxygenation? Describe your method of preoxygenation including how you assess its adequacy.
.
(Sep-2005 Q5) What is the role of a laryngeal mask airway in a failed intubation for laparotomy?
LMA is a "rescue technique" in a failed intubation situation.  It is either for ongoing oxygenation (and waking patient up), or potentially as the airway for the case if urgent (until specialised methods of intubation through LMA can be peformed).
- type
LMA is a "rescue technique" in a failed intubation situation. It is either for ongoing oxygenation (and waking patient up), or potentially as the airway for the case if urgent (until specialised methods of intubation through LMA can be peformed).
- type of LMA important (in terms of risk of aspiration)
- is emergency or elective surgery? May sway which way to go.
- with iLMA, all tube sizes (6,7,8) can fit through all iLMA sizes (3,4,5).
- maintain cricoid?
(Sep-2004 Q13) Describe the technique of applying cricoid pressure to prevent regurgitation of gastric contents.
.
(May-2004 Q2) Discuss the presence of morbid obesity as a predictor of difficult intubation.
Morbid Obesity = BMI > 40
Difficult Intubation - defined several ways
*3 or more attempts at intubation, or need for specialised laryngoscope
*Cormack-Lehane grade 3-4
*failed intubation
*difficulty experience by an experienced operator (vague, but pretty much covers everything!)

- In large retrospective studies, obesity found to be predictor o difficult BMV and intubation
- other studies have not shown this
- Kheterpal (2002?) found obesity associated with
*OSA, snoring
*mandibular protrusion difficulty
*Neck circumference > 40cm
*BMI > 30
all associated with difficult intubation

Overall - would base presence of morbid obesity as one factor, but would look for multiple factors. Definitely predicts difficult BMV.
Positioning may be the key to minimising difficulties with intubation (ie, RAMP position).
(Sep-2002 Q15) Discuss the role of the LMA in the management of a difficult intubation
.
(May-2001 Q14) What are the arguments for and against leaving a laryngeal mask airway in place for unsupervised removal by recovery room nurses?
.
(Apr98) A 50 yo man with gastro-oesophageal reflux and occasional nocturnal pharyngeal reflux is to have knee arthroscopy under general anaesthesia. What are the methods which could be used to minimise the risk of aspiration of gastric contents?
.
(Apr98) What are the adverse effects of the pharmacological agents which could be used for this purpose (to minimise the risks of aspiration of gastric contents)? Exclude any considerations of anaesthetic agents or muscle relaxants.
.
(Apr98) Despite your best management the patient does regurgitate and aspirate at induction. How would you manage this?
.
(Jul97) What are the arguments for and against the use of a laryngeal mask airway for general anaesthesia for laparoscopic tubal ligation?
.
(Apr96 Q1) Explain your bedside assessment of the airway in an adult who has a history of a difficult tracheal intubation.
.
(Oct-2012 Q4) 1. what is the natural history of aortic stenosis? (30%). 2. What are the key echocardiographic features in haemodynamically significant aortic stenosis? (70%)
1)
Natural history depends on aetiology of disease (calcific degenerative often more rapidly progressive than congenital or rheumaitc), and also on whether patient is asymptomatic or symptomatic (regardless of valve area).

- pathophysiology
- obstruction to LV outflow increases gradually over many years, leading to LV concentric hypertrophy, increased myocardial oxygen demand, and potential for ischaemia
- valve area decreases by 0.1cm per year approx
- once symptomatic (exertional angina, syncope, heart failure), survival is reduced to 2-3 years without treatment, and with sx of LVF, 18 months.

2)
Quantitative findings
- Valve area, >1.5, 1-1.5, < 1cm2
- Mean pressure gradient, <25, 25-40, > 40, >50
- diastolic dysfunction as measured by E/A ratio
- decreased LVEF in late stage disease

Qualitative
- severe restriction of aortic valve leaflets
- LV hypertrophy
- LA dilatation (late)
- LV dilatation (late)
- pulmonary HT and RVSP increases late stage
(Oct2012 Q2) A 75 year old man presents for right hemicolectomy for an obstructing lesion of the ascending colon that has failed to settle with conservative management. He had a drug-eluting stent plaed eight months ago, and is currently on clopidogrel and aspirin. Discuss and justify your plan for perioperative management of his antiplatelet therapy.
Factors to be considered included
- risk of delay of surgery and/or transfer to another centre with greater surgical expertise and/or coronary revascularization
- the risks of discontinuing dual therapy before 12 months - surgical risk of bleeding for the type of surgery required - role of bridging therapy - risks of rebound hypercoagulability
- the type of stent and its location within the coronary vasculature - the place of the multidisciplinary team-measures to decrease bleeding - monitoring the effects of the chosen management


-Need to balance risk of bleeding versus risk of stent thrombosis (with associated high mortality) in a patient, who has not completed recommended 12 months DAP therapy after DES.

Preop
- need to assess CVS risk / risk of thrombosis, and would usually require consultation with the cardiologist
*why was stent placed and where?
*any difficulties that infer increased risk
*which vessel (ie, prox LAD worse than distal RCA branch)
*any other CVS risk factors

Discussion with surgeon re: risk of bleeding, and management of such (ie, platelet transfusion?)

A MDT decision about bridging therapy should occur, also depending on hospital protocol
- LMWH (not antiplatelet)
- tirofiban preop
- start clopidogrel postop ASAP and load

Need to decide on best place for surgery (ie, does the hospital have the appropriate care, HDU, and revascularisation facilities?)

Intraop
- minimise bleeding, aggressive warming, treatment of coagulopathy, etc
- continue aspirin periop
- cont other CVS medications

Postop
- monitoring for bleeding or MI
- restart clopidogrel and LOAD.
(Apr-2009 Q9) A 65yo male presents in PAC. He is scheduled for fem-pop bypass surgery for PVD in 4 days time. He has ischaemic rest pain in his leg. Evaluate the usefulness of initiating therapy with beta-blockers to reduce the incidence of perioperative myocardial infarction in this man.
.
(Oct-2008 Q12) List the indications and contra-indications for the use of an intra-aortic balloon pump. Describe how its performance is optimised.
IABP aims to increase myocardial oxygen delivery by increasing coronary perfusion, and decrease myocardial oxygen demand by decreasing afterload.

indications
- refractory ischaemia
- severe CCF (especially bridging)
- post- CPB to adi separation
- post-infarction, severe IHD, MR awaiting recovery/surgery

CI's
- AR
- severe aortic/femoral/iliac disease including aneurysms or previous surgery
- local sepsis
- coagulopathy relative, or CI to heparinsation

Optimising performance
- depends on appropriate inflation in diastole to improve cor perfusion, and appropriate deflation in systole to decrease LV afterload
- timing usally by ECG, but may need to be by aortic pressure triggering if unstable rhythm
- positioning 1-2cm distal to L subclavian (check on TOE or CXR)
- ballon size will depend on IABP selected for patient, and % augmentation
- alternate beats timing (for weaning)
- may also need to change patient's HR, and sinus rhythm is better (As get atrial kick as well).
- need to monitor pressure waveform for appropriate timing and augmentation monitoring
(May-2008 Q10) A patient with an AICD with biventricular pacing presents for elective surgery. Describe how the presence of this device influences your perioperative management of this patient.
Preop/Intraop/Postop

AICD/BiV PPM
- suggests mod-severely impaired LV function, as this is resynchronisation therapy (often used in people with NYHA III/IV patients), or previous arrythmia (secondary or primary prevention for VF?)


Preop
- Hx/Ex focussing on cardiac disease, reason for PPM/AICD, medications, stability of sx and functional capacity. What is the surgery? (Major or Minor), and can it be avoided?
- Ix, including baseline bloods (renal function may be affected), CXR, TTE, PPM check, and non-invasive testing as required depending on risk factors and level of surgery (AHA guidelines).
- need to consult patient's cardiologist
- PPM check, and reprogramming

*Reprogramming
most recent guidelines suggest ICD function turned off or PPM asynchronous only in higher risk of EMI, which is taken to mean use of monopolar diathermy ABOVE the diaphragm. Therefore if surgery below diaphragm, can proceed with magnet (as long as known what the response is).
- especially proceed with BiV pacing as this will improve CO

Intraop
- any reprogramming should be done immediately preop (PPM function, anti-tachycardia function, rate responsiveness)
- continuous ECG monitoring + Defib pads on if ICD turned off
- invasive monitoring likely required given level of surgery (at least a-line)
- attention to haemodynamics for surgery, medications/fluids, acid-base, electrolytes

Postop
- as stated, monitoring needs to occur until ICD function reviewed and switched on
- likely needs HDU bed anyway depending on surgery
(May-2007 Q8) Describe how the ECG should be used to monitor for intraoperative myocardial ischemia in a patient with ischemic heart disease.
ECG should be used in conjunction with other monitors and be interpreted in context. mainly for detection of arrythmias and myocardial ischaemia (ST segment monitoring), but can also be useful for electrolyte disturbance.

Technical Fx
- correct lead placement (and decrease impedance by shaving skin, etc)
- correct filtering, also monitoring mode (ie, ST segment analysis, etc)
- 5-lead vs 3-lead, correct positioning (especially if may interfere with surgical access
- trends?

Clinical Fx
- arrythmias and conduction defects, ST segement changes may be due to ischaemia (but depending on configuration, only 70-80% sensitive).
*lead V5 (75%) is best single lead, V4&5 (90%), II & V5 (80%). Lead II, V4,5 = 98%.
*upsloping ST depression 2mm, horizontal/downsloping ST depression 1mm, ST elevation, TWI, arrythmias, conduction defect
(Sep-2005 Q16) Critically evaluate the use of Beta blockers in the perioperative period to prevent myocardial infarction.
.
(May-2006 Q8) The first patient on your orthopaedic list tomorrow is scheduled for left total hip replacement. He has an implanted (permanent) cardiac pacemaker. Discuss the relevant factors in your pre-anaesthetic assessment of this patient.
See other questions on PPM/AICD.
- Issues are CVS Hx/Ex looking at underlying disease, function of PPM (PPM-dependent? Anti-tachycardia therapy?), functional capacity, meds, etc
- requires consultation with cardiologist, PPM tech/check
- Issues related to surgery (ie, OA vs RA, other joints involved, neck/airway), risks of surgery (bleeding, positioning, DVT prophylaxis)
- Regional vs GA and discussion with patient

This question was generally quite well handled. Candidates who performed poorly: • failed to obtain a cardiac history in order to elucidate indication for PPM • failed to attempt to find out underlying rhythm or ascertain pacemaker dependence • failed to obtain preoperative technical advice / pacemaker interrogation / reprogramming if
required • did not display awareness of practical considerations
eg suggesting that the surgeon avoids use of diathermy, cancelling case if pacemaker has not been checked in last 6mths
(May-2006 Q12) List the causes of acute atrial fibrillation in the perioperative period. Describe your management of acute atrial fibrillation which occurs in the PACU (Post-anaesthesia Care Unit) in a patient who has had a total hip replacement.
Causes
- CVS causes
pre-existing arrythmias, IHD, ischaemia, HT, Valvular HD, sympathetic stimulation/stress from surgery
- "Non-CVS Causes"
Hypoxia, thoracic surgery common, electrolyte disturbance, ETOH, sepsis, endocrine dysfunction/disease

Mx
- AF is common postoperatively, and will often self-revert (60% within 24 hrs)
- more urgent if associated haemodynamic compromise or rate-related ischaemia
- otherwise need to rate control (and consider ongoing mx of anticoagulation based on risk factors if surgeon's in agreeance)
- Initial ABC approach, 100% O2, bloods, 12-lead ECG, ABG for electrolytes, troponins
- if immediate HD compromise, needs electrical cardioversion, otherwise can treat expectantly, anti-arrthymics (beta-blockers, digoxin)
- need to discuss with cardiology, Ortho, ICU team?
- disposition
(Sep-2005 Q10) Critically evaluate the role of cardioversion in the management of intraoperative arrythmias.
Cardioversion is the reversion of a cardiac arrythmia back to sinus rhythm - may be pharmacological or electrical.
- if electrical, needs to be synchronised with cardiac cycle, to avoid R on T, and possible VF.
- indicated (As 1st line), if pt haemodynamically unstable, or after drug therapy if patient stable.
- arrythmias common intraoperatively, but only effective for tachyarrythmias.
Pros
- patient already anaesthetised
- simple & effective treatment
- low risk of problems associated
- avoid risks of drug therapy
- quickest acting in HD unstable patients
Cons
- potential for injury (burns/shock) to patient and staff
- potential for VF if not synchronised
- fire (oxygen, GTN patch)
- risk of emboli (AF Hd unstable but chronic?)
(Sep-2005 Q11) How would you assess the severity of cardiac failure in a 75 year old man presenting for joint replacement surgery? Include any relevant investigations.
Assessment by Hx/Ex/Ix.
Hx
- Exercise tolerance, equivalence in METS or NYHA
*class I, no limitation no dyspnoea on everyday activity, class II, some limitation, dyspnoea on everyday activity, class III, severe limitation, dyspnoea on even less than everyday activity, IV, dyspnoea at rest.
*The history suggests this may be difficult to assess given need for joint replacement, and likely limited exercise tolerance.
- recent admissions, medications, investigations
- SOB, orthopnoea, PND, ankle swelling
Ex
- JVP, hepatomegaly/ascites, leg swelling, lung creps, S3, hypoxaemia
Ix
- ECG
- Echo, LV systolic or diastolic dysfunction, valvular abnormalities, assessment of ventricular wall size/thickness
(Sep-2004 Q11) What is the role for radionucleotide imaging in the assessment of ischaemic heart disease prior to general anaesthesia for non-cardiac surgery?
.
(May-2004 Q1) A 50yo patient with a past history of well controlled ischaemic heart disease is anaesthetised for an emergency laparotomy. Thirty minutes into the surgery, you notice new ST segment depression on the ECG. Describe your management.
ST depression = myocardial ischaemia until ruled otherwise, and is an intraoperative emergency.
Fx include tachycardia/hyper/hypotension, anaemia, sudden blood loss, hypoxia
- ABC approach, and inform surgeon's (stop surgery if possible)
- 100% oxygen, adequate ventilation, MAP/HR (may need increased opioid/anaesthetic/beta-blocker/GTN/Aspirin/heparin or vasopressors, decreased anaesthetic depth)
- depending on severity / haemodynamic compromise / arrythmias may need to involve cardiology for PCI/urgent TTE/TOE. May be difficult to anticoagulate or thrombolyse given surgical setting
- postop Mx to include appropriate monitoring/disposition, referral to Cardiol/HDU/ 12-leadECG, serial enzymes
(Sep-2002 Q14) Discuss the methods available for investigating a clinical suspicion of acute postoperative MI.
.
(May-2001 Q1) A 63 yo man who lives independently, presents with a perforated ulcer requiring laparotomy. He has been treated for cardiac failure for 5 years. How would you assess the severity of his cardaic failure at the bedside?
.
(May-2001 Q2) Justify your choice of deep venous thrombosis prophylaxis.
.
(May-2001 Q3) How would you manage him if he developed pulmonary oedema during his surgery?
.
(Apr98) A 59 yo patient presents for the first time with a subacute bowel obstruction requiring laparotomy in the next two or three days. You are asked by the surgeon to review the patient because on admission his blood pressure is 210/120. Hypertension has not been previously diagnosed in this man and he is on no medications. Describe your assessment of his hypertension by history and examination.
.
(Apr98) How would you proceed with investigation of his hypertension if no cause was apparent from the assessment described above?
.
(Apr98) How would you manage his blood pressure in the peri-operative period if no cause had been found for this hypertension?
.
(Apr96) A man is to have a left shoulder arthroplasty under general anaesthesia. He has a permanent pacemaker located subcutaneously under the left clavicle. How would you assess this aspect of his condition pre-operatively?
.
(Apr96) What precautions would you take to prevent malfunction of the pacemaker?
.
(May-2005 Q11) Discuss the management options for an epidural abscess.
.
(Sep-2004 Q4) Outline the diagnostic criteria for an epidural abscess.
.
(May-2004 Q8) Discuss the indications for peri-operative antibiotic prophylaxis. Include consideration of the appropriate class of antibiotic for each indication.
.
(Sep-2003 Q9) A 35 yo man had a heart transplant 4 years ago. He now requires elective hip surgery. He is on cyclosporine, azathioprine and prednisolone. What are the implications of his immunosuppressive treatment for perioperative anaesthesia care?
.
(Sep-2003 Q10) How may the spread of blood borne viral infectious agents from patient to health care worker be minimised in anaesthesia?
.
(Jul98) A 35 yo man had a heart transplant 4 years ago. He is now troubled by pain from avascular necrosis of the head of the femur, and requires surgery. He is on cyclosporin, azathioprine and prednisolone. What are the implications of his immunosuppressive treatment for perioperative anaesthesia care?
.
(Jul98) How does the history of him having a heart transplant influence your anaesthetic management?
.
(Jul98) Describe the strategies which should be employed to minimise the risk of sepsis associated with his periperhal venous cannula.
.
(Sep-2005 Q7) How would you diagnose a clinically significant latex allergy occurring intra-operatively?
- could refer to intraoperative diagnosis, or diagnosis at a later stage looking back
- "significant" latex allergy is probably referring to anaphylaxis or type I hypersensitivity reaction. This often presents later after induction (30-60mins) into case (ie, after latex comes into contact with mucus membranes, etc).
- Hx may involve atopy, food allergy (bananas, avocado, nuts), hx latex allergy!, multiple urological procedures, spina bifida, healthcare workers, hx rash or Type IV hypersensitivity
- Specific testing may involve skin prick testing or intradermal testing with latex antigen (high sensitivity), or serological RAST tests (most specific, btu less sensitivie). Skin patch testing for Type IV reactions.
(Sep-2003 Q1) Describe the precautions that should be taken to prevent a patient with known latex allergy from having a reaction to latex in the peri-operative period.
Preop
- determine type of latex sensitivity (ie, dermatitis, Type IV or Type I hypersensitivity reaction
- organise 1st on list
- theatre ideally not used from day before (time for any latex aeroallergens to settle)
- ideally check in separately from other patients who may have come into contact with latex

intraop
- No latex in theatre (go by specific checklist, latex-free trolley)
Items that traditionally had latex include
- masks, NIBP cuffs, tourniquets, elastoplast, hair net elastic, stethescope tubing, GLOVES (especially powdered), anaesthetic circuit reservoir bag, ventilator bellows, some IV tubing/bungs, hydrocortisone/haemacell injection ports, PAC, IDC.
- These days virtually all of these latex-free already, gloves have to be out of room, and PAC balloon is still latex.
- proper time out to double check
- avoid "through traffic" in OT

Postop
- recover separately if possible

The pre-admit/recovery points are really for institutions with powdered latex gloves, otherwise ok.
(Aug-2000 Q7) Discuss serum tryptase as an investigation in current medical practice.
.
(Sep-2005 Q15) List the physiological effects of ECT and how they may be modified?
ECT consists of an electrical current applied to the skull/brain in order to stimulate a seizure (which is therapeutic for psychiatric illness).
- CVS
- initial parasympathetic predominance, with bradycardia, possibly hypotension, followedby sympathetic discharge, tachycardia, hypertension, possible myocardial ischaemia
* could be modified by pre-treatment with anticholinergic, or use of opioid/beta-blocker for 2nd phase
- CNS
- induces seizure which increases CMRO2/CBF/ICP
*hypoventilation, use of anaesthetic agent, although don;t want to decrease the seizure (if prolonged it should be aborted)
- MSS
- tonic-clonic convulsions with potential for muscle/bone injuries
* use of muscle relaxant to "modifiy" convulsion, and prevent trauma (although muscle soreness still common from sux and seizure)
(Sep-2002 Q10) As a result of a suicide attempt, a 22 yo male sustained a compound fracture of his lower tibia with extensive skin loss. Other injuries include a t12 and l1 crush fracture with no neurological sequelae. The patient suffers from schizophrenia. He is to undergo a free flap from the lower abdomen to the leg wound. What problems relating to his schizophrenia may impact on anaesthesia?
Schizophrenia is a pscyhiatric disorder characterised by positive/negative symptoms, and thought disorder.

it is treated by Dopamine antagonists (both typical/atypical). Problems can result from the disease, or the treatment itself.

Preop
- difficult with "consent". Unlikely to be deemed competent given they have just tried to commit suicide.
- difficulties with cooperation, rarely aggression/agitation
- thought disorder may impact on postoperative care, including analgesia, nursing care,"", cooperation with rehab, etc
- Impact of medications
*dystonias from dopamine antagonism
*hypotension from alpha-adrenergic blockade (older drugs)
*obesity, glucose intolerance from newer atypicals
*rarely myocarditis/cardiomyopathy from clozapine, agranulocytosis
*NMS (if increase in dose, or recent start to treatment), may b confused with MH
(Sep-2002 Q11) Evalutae the options for postoperative analgesia.
.
(Sep-2002 Q12) He becomes acutely agitated in the post anaesthesia care unit. Discuss the possible causes of this problem.
.
(May-2002 Q14) Outline the anaesthetic risks specific to patients undergoing electro-convulsive therapy.
.
(Apr99) A severely depressed 60yo man presents for electroconvulsive therapy. His history includes controlled hypertension and stable ischaemic heart disease. He is taking MAO inhibitor phenelzine for depression. What are the implications of his phenelzine therapy of relevance to GA for ECT?
.
(Apr99) Describe the physiological effects of electroconvulsive therapy relevant to his anaesthetic management. Omit any considerations concerning phenelzine.
.
(Apr99) Discuss your choice of induction agent for this procedure.
.
(Jul97) Describe the clinical features of an overdose of tricyclic antidepressant.
- mydriasis, coma, hyperreflexia, seizures, hypotension
- sinus tachy, prolonged QRS/QT interval, VT/VF, heart block
- activated charcoal to prevent absorption if presents within 1 hr
- Na HCO3, to aid excretion, and stabilise Na channels in myocadium
- phenytoin is 1st line anti-arrythmic
- dialysis NOT effective
- intubation/ventilation if necessary
Finals Oct-2012 Q1 You are asked to anaesthetise an 80 year old lady with dementia and a fractured neck of femur. She is on no other medication. 1. What are the issues in assessing pain in this patient (50%). 2. What would you prescribe for postoperative analgesia and why? (50%)
1)
- what is the degree of impairment of intellect and communication?
- utilising other forms of communication (ie, non-verbal, patient wincing if moving broken leg)
- third party reports from carers/family that "understand" patient
- coexisting medical problems which may worsen impairment (ie, pain itself, dehydration, electrolyte imbalance, other injuries, etc)
- should be a functional assessment versus a self-report if there is difficulty with this

2)
age-related changes in pharmacokinetics and pharmacodynamics - the possibility of ease of administration to allow for patient cooperation - Rationale for drug choice, route of administration, side-effects, NSAIDS, opioids,
NMDA receptor antagonists, antineuropathic agents and the place of local anaesthetics (central neuraxial or regional blockade).

I chose fascia iliaca catheter, small dose oral opioids, regular paracetamol (dosing reduced, and review). due to patient variability, etc).
(Oct-2009 Q5) A woman who is 10 weeks pregnant presents to the Emergency Department with a closed tibial shaft fracture. 1. Classify the drugs used in pain management according to their safety to use at this stage of pregnancy. (40%) 2. What are the options available for perioperative pain management for this patient? (30%) 3. What would you recommend? Justify your choice. (30%)
- Pregnancy drug classification is as per ADEC
A
- Taken by a large no of pregnant women without proven harmful effects (ie, paracetamol, thio, sux, nitrous)
B1
- Taken by a limited no of women, with no proven harmful effects, AND animal studies also show no increase (gabapentin, ropivacaine)
B2
- Limited no of women, without harmful effects, AND animal studies also limited (sevo)
B3
- Limited again, BUT show increase in harmful effects in animals (des, ketamine)
C
- drugs that have or a suspected of having caused harmful effects by their pharmacological action, but reversible, and no malformations (propofol, NSAIDS)
D
- suspected of having caused harmful effects and increased malformations (anticonvulsants, ACE I)
X
- malformations, should not even be used by those who COULD be pregnant (?).

2.
options are pharmacological and non-pharm.
Non
- ice, elevation, plaster
pharm
- simple, opioid-based, regional-based regimes
avoidance of NSAIDS and tramadol. opioids for short duration
3.
regional technique may be ideal, depending on care for patient postop (ie, nursing acuity, pain team, etc).
neuraxial or sciatic nerve catheter/block
multimodal approach to reduce need for opioids.
(Apr-2009 Q14) A previously healthy 28-year-old male has persistent pain 12 weeks after compound fracture to his lower leg and is on slow-release oxycodone 80mg twice daily and immediate release oxycodone 20mg 4 hourly. Discuss the advantages and disadvantages of switching his opioid to methadone in this situation and how this may be achieved safely.
Methadone is an opioid with unique properties, and is used in chronic pain mx or opioid addiction.

Advantages
- good bioavailability with almost any mode admin
- no active metabolites
- good analgesia for long duration
- has NMDA actions, and monoaminergic pathway actions making it useful in neuropathic pain
- incomplete cross-tolerance with oxycodone means a potential reduction in dosing
- cheap

Disadvantages
- long, variable elimination half-life, with potential for drug interactions (CYP 450)
- variable dosing equivalents published
- risk of under/over dosing
- special prescribing conditions and "stigma" attached (drug abuser)
- potential for QT prolongation


Plan for Mx
- ideally close observation of conversion (inpatient ideally)
- need a full pain assessment looking at biopsychosocial factors, and discussing a full treatment plan that involves pharmacological and non-pharmacological mx
- maximise non-opioid multimodal analgesia
- conversion variable - will try 10:1. Oxycodone to morphine equivalent = 420mg oral morphine, therefore approx 40mg methadone, give in 2-3 divded doses.
- prescribe morphine/oxycodone as breakthrough as required
- careful monitoring over several days for sedation / resp depression or inadequate analgesia


Coversion
- many different ones.
- some just use 10:1 blanket rule (morph/meth)
- some 4:1 (0-90mg morph), 8:1 (90-300mg), 12:1 (>300), or versions thereof (up to 20:1 for 1000mg morph).
Doesn;t matter as long as you say you need to titrate safely and slowly.
(Oct-2008 Q11) List the risk factors for the development of chronic pain following a surgical procedure. Outline possible mechanisms for the progression of acute to chronic pain.
- Pain is defined as an "unpleasant sensory and emotional experience, caused by actual or potential tissue damage, and defined in terms of such damage".
- Chronic pain is pain lasting for for than 3/12 following a surgical procedure (persistent post-surgical pain).
- Could be defined as anaesthetic/surgical/patient factors or pre/intra/post op factors (Ex. report went 2nd).
* Preop
- preoperative severe pain, lasting > 1/12
- repeat surgery
- workers comp
- catastrophising, patient anxiety/depression
- preop chemo/radiothx
- female gender
- younger age (but not neonatal)

*Intraop
- surgical technique associated with nerve damage (ie, amputation, thoracotomy), "nerve-sparing" techniques, such as VATS/laparopscopy may be beneficial
- other high risk surgeries include
mastectomy, caesarian, inguinal hernia repair, cholecystectomy, amputation
- a regional technique with LA may reduce CP postop
- use of ketamine or gabapentin (breast) may reduce CP postop

*Postop
- severe postoperative pain
- acute neuropathic pain
- catastrophising, anxiety/depression, poor coping strategies
- postop chemothx/radiothx

Mechanisms
- Peripheral sensitisation
"inflammatory soup", and lowering of threshold for nearby neurons, with increased firing rate
"sprouting" of peripheral neurons due to nerve injury and ectopic firing
upregulation of sympathetic adrenergic receptors (ie, CRPS)
- central sensitisation
prolonged, intense firing from periphery results in "unblocking" of NMDA receptors, which results in stimulation of central pathways
WDR neurons are activated (higher firing rates causes more pain)
also results in gene transcription changes, and changes to things like opioid-receptor G-proteins
-decreased descending inhibition and cortical re-mapping
- psychosocial factors also play a role in progression to chronic pain (sick role, secondary gain)
(May-2008 Q5) A 65 yo female who weighs 85kg and is 165cm tall (BMI 31) is scheduled for TKR surgery. Discuss the pros and cons of intrathecal morphine for post operative analgesia in this patient.
- Intrathecal morphine is given in small doses (ie, 1/50th) of IV, must be preservative-free.
- analgesic effect can last up to 24 hrs

Pros
- effective analgesic, reduces systemic opioid requirements, and pain scores
- simple technique, single-shot spinal, can be used in conjunction with LA for anaesthesia for procedure (no need for GA)
- a single-shot block potentially less complicated/risky than postop epidural analgesia, with risk of epi haematoma & DVT prophylaxis
- no motor block, and no need for expensive pumps, etc

Cons
- long-lasting analgesia may mean long-lasting adverse effects
*PONV
*Pruritis
*urinary retention
*Respiratory depression and sedation (delayed), especially relevant in this patient who is obese, and may have other risk factors for postop resp depression such as OSA
- although reduces systemic opioid usage, does not reduce opioid-related adverse effects (as above)
- requires dural puncture so risk of infection, bleeding, PDPH, failure of technique
- TKJR is particularly painful and requires a higher dose of ITM (compared with THJR)
- although cost effective in terms of equipment used, many hospitals would require regular nursing observations and potentially HDU for ITM, which negates cost-effectiveness
(May-2008 Q13) Evaluate the role of gapapentin in acute and chronic post surgical pain management.
Gabapentin is an anticonvulsant that blocks voltage-gated ca channels in the dorsal horn of the spinal cord. It is given orally and excreted renally.

Acute Pain
- has been studied, often as single preoperative dose, and does significantly reduce opioid usage, and improve analgesia
- controversial whether it reduces opioid-related adverse effects though. Some evidence to suggest it reduces nausea, pruritis and urinary retention, but increases sedation and dizziness.
- may be particularly useful in acute neuropathic pain (although no studies to confirm this, more extrapolated from chronic neuropathic pain studies).
- Cochrane review says a single dose of 250mg has a NNT of 11, which is inferior to most simple analgesics.

Chronic Pain
- useful for chronic pain, especially neuropathic
- gabapentinoids becoming 1st line due to lower adverse effect profile vs TCA's
- also useful in PREVENTION, when used in single preoperative dose, and postoperatively in some cases
- evidence for Breast (particularly), hysterectomy, cardiac, hip arthroplasty
- evidence is also good in non-surgical states (diabetic neuropathy and post-herpetic neuralgia).
- GBP and pregabalin now both on PBS for chronic neuropathic pain states.
(Sep-2007 Q14) A clinical trail is planned to evaluate a new analgesic. Discuss the ethical considerations in having a placebo group in the trial.
.
(May-2006 Q4) Discuss the role of non steroidal anti-inflammatory drugs for post operative analgesia in adult day surgery patients.
NSAIDS
- nsNSAIDS & COX-2 inhibitors both work on COX enzymes to reduce prostaglandin formation which is resonsible for pain and inflammation

Day surgery
- requires "alertness, analgesia, ambulation and alimentation" (4 A's).

Advantages
- very effective for mild-moderate pain, and as part of a multimodal approach in severe pain (Day surgery likely to have more moderate-mild pain given requirements to be able to control pain at home).
- reduces opioid requirements (and nsNSAID's also reduce opioid-related adverse effects).
- multiple ways of adminstration

Disadvantages
- several significant possible adverse effects
*renal impairment
*peptic ulcer disease
*bronchospasm (in 15% of asthmatics)
*bleeding (nsNSAIDS in adult tonsillectomy)
*higher thrombosis (MI/CVA) with cox-2 inhibitors
Therefore patient co-morbidities must be carefully looked at
- care with cessation of drug at home (ie, give guide to how long to take).
(May-2005 Q4) Discuss the requirements for and limitation of the use of patient-controlled analgesia (PCA) as a technique.
PCA is a technique where patient's are able to self-administer a small bolus of (usually) opioid medication, with associated lock-out period for safety, the idea being that patient's will self-titrate opioid requirements to their own level of analgesia.

Requirements
- Patient
speak english, or adequate understanding via written info
mental requirements (ie, age >6-7, dementia, etc)
physical requirements (able to trigger button or other device)

- Staffing
adequate staff number and training to work pumps safely and perform observations (required regularly)
protocols for staff
availability of anaesthetist to trouble-shoot (ideally run by an APS, but not always available)

- Equipment
locked pump
anti-siphon and anti-reflux valves
programmable lockout, dose, maximum 4/24 limit
IV access

- Medical Fx
requires loading with opioid prior to starting
IV access (not really)
availability of naloxone
no other sedatives or opioids prescribed

Limitations
- require loading, and difficult to "catch-up" once patients in severe pain
- costly to run, and need close nursing supervision (at least initially)
- analgesia does not last "long" periods, need to continue to bolus during night time

from APMSE 2010
- PCA vs traditional IM opioids
* better analgesia, better patient satisfaction, HIGHER opioid dose, MORE pruritis
*no real difference between opioids except lower pruritis with fentanyl v morphine
*morphine bolus 1mg ideal, 0.5mg inadequate, and 2mg resp depression, but previous opioid use should be taken into account, as well as age
*background infusions likely to cause higher adverse effects, but consider in previous opioid use
(May-2004 Q6) A 71 year old man presents with acute herpes zoster involving the ophthalmic division of his left trigeminal nerve. He complains of severe unrelenting facial and eye pain which started 3 days ago. Discuss the pharmacological treatment options. Include information about the relevant efficacy of the treatments you prescribe.
Herpes Zoster involves reactivation of the HSV in the DRG or CN ganglion, to cause "shingles" in that nerve distribution. It increases with age (a lifetime incidence of 20-30%).

Pain associated with HZ is acute HZ pain (approx 80%, an acute neuropathic pain), or post-herpetic neuralgia (approx 20% of HZ will develop PHN, and 75% over 75yo).

Treatments not only include that of the acute pain from AHZ, but also prevention of PHN.
- antivirals (ie, aciclovir), if given within 72 hrs of rash onset, reduces duration of acute pain/rash (no preventative effect as previously thought)
- acute treatment options should involve a multimodal approach, including paracetamol, tramadol and opioids (level II evidence for oxycodone).
- other anti-neuropathics include TCA (amitriptyline for 90 days reduces PHN, but no evidence for acute use), gabapentin reduces acute pain in single dose, but not useful given for 28 days
- topical lignocaine effective (?eye use)
- topical aspirin (? eye use).

overall prevention is best with vaccination, but not an option in this man.
SAQ-Jul05-Q4 Describe the features and management of phantom limb pain.
PLP is a type of neuropathic pain occurring after amputation (usually of a limb).

There are 3 types of pain/sensations occurring after limb amputation - stump pain, phantam limb sensation, and PLP.

Features
- very common (up to 70-80%) following limb amputation
- occurs within a few weeks of procedure
- usually DISTAL in limb
- neuropathic features such as shooting/burning/cramping/aching
- can be intermittent or constant
Risk Fx
*severe pre-amputation pain, lower limb, persistent/poorly controlled stump pain, adults (V kids)

Management
- Pharmacological & non-pharmacological
*Pharm
evidence that perioperative epidural (preincision) and ketamine for 72 hrs (pre) MAY reduce SEVERE PLP, but not PLP overall
for ACUTE PLP
- opioids useful
- TCA's, tramadol, GBP
- calcitonin

*Non-Pharm
Physical therapy
- acupuncture, ultraspund, TENS = weak evidence
CBT
- mirror therapy, mental imagery movement, sensory discrimination training
(Sep-2006 Q9) Nitrous oxide should not be used routinely as a component of general anaesthesia. Discuss.
.
(Sep-2006 Q12)Discuss the role of ketamine in current anaesthesia practice.
.
(Sep-2005 Q13) The hospital pharmacist notifies you as Director of Anaesthesia that Thiopentone is to be withdrawn from the hospital formulary due to minimal usage. Outline and justify your response.
- need to "audit" departmental usage before replying, canvas other staff/opinions.
- use in RSI, perhaps slightly better point of "LOC", better maintenance of cardiac output due to HR increase.
- propofol allergy (rare)
- use in TBI (burst suppression), or status epilepticus
- suggest ongoing audit of usage, and criteria for use given expense
(May-2006 Q9) Describe the clinical features and management of bupivacaine toxicity.
.
(May-2006 Q14) Discuss the role of desflurane in current anaesthesia practice.
.
(May-2005 Q5) Compare the relative merits of gelatin-based intravenous solutions and dextran intravenous solutions.
.
(May-2005 Q9) What significant side-effects are associated with the use of anti-emetic agents?
.
(Jul-2004 Q5) Discuss the problems of clopidogrel for the perioperative period
.
(Sep-2004 Q14) What are the relative merits of sodium nitroprusside and glyceryl trinitrate in the control of blood pressure in anaesthetised patients?
.
(May-2004 Q9) Compare and contrast thiopentone and propofol for use in rapid sequence induction of anaesthesia.
.
(May-2002 Q13) Outline the issues to be considered when a patient wishes to continue self-prescribed herbal preparations in the peri-operative period.
.
(Sep-2001 Q6) Outline the pharmacology of naltrexone relevant to providing anaesthesia and postoperative care for a patient on naltrexone undergoing major abdominal surgery?
.
(Aug-2000 Q8) Compare the relative merits of gelatin-based intravenous solutions and dextran intravenous solutions.
.
(Apr-2000 Q12) Describe the pharmacokinetics and dosing schedule of paracetamol for post op analagesia.
.
(Apr98) Use of at least 30% O2 in the inspired gas has been traditional practice in anaesthesia for healthy adults. Is this practice valid?
.
(Apr97) Outline the undesirable effects of intravenous protamine sulphate.
.
(Aug96) What are the advantages and disadvantages of the use of propofol for sedation of intensive care patients?
.
(Oct-2012 Q14) The mallampati test is a commonly used bedside screening tool to assess the probability of a difficult intubation. Explain the terms sensitivity, specificity, PPV and NPV when applied to this test.
Sensitivity = TP/(TP+FN)
The proportion of patients with difficult airways that the MP test predicts.

Specificity = TN/ (TN+FP)
- The proportion of patients without difficult airways that a negative test predicts.

PPV = TP/ (TP+FP)
Given the MP test is positive (or score high), what is the proportion of difficult intubation?

NPV = TN/ (TN+FN)
Given the MP test is negative, what is the proportion of non-difficult intubation.

The last 2 depend on the incidence of difficult intubation as well as the sensitivity and specificity.

Difficult intubation is rare, and so the PPV will be low, and the NPV will tend to be high.
(Apr-2009 Q15) How is appropriate sample size for a clinical trial determined? (50%) What are the ethical implications of using an inappropriate sample size in a clinical trial? (50%)
Sample Size
- Type I (alpha) error = incorrectly rejecting the null hypothesis
*usually 0.05, if smaller, need bigger SS.

- Type 2 (beta) error = incorrectly ACCEPTING null hypothesis (safer to keep practice the same)
* usually 0.20, and POWER = 1-beta
if smaller, then bigger SS

- Effect size = what is clinically relevant difference?
*bigger effect size = SMALLER SS

- Variance of population
*bigger will need bigger SS
*only one not chosen (need to determine from pilot study or other data

SS calculated from computer software these days taking these into account.

Ethics
- SS too small
may have misleading results, never powered to find a difference, but putting patients through risk of trial
waste of time/resources
not finding a possible benefit?!
- SS too big
another waste of time/resources
prolongs suffering of those denied treatment

Should be monitored by data safety committee (interim analysis).
(Oct-2008 Q13) Explain the terms sensitivity, specificity, positive predictive value and negative predictive value when applied to a diagnostic test.
.
(May-2008 Q14) Describe the advantages and disadvantages of multi-centre trials in anaesthesia research.
.
(Sep-2005 Q2) Discuss ways in which you can decrease bias in a clinical trial for a new antihypertensive agent.
Bias is a "systematic deviation from the truth".
- part of decreasing bias is formulating a clear question/null hypothesis, and literature search (to see what factors might influence the point of interest, therefore potentially allowing for randomisation/stratification of these factors)
- randomisation most important. May involve stratification of KNOWN variables affecting outcome of interest. need strict exclusion criteria
- blinding decreases observer and reporting bias.
- measurement bias (objective, ie, machines to measure BP, and no rounding)
- report negative trials (publication bias).
(May-2005 Q12) Discuss the value of case reports to anaesthetists in the era of evidence based medicine.
.
(Sep-2001 Q14) What are the features of systematic reviews which contribute to their value in the context of evidence based medicine?
A systematic review is a literature review based on a specific research question, in which the goal is to identify, appraise, select and synthesize the highest quality evidence on a subject.

Steps involved
- Defining research question
- searching for studies (must state which databases searched)
- selection/data collection
- assessing risk of bias
- analysing data, and META-ANALYSIS
- addressing reported biases
- presenting results
- interpreting results and concusions

IDENTIFY/APPRAISE/SELECT/SYNTHESIZE

A SR of all RCT's on a subject is usually given the highest "level of evidence" in terms of evidence-based medicine
(Apr97) A clinical trial is planned to evaluate a new analgesic. Outline the methods which could be used to reduce bias in this trial.
Bias = Sytematic deviation from the truth

Blinding (triple if possible).
- reduces reporting, observation and analytical bias.
Choosing a valid measurement tool that is used for everyone (measurement bias), and is objective (observer/reporter bias).
Randomisation
- selection bias to some extent
- journals should publish NEGATIVE studies as well as positive studies (publication bias).
- statistics should be also performed by a blinded person.
(Apr97) A clinical trial is planned to evaluate a new analgesic. What are the ethical considerations in having a placebo group in the trial?
.
(Aug96) From a study of two groups of patients, each of 43, it is concluded that drug A is better than drug B because fewer patients vomit when given A, (p=0.04). What is your view of the sample size with respect to the conclusion?
.
(May-2012 Q1) In regard to serotonin syndrome: a. What are the risk factors? (20%) b. what are the clinical manifestations? (40%) c. What is the treatment for an acute epidsode of serotonin syndrome? (40%)
a) drugs which increase serotonin levels (initiation, dose increases, etc)
- SSRI's, TCA's, tramadol, pethidine, St John's Wort, Ginseng, Bupropion, MAOI, fentanyl, Ecstasy/Cocaine, Ondansetron (due to inhibition of CYP450)

b)
- Changes in CNS/CVS/Autonomic instability
- confusion, coma, agitation
- fever/sweating
- tachycardia, labile BP, flushing
- clonus/hyperreflexia, rigidity, shivering
- mydriasis, ataxia
DDX
- NMS, MH, Thyroid Storm

C)
- stop serotonergic treatments
- Supportive
A, B, C
may need intubation/ventilation, BP support, rehydration, cooling
- Specific
Cyproheptadine, Chlorpromazine
Usually need ICU monitoring/treatment
(Apr-2009 Q5) A 70 year old man with a 10 year history of Parkinson's disease presents for a total knee joint replacement. He is on levodopa / carbidopa five times a day. Outline the main issues to consider in relation to his Parkinson's disease in planning the perioperative management of this patient.
PD is a degenerative neurological movement disorder affecting dopaminergic neurons.

Main issues:
- cessation of oral therapy may worsen PD (no current IV agent used regularly), therefore minimise fasting perioperatively
- anti-dopaminergic agents (antiemetics) are contraindicated
- autonomic neuropathy (risk of reflux/aspiration, cardiovascular instability)
- difficulties with nutrition, postop mobilisation, communication
- withdrawal of PD meds may lead to NMS

- Overall may lead to differences in monitoring (ie, A-line for instability), ETT/RSI (aspiration), aggressive non-antidopaminergic antiemetics and potential for NGT for PD therapy, RA/multimodal analgesia to preserve lung function
(Oct-2008 Q15) What symptoms and signs suggest the presence of OSA in a patient presenting for pre-operative assessment? How does the presence of OSA alter your anaesthetic plan?
Sx/Sx
- Hx
Snoring loudly, observed apnoeas by partners or parents, daytime somnelence, hyperactivity and behavioural problems (kids), previous surgery/investigations such as sleep studies

- Ex
often obese (BMI > 35 adults), male (adults), neck circumference (>40), age > 50, Hypertensive
micrognathia, retrognathia, macroglossia, increased OP soft tissues, large tonsils/adenoids, syndromic (craniofacial abnormalities, esp children, such as Down's, Pierre-Robin, Goldenhaar, etc)

May have had investigations such as polysomnography

Anaesthetic Plan
- No/minimal sedation due to increased sensitivity
- potentially premed antacid (if obese), preparation for difficult airway (particularly BMV), or at least of appropriate positioning
- would choose short-acting anaesthetic drugs (propofol or desflurane) intraop
- minimise opioids, and short-acting opioids if necessary
- maximise non-opioid analgesia (multimodal approach with regional, etc)
- extubate awake, and sitting/lateral, potential for early nasopharyngeal airway
- CPAP in PACU if required, certainly on ward
- may have increased risk of apnoeas/obstruction for 48-72 hrs depending on anaesthetic technique and surgery
- HDU monitoring possibly, CPAP for otherwise stable patients on ward, overnight pulse oximetry if necessary
- may not be suitable for day stay unless stable disease, CPAP use, and surgery ok for non-opioid analgesia. Otherwise consider overnight admission.

2012 SAMBA guidelines on OSA and Day surgery
- OK if known OSA, stable/no comorbidities, and use CPAP at home (and minimal opioids)
- maybe ok if suspected OSA, no comorbidities/stable disease, and low risk surgery/no opioids
- consider non-ambulatory surgery for suspected OSA with other comorbidities or type of surgery precludes (ie, airway), OR known OSA and refusing CPAP.
(Sep-2007 Q15) Draw flow volume loops associated with a) Fixed upper airway obstruction; b) Variable extrathoracic airway obstruction and c) Variable intrathoracic airway obstruction.
a) Fixed
Will have difficulty on inspiration & expiration

b) Variable Extrathoracic obstruction (ie, upper tracheal)
collapse on inspiration, and opens on expiration (positive pressure from inside airway)

c) Variable Intrathoracic obstruction
Col
a) Fixed
Will have difficulty on inspiration & expiration

b) Variable Extrathoracic obstruction (ie, upper tracheal)
collapse on inspiration, and opens on expiration (positive pressure from inside airway)

c) Variable Intrathoracic obstruction
Collapse on expiration (external positive pressure), and ok on inspiration
(Sep-2006 Q5) List and explain the typical electrolyte abnormalities of chronic renal failure.
Hyperkalemia
- due to decreased ability to secrete in distal nephron
- also due to increasing acidemia, there is exchange across cell membranes for H+ ions

Na
- can be normal, high or low. generally Na is retained, but water is also, so probably minimal change in serum sodium concentration

Mg
- handled similarly to K
- poor excretion via kidney leads to hypermagnesemia

Hypocalcemia / Hyperphosphatemia
- decreased calcitriol production via kidney results in decreased GI absorption of Calcium
- decreased PO4- excretion via nephron leads to hyperphosphatemia
- resulting hypoCa/HyperPO4 leads to increased PTH secretion from PTH gland (secondary HyperPTH), which results in Ca mobilisation from bone and renal bone disease (Calcium is deposited in soft tissues, otherwise it would bind with PO4 and be insoluble)

- decreased HCO3 and progressive metabolic acidemia
- due to decreased ability to regenerate HCO3 in kidneys
- decreased ability to excrete H+ ions also due to decreased PO4- excretion (titratable acids)
(May-2006 Q3) A sixty-five year old woman presents for a total abdominal hysterectomy. She has non-insulin dependent diabetes mellitus that is normally controlled with an oral hypoglycaemic agent. Describe your perioperative management of her blood sugar.
Ideally, this patient's blood sugar should be controlled between 6-10 perioperatively, to reduce risks of hyperglycaemia (such as increased wound infection), and hypoglycaemia.

She MAY require insulin at some stage perioperatively, +/- consultation with endocrinology.
Institutional protocols should be followed where in place.

Her oral hypo-glycaemic medications should be with-held on the day of surgery, and ideally she should be operated on 1st on the list (to minimise fasting times).

Her BSL should be checked preop, and every 2/24 until eating again.
If BSL high pre-op, a sliding scale of s/c short-acting insulin could be written (<10)
If low pre-op, a dextrose infusion should be started.

Intraop
- regular BSL checking
- No 5% dextrose unless hypoglycaemic or requiring insulin
- may require insulin infusion if unstable

Postop
- if able to E&D, can have normal nocte meds
- sliding scale could be used for high BSL's


Examiners report also wanted discussion about different OHG agents, assessment of HBA1c.

Patients on Insulin
- in general, need to continue insulin (especially type I).
- and long-acting basal insulin can be continued (nocte doses normal, mane doses 50%)
- short-acting insulins should not be given
- intermediate-acting (protophane), 50% dose mane
- pre-mixed insulins, 50% given as protophane
(Sep-2005 Q3) What are the symptoms, signs and anaesthetic implications of an autonomic neuropathy associated with diabetes mellitus?
Diabetes may cause a small fibre neuropathy of sensory and autonomic nerves.
AN may present as GI or CVS sx.
GI
- decreased gastric emptying, reflux, gastropathy in severe form with vomiting
*risk of aspiration with GA, and may need RSI/cricoid
CVS
- decreased HR variability with respiration, "denervated" heart
- silent ischaemia, arrythmias
- orthostatic hypotension, dizziness
*potential for haemodynamic instability with anaesthesia, poor heart rate response, impaired response to valsalva
*potential for Myocardial ischaemia
* poor temp regulation
*may require increased cardiac investigations preop depending on risk, may need IAL/5lead ecg.
(May-2005 Q13) How would you assess a patient's thyroid function preoperatively at the bedside?
By Hx/Ex/Ix

Hyperthyroidism
Hx
- weight loss, increased appetite, heat intolerance, sweating, palpitations
Ex
- "frightened stare" of Graves, exopthalmos
- tremor, clubbing, sweaty palms
- proximal myopathy
- tachycardia, AF,
- increased reflexes, pretibial myxoedema

ECG - may show ST or AF

Hypothyroidism
Hx
- weight gain, tiredness, fatigue
- dry skin, hair, cold intolerance, constipation
Ex
- slow mentation, hypothyroid "voice", dry skin, periorbital oedema, "doughy skin"
- mild bradycardia, hypothermia
- reflexes with delayed relaxation
- peripheral neuropathy

Also need to explore patient notes, hx of previous medications/surgery, etc, known TFT's, thyroxine replacement, etc.
(May-2002 Q10) A 75 yo woman with severe long standing rheumatoid arthritis presents for hip replacement. How will her rheumatoid arthritis influence your anaesthetic management?
.
(May-2002 Q11) She has the operation under a spinal anaesthesia and light sedation. Just as the femoral prosthesis is being cemented in, she loses consciousness. How will your manage this?
.
(May-2002 Q12) Discuss the advantages and disadvantages of using cox-2 selective non-steroidal anti-inflammatory drugs as part of your postoperative analgesic plan in this patient.
.
(Jul97) Describe the management of a thyroid crisis occurring 12 hrs after thyroidectomy.
.
(Apr97) Discuss your options for providing muscle relaxation during surgery.
.
(Apr97) 48 hours post-operatively the patient becomes profoundly weak. How would you manage this?
.
(Aug96) What are the anaesthetic implications of autonomic dysfunction in a diabetic patient?
.
(Apr96) An apparently healthy 71 yo woman presents with a fractured neck of femur requiring internal fixation. She smokes 15 cigarettes each day and drinks approximately 30 gm of alcohol daily. She takes no medications. Justify the blood tests you would request when making your pre-anaesthetic visit.
.
(Apr96) How would you interpet a pre-operative serum potassium of 3.2 mmol/L in this woman if the laboratory normal range is 3.5-5.0 mmol/L?
.
(Apr96) If her pre-operative serum potassium had been 5.7 mmol/L, how would this influence your anaesthetic management?
.
(Oct-2012 Q9) A developmentally delayed, uncooperative adult requires a magnet resonsance imaging scan for investigation of deteriorating control of seizures. What issues do you foresee in terms of providing general anaesthesia in the MRI suite for this patient?
Could be answered by the 5 P's.
Patient/Pathology, Place/Protection/Postop
or patient/MRI fx

MRI suite: remote environment, remote access to patient inside scanner, personnel unfamiliar with anaesthesia requirements, availability if emergency help, monitoring issues, dangers of ferromagnetic anaesthetic equipment - burns or missiles


- patient factors: ferromagnetic materials within the patient, consent, cooperation with anaesthesia induction, premed/IV/Gas, utility of previous GA's and carer knowledge, epilepsy control & associated conditions (ie, syndrome associated with CHD, etc)
(May-2008 Q15) Outline the problems of providing anaesthesia for an adult in the MRI suite.
.
(Sep-2005 Q6) How does anaesthesia alter temperature homeostasis?
Increases the interthreshold range, by decreasing threshold for vasoconstriction and shivering, and increasing for sweating (marginally).
3-phase drop in core temp with anaesthesia
- 1, due to loss of tonic vasoconstriction and REDISTRIBUTION of heat from core to periphery. no net heat loss at this stage
- 2, due to net heat loss, still no vasoconstriction or shivering given decrease in threshold and change in IT range, core temp drops further
- 3, "plateau", again no real net loss. body starts vasoconstricting again.
(May-2005 Q10) What are the problems with the prone position for surgery?
see 256.
(Sep-2004 Q1) Discuss the advantages and disadvantages of the methods used to avoid hypothermia in the operating theatre.
.
(Sep-2004 Q3) What are the problems of using the beach chair position for shoulder surgery?
.
(Apr98) How may physiological control of temperature regulation in adults by altered by general anaesthesia?
.
(Jul97) A 70 yo woman is scheduled for colectomy under GA. Comapre the effectiveness of the methods you would use to prevent her becoming hypothermic by the completion of surgery.
.
(Jul97) Describe the adverse effects of a core temperature of 34.0 degrees centigrade at emergence from general anaesthesia.
This probably represents mild-moderate hypothermia.

Several problems can occur

- increased arrhythmias, myocardial ischaemia (vasoconstriction, increased myocardial oxygen demanad from shivering and increased SVR/afterload)
- increased wound infection and poor wound healing rates (due to tonic vasocontriction)
- blood loss and coagulopathy (platelet dysfunction and coagulopathy), increased transfusion requirements
- delayed emergence from anaesthesia
- delayed metabolism of drugs, including NMBA's
- delayed recovery stay (and hospital stay).

Nice table in CEACCP - Hypothermia & thermoregulation 2003.
(Sep-2007 Q11) An 18 year old otherwise healthy female is to have 2 impacted wisdom teeth surgically removed as a day stay patient. Describe and justify features of your anaesthetic technique that may help prevent the common postoperative problems you would anticipate in this patient.
.
(May-2004 Q3) Describe and justify your usual anaesthetic technique for colonoscopy in an otherwise uncompromised patient
.
(May-2003 Q5) Desflurane should be used only for day case procedures. Discuss this statement.
.
(Sep-2002 Q4) (Jul98) A healthy 34 yo man requires colonoscopy under intravenous sedation because of a strong family history of bowel cancer. Describe the composition and effects of bowel preparation solutions commonly used before colonoscopy.
.
(Sep-2002 Q5) (Jul98) Soon after colonoscopy begins his pulse rate falls to 40 beats/min and blood pressure to 60/40 mmHg. Describe your management.
.
(Sep-2002 Q6) (Jul98) What criteria would need to be met before he can be discharged home from the day procedure unit?
.
(Aug96) Outline the criteria you would use in determining a patient's suitability for discharge from a day surgery unit.
.
(Oct-2012 Q7) In regard to total parenteral nutrition: 1. What are the indications? (30%). 2. What are the complications? (70%).
1)
- malabsorption of any cause
*short gut syndrome, ileus
*increased metabolic requirements (burns, sepsis)
- to prevent muscle wasting
- to improve wound healing and clinical outcomes

2)
relating to CVC
- bleeding/haematoma
- infection, local and systemic
- VTE
- pneumothorax
- cardiac tamponade if malpositioned

Relating to TPN
- electrolyte disturbance (Na, K, Mg, Ca, PO4
- hyperglycaemia, hypoglycaemia
- fluid overload
- hyperlipidemia
- acidosis, liver dysfunction apparently
- trace element deficiency
- immunosuppression
(Oct-2008 Q6) You are covering ICU in your local district hospital when a 14-year-old boy presents to your ED obtunded and hypotensive with a rash suggestive of meningococcal sepsis. Describe your resuscitation
.
(Sep-2007 Q6) Define circulatory shock Categorise the causes of circulatory shock and give an example in each category.
.
(Sep-2007 Q7) A 25 year old, 65kg woman with acute severe asthma requires intubation and ventilation. Explain the problems associated with initiating ventilatory support in this patient and describe how you would overcome them.
.
(May-2007 Q1) What are the principles of ventilatory management of patients with acute respiratory distress syndrome (ARDS)?
.
(May-2007 Q5) Discuss the usefulness of the continuous measurement of mixed venous oxygen saturation in the intensive care patient?
.
(May-2007 Q13) Ambulance officers performing CPR with bag and mask ventilation. She has been rescued from a swimming pool. Describe how basic life support should be provided in the emergency department. She has no pulse and her ECG shows ventricular fibrillation. (2) Outline the advanced life support algorithm you would now follow.
.
(May-2007 Q15) Outline the steps necessary to diagnose brain death in a 38 year old woman who is comatose following a subarachnoid haemorrhage.
.
(Sep-2006 Q15) Discuss the usefulness of the ASA grading as a measure of perioperative risk.
.
(May-2008 Q6) The electrolyte results below were taken from a 38 year old woman found obtunded 30 hours after abdominal hysterectomy. She had no intercurrent llnesses prior tosurgery. Explain how these electrolyte abnormalities are most likely to have arisen and describe how you would correct them.
Result mmol/L Normal range mmol/L
Sodium 110 135-145
Potassium 3 3.0-5.0
Chloride 80 95-105
Bicarbonate 25 20-32
Glucose 5.0 3.0-5.5
Urea 3.0 3.0-8.0
Creatinine 0.06 0.06-0.12
Measured Osmolality 225 mosmol/kg 280-295mosmol/kg
.
(Apr-2009 Q3) A previously well 80kg 19-year-old male is anaesthetised for ORIF of # tib and fib. He has a RSI including Suxamethonium and is intubated and ventilated via a circle system at 12 breaths per minute and a TV of 700mL with a FiO2 of 0.5. He has had 500mcg of fentanyl and anaesthesia is maintained with 1.5 MAC Sevoflurane. He develops an increasing sinus tachycardia to 160/min with frequent ventricular ectopic beats and his ET CO2 rises to 60mmHg despite increasing his ventilation. There is no rebreathing evident of capnography. ABGs now
pO2 105mmHg
pCO2 65mmHg
pH 7.12
HCO3 20.7mmol/L
BE -10
Outline the steps you would follow to manage this situation.
.
(Sep-2007 Q12) A 3 week old male infant who was born by uncomplicated vaginal delivery at term presents with projectile vomiting for 2 weeks. His weight is now 2.8 kg from a birth weight of 3.1kg. His presumed diagnosis is pyloric stenosis. His blood chemistry results are:
Measured Normal Range
Na 129 mmol/L 135-145 mmol/L
K 3.0 mmol/L 3.5-5.5 mmol/L
Cl 84 mmol/L 95-110 mmol/L
HCO3 36 mmol/L 18-25 mmol/L
Creatinine 69 μmol/L 20-75 mmol/L
Glucose 3.0 mmol/L 2.5-5.5 mmol/L
Explain how these abnormal results come about. Describe an appropriate fluid resuscitation regime for this infant. List the laboratory criteria by which you would consider him sufficiently resuscitated for surgery.
Pyloric stenosis is usually idiopathic, and presents in infants from about 4-8 weeks of age. presents with projectile, non-bilious vomiting, and appears hungry - followed by features of dehydration.
The result is a typical hypokalemic, hypochloremic, metabolic alkalosis.

Pathophysiology
- Vomiting gastric contents only = loss of H+/Cl/K+.
- Kidneys respond by excreting HCO3 (alkaline urine initially) due to resultant metabolic alkalosis
- due to hypovolemia there is activation of renin-AT-aldosterone system, which aims to conserve Na/H2O - this leads to excretion of K (the main cause of hypokalemia).
- Due to Na resorption, some HCO3 starts to get resorped as well, due to not enough Cl to maintain electroneutrality. There is also preferential reabsorption of K+ for H+ in severe hypokalemia. Leads to paradoxical acid urine, and worsening of the metabolic alkalosis.

b) This infant requires fluid resuscitation, and is at least moderately dehydrated (>10% of body weight lost). Initial 10ml/kg bolus of Nacl would be appropriate, and continued until evidence of good peripheral perfusion (ie, Hr/BP normalise, MM, skin turgor, cap refill, urine output, colour, conscious state, etc. The extra volume may initially be enough to correct some of the acid-base disturbance.
K should be replaced - often is added to maintenance therapy (1/2 N/S & 5% dextrose + 20mmol/L KCL) at an increased rate (ie, 6ml/kg/hr)

c) The child needs to be adequately resuscitated with no signs of shock / body weight pre-=morbid.
The lab values for electrolytes should be basically NORMAL. A VBG/electrolytes within 6/24 operation should be obtained.
* Na>135, K>3.5, Cl>105, HCO3<26
*lactate normal
*urinary Cl > 20mmol/L
pH<7.45

The CSF is thought to be still alkaline for a while (lag behind serum) - thought to be at risk of postop apnoea due to this.
(2009-B) There is a 70-year-old female on your emergency list for an urgent laparotomy. She was involved in a motor vehicle accident this morning and sustained multiple trauma. Her medications include clopidogrel to cover the insertion of bare metal stents into her coronary arteries 2 months ago. 1. Describe the mechanism and duration of action of clopidogrel. (30%) 2. What are the major considerations for the perioperative period in view of the patient’s stent? (70%)
.
(2009-B) List the advantages and disadvantages of tight glycaemic control perioperatively in a diabetic patient on insulin. (30%) How would you manage the glycaemic control for such a patient having a minor procedure under general anaesthesia? (70%)
.
(2009-B) A 49-year-old woman has just arrived in the Recovery Room following a total abdominal hysterectomy under general anaesthesia. She is agitated and complaining of difficulty breathing. 1. List your differential diagnoses. (40%) 2. How would you determine if this was caused by residual neuromuscular blockade? (40%) 3. What is the role of sugammadex in the treatment of residual neuromuscular blockade? (20%)
.
(2009-B) A woman who is 10 weeks pregnant presents to the Emergency Department with a closed tibial shaft fracture.
1. Classify the drugs used in pain management according to their safety to use at this stage of pregnancy. (40%) 2. What are the options available for perioperative pain management for this patient? (30%) 3. What would you recommend? Justify your choice. (30%)
already answered
(2009-B) Identify the structures labeled A to H on this normal chest X-ray. (40%) Describe the arterial blood supply and venous drainage of the myocardium. (60%)
.
.
(2009-B) A 27 year old male presents with a glioblastoma for a craniotomy. As part of your anaesthetic technique, you decide to use a remifentanil infusion.

1. Discuss the characteristics of remifentanil with respect to its use as an infusion. (50%) 2. What are the advantages and disadvantages of using effect site calculations to guide remifentanil infusions? (50%)
.
(2009-B) 1. Outline the factors that determine oxygen delivery to the tissues. (30%) 2. How might you increase the oxygen delivery to the tissues in an anaesthetised patient. (40%) 3. How does a hyperbaric chamber influence oxygen delivery to the tissues? (30%)
.
(2009-B) 1. What are the indications for prophylaxis against perioperative bacterial endocarditis? (50%) 2. Justify your choice of antibiotics. (50%)
.
(2009-B) 1. Describe the differences between biphasic and monophasic manual external cardiac defibrillators. (50%) 2. What is the “synchronize” button for? When would you use it? (20%) 3. List the potential hazards of defibrillation. (30%)
a)
Monophasic
- old, no longer being manufactured
- unidirectional polarity of waveform, oten a "dampened sinusoidal" waveform
- energy 200-360 J required

Biphasic
- initial wave one direction, then reverses polarity
- allows defibrillation at lower energies (less risk Myocardial dysfunction and burns)
- more effective at TERMINATING VF ("defibrillation", for > 5 secs), but NO improved outcome in ROSC, or longer term outcomes
- 1st shock efficacy of 90%

b)
Synchronise button is used in cardioversion (the reversion of a perfusing rhythm to sinus rhythm). Needs to be synchronised so that the shock is timed with the QRS complex and does not fall on a T wave (could cause VF).
c)
Hazards to Patient
- burns (esp w GTN patches)
- myocardial stunning
- worsening of arrythmias (ie, VF)
- failure

Staff
- inducing arrhythmias
- burns
- fires (oxygen)
(2009-B) What are the key objectives of ethical review of a research project?
.
(2009-B) You are asked to give a practical tutorial on paediatric airway management to Emergency Department registrars at a large hospital. What are the important aspects of paediatric airway management that you would present to them?
- Anatomical changes
- Physiological differences
- equipment and technical differences
(2009-B) You see a 28-year-old woman at the pre-admission clinic who is 32 weeks pregnant. She weighs 150kg and has gestational diabetes. She is hoping to have a normal vaginal delivery at term. 1. What are the issues you would discuss with her during the appointment? (50%) 2. What would you recommend for her management when she goes in to labour? (50%)
a)
Discussion and assessment of comorbidities
- DM, Cardiac disease? HT? Reflux? OSA?
GDM - potential for insulin mx, fasting during labour, need for c/section given ass w macrosomia, and brief discussion re: neonatal mx
- technical difficulties associated with weight (IV access, epidural/spinal, airway)
- discussion of early IV/Epidural for safety and higher potential for operative delivery

b)
- Mx in tertiary centre
- early IV/epidural if consenting
- fasting (clear fluids), prophylactic regular antacid (ranitidine)
- regular glucose monitoring, sliding scale insulin
- possible need for DVT (definitely after c/section)
(2009-B) A patient has smoked 20 cigarettes a day for over 25 years. 1. What are the expected physiological changes that would occur in the first 3 months following cessation of smoking? Include a time frame for the changes you describe. (60%) 2. What are the clinical benefits, with regard to anaesthesia, of smoking cessation in this patient? (40%)
.
(2009-B) Describe the principles of cerebral protection in a patient with an isolated closed head injury.
see 82
(2010-A) a. List the complications associated with the use of limb tourniquets during surgery. (60%) b. How can these complications be minimised? (40%)
see previous 91.
(2010-A) a. List the hazards to the patient associated with the prone position under general anaesthesia. (60%) b. How can these hazards be minimised? (40%)
see 256.
(2010-A) A 20 year old female with a body mass index of 48 kg/m2 presents for an elective diagnostic laparoscopy for endometriosis. She has no other medical conditions. Describe the potential problems associated with anaesthetising this patient.
Monitoring difficulties
Equipment capabilities and limits
Airway management
Intravenous access
Difficulties in ventilation
Pharmacological considerations
Surgical difficulties
Pain management
Postoperative complications (eg DVT risk, respiratory complications)

Comments about how this question was answered:
Good answers were able to prioritise the important potential problems in the management of this patient, and include comments on the likelihood and likely severity of the problems Esoteric and excessive management suggestions for what is a common anaesthetic scenario subtracted from the value of some answers
Long lists without reference or relevance to the clinical case were not awarded many marks In general, the respiratory and airway problems, and the difficulties with pneumoperitoneum and Trendelenburg positioning were well handled. Some answers were difficult to read
(2010-A) a. Describe the pathophysiological changes associated with a haemoglobin of 75 g/L. (50%) b. Outline the patient factors that would indicate the need for a perioperative red blood cell transfusion in a patient with a haemoglobin of 75 g/L. (50%)
- changes depend on chronicity and compensation
- Anaemia results in less oxygen delivery to the tissues. The tissues will attempt to extract what they need anyway, which may result in a greater extraction ratio. Any lack of oxygen will result in anaerobic metabolism - resultant H+/lactate will cause r-shift of Hb-O2 diss curve and greater oxygen extraction from Hb
- will also cause vasodilation locally, and increased blood flow/CO
- chronically, there will be increased 2,3, DPG which will also R-shift the curve
- the lower hct/viscosity will lower TPR, and increase CO
- this resultant increase in CO/myocardial oxygen demand, may cause cardiac ischaemia
- other changes include redistriubtion of blood flow to vital organs, stimulation of EPO production, and new red cell formation

2)
Patient factors
- if signs if inadequate o2 delivery
* myocardial ischaemia
*decreased cerebral oxygenation / mentation / LOC
- ongoing blood loss
- The TRICC trial (in ICU) demonstrated a restrictive threshold of 70 showed no overall difference in mortality, especially in <55yo, and less unwell patients. Although no difference was also shown in patients with AMI/UA - the investigators concluded that these patients may benefit from higher threshold
- BTF recommends Hb > 100 in TBI patients
- transfusion may not be warranted if chronic anaemia, no signs of ischaemia (ie, chronic renal failure, sickle cell anaemia)
- religious beliefs may preclude transfusion also (JW).
(2010-A) A 26 year old woman with subclinical myotonic dystrophy presents to the high risk obstetric clinic. She is 25 weeks pregnant in her first pregnancy and otherwise well. She hopes for a normal vaginal delivery. Describe and justify your recommendations for the management of her analgesia for labour and the perioperative management of any potential operative delivery.
This patient has many potential systemic problems
o Respiratory impairment due to diaphragmatic weakness and expiratory muscle weakness
o Airway muscle weakness, increasing the risk of aspiration
o Cardiac abnormalities such as cardiomyopathy and arrhythmias
o Gastrointestinal abnormalities such as hypomotility, again increasing the risk of
aspiration

Should have appropriate preop Ix such as RFT's (restrictive defect?), and TTE (cardiomyopathy, MVP) / ECG (conduction defects).

o o o o
The difficulties with the use of muscle relaxants, especially suxamethonium
Increased aspiration risk Increased sensitivity to sedative/hypnotics
Myotonic responses from mechanical/electrical stimulation


Early epidural analgesia for labour is recommended given the risks with opioids, nitrous oxide and other sedatives
Monitoring during labour (oximetry, cardiotocography) should be described, and in appropriate environment
An appreciation of the increased risk of bleeding post partum in this patient, and how to plan for this (eg additional IV access, ensuring cross matching has been performed etc)
Avoiding complications relating to the management during Caesarean section, in particular:
Regional technique ideal, avoiding risks of aspiration, intubation with muscle relaxants (certainly avoid sux)

A postoperative plan for analgesia that avoids sedatives, and for postoperative monitoring eg in a
high dependency area

Comments about how this question was answered:
Good candidates mentioned other aspects of myotonic dystrophy, such as insulin resistance, hypothyroidism, adrenal insufficiency and mitral valve prolapse
Good candidates were also able to balance the potential complications from both regional and general anaesthesia techniques for Caesarean section
Poor answers gave a brief description of a management plan with little or no justification included.
(2010-A) A 40 year old man with hypertrophic obstructive cardiomyopathy (HOCM) presents for elective laparoscopic cholecystectomy. a. Describe the principles of intraoperative haemodynamic management for this patient. (40%) b. How would you manage hypotension post induction of general anaesthesia in this patient? (60%)
HOCM is an autosomal dominant condition characterised by risk of sudden death (Vent arrythmia) and LVOT obstruction. Most commonly there is abnormal relaxation and diastolic dysfunction, rather than LVOT obstruction (dyspnoea, syncope, angina).

a)
Aims of haemodynamic management are to avoid LVOT obstruction (especially systolic anterior motion of the mitral valve). LVOT occurs when preload or SVR is reduced, or when contractility or HR is increased.
- maintian preload (fluids, care with pneumoperitoneum pressures), maintain SR (atrial contribution important), maintain/reduce contractility (decreases dynamic LVOT obstruction), reduce HR/sympathetic drive with beta-blockers (avoid from pain and hypercarbia), maintain higher afterload (to reduce LVOT obs).
- mx would depend on
* pt severity/sx, treatment (ie, beta-blockade, AICD and mx of this)
*need recent Ix including TTE/ECG/PPM check

b)
Hypotension may be from cardiac/non-cardiac causes, and also mild or severe.
Mx would depend on severity, and would include simultaneous dx/rx.
- If severe,
need to inform surgeon/OT, call for help
- A, B, C as per ALS approach
- If less severe
- potential for LVOT obstruction, but mx much the same intially
- fluid bolus (preload), vasopressor (afterload). Need to check HR/stimulation - consider opioid +/- beta-blocker. Release pneumoperitneum!
- any rhythm disturbance must be treated.
- DDX should include myocardial ischaemia, anaphylaxis, drug error, etc.
(2010-A) A 43 year old female with a Grade 1 subarachnoid haemorrhage is scheduled for coiling of her middle cerebral artery in the radiology suite. Discuss the important issues to consider when providing anaesthesia for this patient
A grade 1 SAH has a relatively good prognosis, and means the woman has GCS 15 (and potentially normal ICP). It is less likely she has complications arising from her SAH.

Issues
Preop
- Assess Neurology
- Assess for complications (rebleed, vasospasm / nimodipine started?, hydrocephalus, seizures, cvs/resp cx, electrolyte disturbance
- usual anaestethic ax
- particularly focusing on BP
- Preparation of the environment (remote location, familiarise self, need for adequate MADE).
- Hx/Ex/Ix and consultation as needed.

Intraop
- Aims of the anaesthetic are to maintain MAP/CPP, prevent rapid changes in transmural pressure on aneurysm (esp induction), and prevent cerebral ischaemia.
- Closed cranium procedure - potential risk of bleeding is catastrophic
- non-stimulating procedure, likely to need vasopressors to maintain BP
- Adequate monitoring - usual monitoring, + IABP pre-induction
- premed if anxious, but otherwise avoid.
- Use Remi/Des, aim for HD stable induction/maintenance/emergence, and rapid neurological recovery/ax, particularly on laryngoscopy when aneurysm not yet secured
- maintain normothermia, normoglycaemia, normocarbia
- may need heparin +/- protamine
- prepare for rupture (large IV, plan)
- antibiotics, seizure prophylaxis, and nimodipine for prevention of vasospasm (may need CVC)

Postop
- likely to extubate unless problems, so rapid emergence and ax
- may need CT afterwards
- adequate filling
- nimodipine
- postop Neuro HDU

Issues with Remote site (4 P's)
- Place/Patient/Procedure/Postop
Place
- familiarity
- lines/tubes pulled out by angio equipment
- access to patient poor
- away from help / main OT

Patient
- as above

Procedure
- non-stimulating, needs to be STILL (GA)
- closed cranium, bleed could be catastrophic
- use of heparin
- risk of bleeding/arterial damage

Postop
- neuro HDU
(2010-A) a. Describe the anatomy of the eye relevant to a sub-Tenon’s eye block. (40%) b. Discuss the potential advantages and disadvantages of this technique for providing regional anaesthesia for eye surgery. (60%)
(2010-A) a. Describe the anatomy of the eye relevant to a sub-Tenon’s eye block. (40%) b. Discuss the potential advantages and disadvantages of this technique for providing regional anaesthesia for eye surgery. (60%)
.
Anatomy
- eye is a spherical object, usually < 25mm axial length, sitting in the bony orbit
- anterioer eye has cornea, conjunctiva (covering the sclera)
- the sclera is the "white of the eye", and contains the ocular contents
- tenon's capsule is the fascial external layer of the eye, extending from the corneoscleral junction anteriorly, to the optic nerve sheath posteriorly, and is external to the sclera (and internal to the conjunctiva anteriorly).
- subtenon's space is a potential space between sclera and tenon's capsule, and this fuses posteriorly where the recti attach to the eyeball
- there are 6 ocular muscles, superior, inferior, medial and lateral recti, and superior/inferior oblique muscles (posteriorly)
- sensory supply to the eye is via the short and long ciliary nerves from the ciliary ganglion
- a subtenon's block fills the tenon's space with LA, which blocks these nerves, as well as the motor nerves supplying the ocular muscles directly (by spreading into the muscle sheath). Does not block orbicularis oculi muscle. Blocks LPS due to blocking fibres of the 3rd nerve and ciliary ganglion.
- performed commonly inferonasal quadrant to avoid attachments of the recti muscles

b)
Advantages
- lower risks of globe perforation, bleeding/retrobulbar haemorrhage compared to other eye blocks
- good/rapid anaesthesia due to blocking direct motor fibres of recti muscles
- good analgesia
- easy to "top-up"
- relatively smaller increase IOP
- simple technique to learn and perform

Disadvantages
- chemosis and conjunctival haemorrhage common and unsightly
- incomplete akinesia may occur due to inferonasal quadrant being blocked 1st and inadequate spread to others (also no block OO)
- contrainidicated in previous retinal/vitreous surgery with scleral buckle (likely to cause globe perforation)
- other disadvantages common to regional
*can;t lie flat, only eye, tremor/movement disorder, cognitive impairment
- need for surgical technique and specialised equipment
(2010-A) a. Describe the factors that influence emergence delirium in children. (50%) b. How would you manage emergence delirium in a 3 year old child having had myringotomy tubes inserted under general anaesthesia? (50%)
Emergence Delerium is a phenomenon occurring in young children post-anaesthesia which is characterised by inconsolability, disorientation, non-purposeful behaviour, and is a diagnosis of exclusion.

a)
Patient Fx
- Age 2-5
- Past Hx ED
- distress on induction
- less social / more impulsive personality

Surgical Fx
- ENT / Head surgery
- MRI?

Anaesthetic Fx
- modern volatiles (rapid offset?) versus older or IV propofol
- midazolam pre-med is controversial (some have shown decreased, some increased, perhaps due to implicit recall)
- decreased by intraop fentanyl, clonidine, propofol pre-emergence

Parental presence on emergence is a possible protective fx.

b)
Assess the patient - need to ensure it truly is ED, and not pain, hypoxia, etc, as well as ensure safety of child (ie, not pulling out IV's, oxygen mask, etc)

Non-pharmacological Rx
- self-limiting usually
- parental presence
- quiet/dark area
- parental reassurance

Pharm Rx
- fentanyl 1mcg/kg
- clonidine
- small dose propofol
- midazolam has been proposed too, but examiners report did not like this

Adequate explanation is required to parent.
(2010-A) Describe the pathophysiological effects of an inhalational injury following a house fire. (60%) b. What implications would this have for anaesthesia one week after the injury? (40%)
a) An inhalational injury is suggested by Hx/Ex (and later Ex).
- explosion or fire in enclosed space
-LOC after a fire (CO poisoning)
- burning of toxic chemicals (ie, furniture)
Ex - facial burns, lips/tongue swelling, singed nasal hairs/eyebrows, voice changes, carbonaceous sputum

The upper airway usually protects lower airway from heat (efficient heat exchanger), so inhalational injury means significant burn or enclosed space
results in oedema, sloughing of mucosa, potential bronchiolar plugging, inflammatory cascade
resulting in atelectasis, ARDS, SIRS with poor gas exchange, need for ventilation and potentially need for vasopressors/fluids.
CO poisoning - >20% may be significant, as displaces oxygen from Hb and causes uncoupling of cytochrome oxidase in mitochondria - leading to poor oxygen delivery and utilisation in tissues, and anaerobic metabolism
Cyanide poisoning - again, leads to uncoupling of oxidative phosphorylation, with lactic acidosis

b)
Anaesthesia will be affected by the A, B, C.
Airway likely to be secured, but oedematous, need to ensure ETT does not come out. Difficult to secure if facial burns, and would be difficult to BMV due to oropharyngeal oedema and greasy skin.
Ventilation may be BAD. ARDS picture likely with high FiO2 requirements, and difficult ventilation pressures. Need to maintain lung protective ventilation strategy with permissive hypercapnia if necessary
C - may have significant SIRS/sepsis, with vasopressor support. Other burns needing debridement may lead to significant blood loss, so good IV required, and transfusion likely.
(2010-A) A 34 year old, opioid-dependant woman is complaining of severe pain on the day after a first metatarsal osteotomy. The nurses are concerned she is drug-seeking. a. How would you assess this patient? (60%) b. Outline your pain management plan. (40%)
a) Ax is Hx/Ex/ ?Ix.
Hx
Pain Hx - location, radiation, quality, severity (rest/dynamic), relieving/aggravating fx
Symptoms suggesting neuropathic pain (paroxysmal, burning/shock/parasthesia/dysasthesia, etc)
Hx of opioid use, other drug use - perhaps is not receiving proper dose or adequate analgesia
Social Fx / Psychological Fx
Hx from file / drug chart / anaesthetic chart (ie, underdosed?)
Hx form nurses (ie, absenteeism, intoxicated after friends visiting, difficult behaviour

Ex
Wound - signs of infection/inflammation
signs of withdrawal? Signs of intoxication?
Colour changes, sweating, swelling
distribution of pain? dermatomal/peripheral nerve?
hyperalgesia/allodynia/hypoasthesia

b)
Plan should involved discussion with patient/nursing staff, and potentially long term opioid prescriber (especially if legally need to).
Pain Mx in the opioid tolerant patient aims to
- prevent withdrawal
- provide adequate analgesia
- manage tolerance / OIH
- manage addiction (if present)

- After Ax, would try to maximise analgesia, maximising non-opioid multimodal techniques
*clonidine, ketamine, paracetamol/NSAIDS, tramadol, GBP
*titrate opioids - ensuring usual dose at least is prescribed, providing realistic doses either orally or by PCA (this may be avoided if patient can tolerate oral medications, and there is concern about opioid use in this situation).
There should be a discharge plan also
- need to reduce medications to a realistic outpatient level
- ideally the patient goes home on what they were admitted on
- communication with the GP, especially if change in opioid dosage
- referral to chronic pain clinic if thought necessary or D&A services.
(2010-A) a. Describe the aims of a quality assurance program. (40%) b. Outline the steps you would take to set up a quality assurance program for your anaesthesia department.
Quality assurance is defined as "an organised process that assesses and evaluates health services to improve practice or quality of care".

The steps involve
Planning, IMplementation, Review and Setting Standards.

- Planning
careful design, definition of topic to be evaluated, data collection, methods to analyse data
- Implementation
collection of data, reviewing results, determining action required (quality of care, deficiencies identified, implementing changes)
- Review
closing the loop, and demonstrating how the QA program has impacted
- Setting Standards
writing improvements into official guidelines or protocols
(2010-A) a. What do you understand by the term “Universal Precautions”? (40%) b. Describe how you apply these precautions in your daily anaesthesia practice. (60%)
Universal precautions = precautions taken to minimise risk of transmission of infectious agents from patient-HCW or vice-versa (particularly blood-borne agents or bodily fluids).

Daily Practice
- hand hygiene
- use of gloves, masks and eye goggles (esp for invasive procedures)
- single use items
- disposal of single use drug ampoules
- appropriate decontamination, disinfection, and sterilisation
- sharps dispoal
- waste disposal in yellow bins
- vaccination of staff
(2010-A) A 58 year old man presents for tonsillectomy for a tonsillar tumour. He has a 2 year history of intermittent palpitations. His electrocardiogram at diagnosis shows the following a. What is the diagnosis? Describe the electrocardiographic changes that support your diagnosis. (30%) Following the administration of neostigmine and atropine for reversal of neuromuscular blockade, you see the following rhythm on your monitor. b. What is this rhythm? How would you manage this situation? (70%)
.
(2010-A) What is the physiological basis of preoxygenation? (50%) b. Describe your method of preoxygenation including how you assess its adequacy. (50%)
a) Preoxygenation involves the administration of oxygen to a patient prior to induction, in order to increase their oxygen stores and delay any desaturation in event of airway difficulties.

FRC is approx 30ml/kg, approx 2.1L in adult. This usually only contains 21% O2. If closer to 100% (not achievable), then these is a higher store of oxygen to transfer into pulmonary blood before desaturation occurs. At an average O2 consumption of 250ml/min, an oxygen store of about 2L will be used up in ~ 8 mins vs < 2mins.

Preoxygenation involves replacing nitrogen in lungs with oxygen. This requires an appropriate alveolar ventilation to ensure wash-in of oxygen. Hyperventilation will slightly increase the oxygen store due to lower CO2 in the lungs, but will never reach 100% oxygen. ETO2 of approx 87% is probably maximum.
There will be slight increase in dissolved O2 in blood, which is clinically insignificant.

b) I apply 100% oxygen, by tight fitting face mask, and ask the patient to take deep breaths.
I monitor ETO2 and ETCO2 to ensure that seal is adeqaute, trace is normal, and that oxygen reaches mid 80% before induction
If no ET gas monitoring, I wait approx 3 mins. It is useful to confirm that patient has 100% oxygen on oximetry, but not always likely (ie, lung disease).
(2010-B) (a) What are the clinical consequences of hypothermia to 34⁰C in adults? (50%) (b) How can you manage body temperature in a multi-trauma patient? (50%)
The following were key components required to pass this question:
(a) Physiological changes include behavioural changes (eg adding clothing, moving to a warmer environment etc), vasoconstriction, shivering, impaired immune function and platelet dysfunction.
Clinical consequences include impaired wound healing, altered drug metabolism (eg prolonged muscle relaxation), possible cerebral protection in certain circumstances, increased sympathetic tone with hypertension and tachycardia and the consequences of shivering (eg on metabolic rate and myocardial oxygen demands, difficulties with monitoring, haemodynamic instability and patient discomfort) particularly in the postoperative period.
(b) Measures to limit heat loss include preoperative “prewarming”, increasing the operating room temperature, forced air warming blankets, warming fluids, wrapping/covering body parts, limiting wound evaporative losses, heat and moisture exchangers (HMEs) or humidification of the breathing circuit
Comments about how this question was answered:
In general most candidates were able to provide a list of consequences and describe the prevention and management of hypothermia. Additional marks were available in part (a) to those who showed an understanding of the relative importance of the clinical consequences mentioned, and in part (b) if the relative efficacy of the intervention was discussed.
The question was divided into two sections each worth 50%. Candidates generally spent longer answering the first section of the question related to the clinical consequences of hypothermia, and little time discussing the management of a patient with multi trauma. They therefore limited their ability to gain full marks.


There was a good understanding shown of the general effects of hypothermia, but little effort in separating those effects of mild (34 degrees) as opposed to more severe hypothermia. Physiological changes (such as a left shift in the haemoglobin-oxygen dissociation curve) gained marks if they could be linked to clinical consequences.
(2010-B) (a) Describe the arterial blood supply of the spinal cord. (50%) (b) Why is spinal cord function at risk during open repair of a thoracic aortic aneurysm and what measures are available to reduce this risk? (50%)
Arterial Supply
- Ant Spinal Artery (formed via 2 branches either side from vertebral arteries)
- runs in anterior median Fissure
- supplies 2/3-75% of the anterior spinal cord (Millers, A&A, Examiners Report WRONG)
- serially reinforced by spinal bra
Arterial Supply
- Ant Spinal Artery (formed via 2 branches either side from vertebral arteries)
- runs in anterior median Fissure
- supplies 2/3-75% of the anterior spinal cord (Millers, A&A, Examiners Report WRONG)
- serially reinforced by spinal branches of vertabral, deep cervical, intercostal/radicular branches, lumbar
- main branch (radicular) is the Artery of Adamkiewicz, usually arises T10-L1 on L but variable
*probably supplies much of the lower 2/3 of the spinal cord

Posterior Spinal Arteries
- 2-4, arise from PICA's
- supply the posterior columns (25%-1/3 posterior cord)
- reinforced by similar segmental branches, and AoAkwz

There is no anastmaoses between posterior/anterior blood supply, so thrombosis can cause infarction of large segments of cord (ie, ASA syndrome).
(2010-B) A 45-year-old man with a longstanding history of alcoholism is booked for upper gastrointestinal endoscopy and banding of oesophageal varices following an episode of haematemesis. (a) How is the severity of this patient‟s liver disease assessed? (50%) (b) How do these findings influence your evaluation of this patient‟s perioperative risk? (50%)
a)
Assessment should be by Hx/Ex/Ix and consultation with his gastroenterologist.
Hx
- previous variceal bleeding/banding, diagnosis of portal hypertension, bruising/coagulopathy, treatment of ascites, encephalopathy
- cardiac disease, cardiomyopathy
- immunosuppression, infection, anaemia, thrombocytopenia

Ex
- severity of jaundice, hepatosplenomegaly
- signs of portal hypertension such as ascites, leg oedema, abdominal wall veins
- signs of coagulopathy such as bruising
- signs of hepatic encephalopathy
- cyanosis/o2 sats (hepatopulmonary syndrome)
- signs of PHT/RHF (portopulmonary syndrome)

Ix
- Hb (blood loss), thrombocytopenia, coags
- liver function, esp INR, albumin, BR
- Any TTE? (cardiomyopathy, PHT/RHF)

b)
These findings can be used to estimate periop morbidity/mortality, and guide management (such as HDU care). A risk score such as Child-Pugh's score has been validated for periop risk
- Albumin
- INR
- BR
- PResence of ascites
- presence of encephalopathy

Higher scores indicate higher risk of periop death/morbidity, and may warrant HDU care - however, this is a minor procedure, buyt with potential for catastrophic bleeding.

Emergency surgery for ongoing bleeding would increase risk, as would risk of aspiration, etc be higher.

Presence of other co-morbidities such as respiratory disease, cardiomyopathy, or renal failure also shows a higher risk
(2010-B) A 68-year-old man in hospital awaiting definitive surgery for a supraglottic squamous cell carcinoma of the larynx has worsening stridor at rest. (a) How might his symptoms be improved in the preoperative period? (30%) (b) Describe your evaluation of his airway and how this will influence your intraoperative airway management plan. (70%)
ex report
(2010-B) How would you critically appraise a paper published in a journal?
Critical appraisal is a systematic approach to assessing the strengths/weaknesses, as well as usefulness and validity of research findings. (ie, a paper may be done very well - but if not of much value to clinical practice then not so great).

Factors
- Type of article (ie, editorial, case report, RCT, meta-analysis)
- presentation
- methodology
*hypothesis
*study design, statistics, power calculation, ethics, appropriate measures to reduce bias and confounding
*clinical importance
*positive/negative study, and strength of answer (in terms of statistical and clinical significance)
*conflicts of interest (ie, drug company involvement)
*respectable journal?
(2010-B) (a) Describe the common classification code for permanent pacemakers. (30%) (b) Outline the principles involved in the perioperative management of patients with a permanent pacemaker.
a) PPM are a class of Cardiac Implantable Electronic Device (along with Internal Cardiac Defibrillators).

There is a 5-letter code.
1 - chamber paced (A, V, D)
2 - Chamber sensed (A, V, D)
3 - Response to sensing (Inhibit, Trigger, Dual, O)
4 - Programmability & Rate modulation (O, P, M, C, R)
5 - Anti-tachycardia function (O, Pacing, Shock, Dual)

b)
Preop
- Ax particularly focusing on Pt's PPM and Cardiorespiratory state
*PPM
- indication for insertion (symptomatic bradycardia, arrythmias, CCF or primary prevention of VF = ICD), mode, PPM-DEPENDENT?!, recent check (within 12/12) for interrogation and battery life, response to magnet
- Hx and Ex focusing on cardiac status, stability of disease, cardiac ax as per AHA guidelines, any sx of PPM dysfunction?
- Need ECG and electrolytes at bare minimum

Intraop
- Need appropriate monitoring (at least plethysmography, maybe arterial line for evidence of mechanical capture)
- may not have time for reprogramming, latest CIED guidelines suggest that reprogramming to asynchronous mode only necessary for PPM-dependent patients with surgery above umbilicus (and use of monopolar diathermy).
- Defib functions should be turned off if present (if above diaphragm?)
- patients need defib pads applied if defib turned off, otherwise at least have defib available
- care with use of sux and rate modulation
- care with use of CVC's if recently inserted PPM
- If not PPM dependent, a magnet just needs to be available. surgeon should use short bursts of diathermy, and bipolar where avail.
- ECG monitoring should be set to PPM mode.

Postop
- if magnet or reprogramming used, needs reprogramming afterwards
- if AICD function turned off, needs to be monitored until this has occurred
(2010-B) How and why is cardiopulmonary resuscitation modified for the pregnant patient at term compared with the non-pregnant patient?
A
- potentially difficult airway
- risk of aspiration - ETT earlier
- earlier risk of desaturation

B
- lower FRC = rapid desaturation, and potential difficult BMV due to gravid uterus
- same ventilation ratio

C
- aortocaval compression makes CPR difficult, need L lateral tilt
- compressions higher up chest
- pads better AP
- may need perimortem C/S, ideally within 5 mins, which results in improvement in maternal haemodynamics
- same CPR ratio, drug doses, defib energy

Major causes of cardiac arrest in pregnancy (Blue Book 2009)
- VTE, AFE, Haemorrhage, hypertensive disorder, sepsis, trauma, cardiac disease, iatrogenic (medication, anaphylaxis, anaesthetic).
(2010-B) (a) List the predisposing factors for pain persisting for more than three months postoperatively. (50%) (b) Outline the interventions that have been demonstrated to be efficacious in the prevention of persistent postoperative pain. (50%)
a) see pain cards above.
risk factors can be An/Surg/Pt fx or pre/intra/postop.

Pt
- catastrophising, anxiety/depression, preoperative severe pain (>1/12), preop chemo/radiothx, female, young age

Surgical
- nerve damage (ie, amputation)
- specific surgeries, including amputation, thoracotomy, mastectomy, cholecystectomy, hernia repair
- minimally invasive techniques may reduce pain

Anaes
- use of pre-emptive analgesia (epidurals, ketamine?)
- use of preventative analgesia (epidurals/spinals, gabapentin, ketamine)
- regional techniques with LA (breast surgery = paravertebral or EMLA cream, or spinal for c/section)
- adminstration of Vit C for prevention of CRPS in Colles fractures
- good postoperative pain control

b)
"Pre-emptive" (ie, given before incision is more effective than after)
- epidural, LA, NSAIDS

"Preventative" (ie, doesn;t matter when given, but lasts longer than pharmocokinetics suggest)
**note , above are definitions only, they are not part of question acc to Ex. report.

- optimise pre and postop pain mx
- manage pt's psychosocial issues
- Regional and systemic techniques

- Regional
- epidural (reduces chronic post-thoracotomy pain, and SEVERE PLP after limb amputation)
- spinal (reduces chronic pain after c/s and hysterectomy)
- wound infiltration (craniotomy)
- paravertebral/EMLA (breast)

Drugs
- GBPoids
* reduce post-mastectomy pain but not PLP
*reduce post hysterectomy pain
- ketamine
reduces PLP if give before incision and for 72 hrs
- LA?

- ketamine, continuous periop epidural
(2010-B) An adult patient who was intubated for tonsillectomy is noted to have an upper central incisor tooth missing in the Recovery Room after extubation. (a) List the predisposing factors for perioperative dental damage. (50%) (b) What is your management of this situation? (50%)
.
(2010-B) A 78-year-old female presents for fixation of a displaced femoral fracture. She has longstanding mitral regurgitation and is known to have a mean pulmonary artery pressure of 60mmHg. She reports orthopnoea but is not short of breath at rest. (a) What are the issues of concern in your preoperative assessment? (50%) (b) How would you manage pulmonary vascular resistance perioperatively? (50%)
a)
Issues of concern relate to
Patient
- severe PHT and likely at least mod-severe MR, given secondary PHT. High change of perioperative morbidity and mortality because of this. Needs consultation with cardiology and ICU
*need to ascertain functional capacity, and any features of LVF or RVF, but realising that this is emergency surgery, so no time for optimisation
*need to discuss with patient the issues, limitations on treatment, and also discuss ICU (potentially)
*need to discuss possible aetiology of fall (ie, is it cardiac-related syncope)
*femoral fracture may result in blood loss, as well in complications of DVT/PE, etc, so needs to be fixed
*analgesia, may effect PHT.

b)
Aims are to minimise increases in PVR, and maintain RV perfusion.

Preop
- minimise opioids, consider regional block for analgesia preop (to avoid hypercarbia)
- supplemental oxygen
- treatment of LVF

Intraop
- regional? Spinal may drop SVR too much, and cause poor RV perfusion, and R-L shunting, so careful!! also no way of controlling CO2
- GA, ETT, controlled ventilation, minimise ventilation pressures
- need monitoring (A-line, CVC?)
- use balanced technique, short-acting opioids
- avoid nitrous, ketamine, sympathetic stimulation
- multimodal analgesia, regional block
- vasopressors to ensure RV perfusion
- pulmonary vasodilators sich as NO or nebulised iloprost should be considered if available, milrinone for inotropy and some pulmonary vasodilation.
- avoid desflurane, Iso/sevo probably best.
- TOE could be considered for cardiac function

Postop
- minimise opioids with multimodal approach as discussed
- usually postop care
- ?HDU - would ned to discuss the realism of this preoperatively with patient/ICU
(2010-B) What are the advantages and disadvantages of general versus local anaesthesia for carotid endarterectomy?
GA - Advantages
- airway secure
- haemodynamics easily controlled with vasoactive medications and opioids, muscle relaxation provides good surgical conditions
- allays patient anxiety about being awake
- trials which showed benefit from CEA (ie, NASCET) almost exclusively used GA

Disadvantages
- potential for HD instability from induction agents
- cardiac ischaemia possible, particualrly on induction/emergence
- risks of GA including dental damage, aspiration, PONV, etc
- either no or selective use of cerebral monitoring required, with no one technique thought to be of great benefit, therefore risk of neurological complications

LA
Advantages
- main advantage is the "gold standard" CNS monitoring, as able to assess patient's neurological function continuously, to decide whether need for shunt insertion after clamp
- reduces postop bleeding (GALA trial), and need for shunt insertion (has its own risks).
- avoids HD lability around induction/emergence
- avoids other general prbs with GA
- if postop bleeding, block may still be working, avoid difficult intubation?

Disadvantages
- difficult access to airway in emergency
- anxiety for patients
- ?more HT postop
- problems with the block
*need for top-up by surgeon often
*inadvertent IV, IT, epidural injection with possible consequences
*phrenic N, RLN palsies, HOrner's syndrome
*No demonstration that is any better in terms of mortality/MI/stroke outcome (GALA).
(2010-B) Outline the principles of an initial management plan for diabetic ketoacidosis, having regard to the physiological derangements involved.
.
(2010-B) You are on the interview panel appointing new Assistants for the Anaesthetist. What are the educational requirements and the practical responsibilities expected of the applicants?
Educational requirements
- must have completed a course, involving practical experience and assessments/assignments
- 3yr duration if no previous hospital experience, 1 yr for a RN1
- must have regular CME
- course must involve Basic Sciences, Anaesthesia (equipment, safety, techniques, pain, emergency care), and Management issues.

Practical Responsibilities
- assistance to anaesthetist (immediately available for preparation/induction/emergence, at short notice throughout case - NOT exclusively for case as stated in examiner's report!)
- responsible for preparation/application of monitoring and other equipment*
- responsible for checking/maintenance of equipment*
- cleaning, decontamination, sterilisation*
- restocking*
- to work within team in appropriate manner*


*Not stated in ANZCA professional document.
(2011-A) Question 1 (a) What is the role of dexamethasone in the management of postoperative nausea and vomiting? (70%) (b) What are the potential problems associated with its use? (30%)
a) Used for PROPHYLAXIS, usually in dose 4-8mg IV.
- given at induction
- decreases prostaglandin formation, as well as TNF, cytokines, and potentially 5HT formation
- shown to decrease PONV by 25%, better ABSOLUTE risk reduction if given to patients with higher risk of PONV (female, non-smoker, hx PONV, postoperative opioids)
- similar efficacy to ondansetron 4mg, droperidol 1.25mg, but works in additive manner to these agents
- usually given as single perioperative dose, and not really studied adequately in TREATMENT setting postop

b) Of note, no adverse effects have been identified from a single periop dose (SAMBA 2007). Possible problems may include
- increased BSL with associated problems
- potential immunosuppression and effect on wound healing
- peptic ulceration
- pain on injection to awake patient (particularly perineal burning/itching sensation)
(2011-A) Question 2 A patient with known idiopathic pulmonary fibrosis (fibrosing alveolitis) presents for an open right hemicolectomy. (a) What are the respiratory issues facing this patient with regard to their general anaesthetic? (70%) (b) Explain your intraoperative ventilation strategy. (30%)
Ex report says An/Pt/Surg Fx but doesn't seem to answer Q....

a) Pulmonary Fibrosis a chronic restrictive lung disease, characterised by decreased lung volumes (particularly FVC, FEV1, with normal or high FEV1/FVC ratio).
Potential Resp Issues include:
Patient factors
- severity of lung disease? PFT's? home oxygen?
- treatments (incl immunosuppresants and steroids)
- existing infection? Other co-morbidities such as PHT
Surgical Fx
- large fluid shifts
- inflammatory response and risk of ALI
- risk of blood transfusion
Anaesthetic Fx
- aspiration (apparently pts at HIGH risk of reflux disease)
- other iatrogenic (CVC causing pneumothorax)
- risk of ALI with mechanical ventilation, barotrauma
- high FiO2 causing worsening of fibrosis? Only in prev bleomycin I think
- intubation/GA causing VQ mismatch, reduction in FRC, atelectasis
- poor sputum clearance/atelectasis postop caused by pain, surgery, residual drug effects (opioids, sedatives, NMBA's)
- risk of postop ventilation

b) Need "protective lung ventilation" to prevent ALI. This also tends to mimic their natural ventilation.
TV ~ 6ml/kg, plateau pressure <35cmH2o, increased resp rate to control CO2 (permissive hypercapnia may be necessary). PCV vs VCV.
Addition of PEEP to minimise atelectasis/VQ mismatch, improve lung compliance, titrated to sats
FiO2?
(2011-A) Question 3 Explain the professional attributes of an anaesthetist in specialist practice.
As per the ANZCA (CanMeds) attributes.
- Medical Expert, Communicator, Collaborator, Scholar, Manager, Professional, & Health Advocate.
as per Mod 2 notes
- Med Exp. striving for clinical excellence, patient safety, evidence based medicine, and to maintain technical skills and knowledge by adequate reflection & QA
- Communicator. Eff relationships with pts, families, colleagues, use ethical reasoning. Consent issues.
- Collaboration. Demonstrates good practice as part of MDT, flex leadership, consultation, delegation. Respect for others.
- Manager. patient care, OT room, department, personal issues. Allocation of health resources ethically.
- Health Adv. Maintains own health, and promotes health/safety of others (pts & colleagues).
- Scholar. commitment to ongoing learning, QA, evidence-based medicine. Teaching others.
- Professional. Respecting pt confidentiality, and adhering to ethical principles. Complies with appropriate guidelines (ANZCA & government, etc). Practices with integrity, honesty, compassion.
(2011-A) Question 4 Evaluate the use of human albumin in perioperative volume replacement.
- what is albumin, 4% & 20%, stays in intravascular space due to MW, and oncotic pressure means bigger increase in circulating volume for given dose. no x-matching. In normal saline.
- Benefits, cheap (for hospitals), less peripheral oedema, lower risk anaphylaxis (v other colloids), less probs with coagulation/renal/pruritis (v starches)
improved outcome v N/S in sepsis (SAFE)
- Cons. expensive to produce (government/community), problems with N/S (met acidosis), higher anaphylaxis (v crystalloid), equivalent to N/S re: morb/mortality. worse outcomes in TBI (SAFE).
(2011-A) Question 5 (a) How would you clinically assess a patient complaining of leg numbness the day after a spinal anaesthetic for an emergency caesarean section? (70%) (b) How would you manage the situation? (30%)
a) Ax re Hx/Ex/Ix. Need to differentiate between serious (ie, epidural haem/abscess) or other (Obs/Anaes/pre-existing Neuro disease).
- Obs Hx: labour length, baby positioning/size
- Anaes: Diff w spinal insertion, blood, pain/parasthesia
- Sx/SX, timing, associated sx (back pain, fever, motor weakness, targeted neuro exam.
- Peripheral N = Obstetric, Nerve root = Anaesth
b) Attend pt ASAP. Mx depends on findings. potentially urgent MRI, otherwise careful documentation findings, pot Neuro referral
- explanation to patient re: natural progression if neuropraxia
- ensure f/u, may need NCS
- notify m&m, insurer.
(2011-A) Question 6 A 60-year-old man is booked for plating of a fractured ankle. He arrests on induction. His ECG shows ventricular fibrillation. Outline the immediate management of his cardiac arrest with particular reference to current resuscitation guidelines.
2010 ILCOR guidelines
- emphasis on CIRCULATION. C, A, B
- Treatment is immediate defibrillation. However, there will be time to get the crash cart and defib pads on, so chest compressions should start immediately - emergency buzzer should be pressed!. Defib ASAP. 150-200j (bipolar) as per manufacturer's guideline. NO STACKED SHOCKS.
- immediately recommence CPR. Rate 30:2 breaths, at HR 100bpm.
- an advanced airway can be placed, when trained person available - as long as does not stop compressions (LMA or ETT).
- Rhythm should be assessed every 5 cycles (2 mins), and defib if appropriate, followed by immediate compressions again.
- Adrenaline 1mg IV/IO given after 2nd shock, and every 2nd cycle after that
- Amiodarone 300mg IV given after 3rd shock (refractory)
- Consider 5H/5T
H
*Hypothermia
*Hypovolemia
*Hypo/Hyperkalemia
*Hydrogen ion
*Hypoxia

T
*Tension Pneumothorax
*Tamponade
*Thrombembolism
Cardiac & Pulmonary
*Toxins (LA? consider intralipid).

Person doing chest compressions should be rotated every 2 mins to avoid fatigue and improve CPR quality.
(2011-A) Question 8 (a) Describe the anatomy, including surface landmarks, relevant to performing cricothyroidotomy. (50%) (b) What are the complications of this procedure? (50%)
a) Cricothyroidotomy is needle/surgical procedure through CT membrane, which lies between the thyroid cartilage superiorly, and cricoid cartilage (level of C6) inferiorly.
- Boundaries
*Anterior - skin and subcutaneous tissue anterior midline neck
*Pos
a) Cricothyroidotomy is needle/surgical procedure through CT membrane, which lies between the thyroid cartilage superiorly, and cricoid cartilage (level of C6) inferiorly.
- Boundaries
*Anterior - skin and subcutaneous tissue anterior midline neck
*Posterior - vocal cords superiorly
*Superior and inferior mentioned above
Also lateral borders are the same cartilages
-Inferiorly, the thyroid gland and isthmus, occasionally extends superlaterally.
- laterally - paired cricothyroid muscles, with superior laryngeal nerve supply (also superior thyroid arteries)
Also lateral are the RLN's, common carotids and IJV's. inferior thyroid arteries run with RLN's.

b)
- Bleeding, although membrane avascular, other vessels around!
- Neural damage, potentially resulting in VC palsy
- failure (!)
- Other structures, including oesophagus perforation, tracheal perforation, sub cut emphysema and false passages, barotrauma, subglottic stenosis
(2011-A) Question 9 You hear a cardiac murmur in a two-year-old child presenting for elective minor surgery. (a) What are the features of the murmur that would differentiate an innocent from a pathological murmur? (50%) (b) How would you evaluate this child’s fitness for anaesthesia from the cardiac perspective? (50%)
a) Examiners report did not really break into A and B.

BUT
Hx
- Birth Hx? Ex-prem?? Syndrome (Down's, CHARGE, VATER, etc)
- recurrent chest infections, tachypnoea, sweating
- failure to thrive, failure to "keep up","", syncope, squatting
- family Hx (ie, HOCM, sudden death)

Ex
- pulses 4 limbs (radiofemoral delay in coarctation)
- vitals, BP, sats!
- associated signs of HF (lung creps, ascites, hepatomgealy)
- Murmur Fx
* Timing - diastolic, pansystolic, late systolic bad. Continuous may be bad or good (venous hum)
*Loud, harsh murmurs, thrills, vs soft/musical/vibratory (innocent)
*Dynamic man's - HOCM louder on standing, venous hum disappears lying down. Pathological "rarely" varies with posture (CEACCP2011)

b) basically that above. Refer those with concerning hx/ex/ix prior to elective surgery, otherwise, should be able to continue.
(2011-A) Question 10 (a) What factors contribute to acute kidney injury in the perioperative period? (70%) (b) Outline the efficacy of perioperative strategies to reduce acute kidney injury. (30%)
AKI defined by the RIFLE criteria (Risk, Injury, Failure, Loss, End-stage) (creatinine increased by 1.5, 2, 3x).
a)
Patient Fx
- Age > 65, pre-exiting Renal failure, DM, HT, PVD, CCF, sepsis, trauma, medications (ACE I or nephrotoxins)

Anaesthetic Fx
- Hypovolemia, inadequate fluid resusc
- hypotension, maintenance of MAP, nephrotoxins (NSAID's, aminoglycosides)

Surgical Fx
- Cross-clamps decreasing perfusion or extracorporeal circulation (ie, AAA, or CPB)

b)
- Screen at risk patients, and take precautions
- avoid nephrotoxins
- maintain fluid volume and MAP
- No role for pharmacological therapy (except perhaps in NAC for contrast-induced nephropathy if ised) in periop period. Previously mannitol, frusemide (increasing urine flow and decreasing renal O2 demand) thought good, dopamine low dose no evidence. NaHCO3 limited evidence, may be due to fluid volume loading.
(2011-A) Question 11 (a) Describe the clinical pharmacology of codeine including an outline of its therapeutic use. (70%) (b) Describe the influence of pharmacogenetics on the variability of patient response to codeine. (30%)
a) usually given orally, but can be given IV. oral dose 30-60mg 4-6/24. morphine dose about 5% parenteral equivalent.
metabolised to morphine by liver, which is how it has its action. Morphine further metabolised and glucuronide by-products are renally excreted.
used for moderate pain (often combined with paracetamol), but also as anti-tussive and anti-diarrhoeal agent).
side effects inlude nausea/vomiting, constipation, and histamine release when given IV.

b) Metabolised by CYP2D6 enzyme in liver. polymorphism of this enzyme means some people metabolise fast or not really at all. About 10% caucasians do not metabolise codeine and therefore get no effect from it. There is potential for inhibition of this enzyme by other drug interactions which may also have an effect.
(2011-A) Question 12 (a) What are the complications associated with residual neuromuscular blockade? (30%) (b) Evaluate the methods available to assess residual neuromuscular blockade. (70%)
a)
Problems wit A, B, C
Airway - obstruction, aspiration due to poor swallowing/bulbar dysfunction.
Breathing - poor cough, respiratory failure, aspiration
Circulation - if well awake, hypertension/tachycardia due to distress at weakness, with possibility for resulting myocardial ischaemia

b)
Clinical
- eye opening, head lift, tongue depressor bite between teeth, vital capacity breath. Patient "twitchy"
- relatively poor sensitivity, will pick up moderate to profound block only.

Monitoring
- usually using nerve stimulator over ulnar nerve for adductor pollicis (but can use orbicularis oculi, posterior tibial, or peroneal nerve stimulation).
- Can be "tactile/visual" or "quantitative".
- use different modes of stimulation to assess (ie, single twitch, insensitive unless measured carefully in lab, double-burst stimulation - best for visual/tactile, but still only equivalent to a TOF ratio of 0.6, and TOF count and ratio - most sensitive if quantitatively measured).
- visual/tactile measurement is probably only able to pick up a TOF ratio of 0.4 or less (very insensitive).
- DBS may pick up TOF ratio (equivalent) of 0.6 (tactile)
- Quantitative measurement using accelerometry, or other equivalent is the gold standard, and can be done in the operating theatre these days (kinesiomyography).
- tactile stimulation and clinical assessment are good due to no need for specialised equipment, or measurement modules (expense). However, as seen above, they do not reliably detect significant levels of NMB.
(2011-A) Question 13 You are involved in the planning of a new Day Surgery Unit. (a) What systems would you put in place to reduce the likelihood of a power failure? (50%) (b) Outline a protocol for dealing with power failures. (50%)
a)
To reduce likelihood of critical power failure, there should be multiple back-up power supplies available.
- separate Mains connection (to avoid power failure if main street wires damaged, ie, by MVA into power pole)
- On-site back up generator (diesel?) for "essential power supply" (Red power points), may take up to 2 mins to kick in. may have several back-up generators depending on hospital size.
- UPS (battery supply). This is charged by usual mains supply, where AC is rectified and converted to DC, which supplies large battery stack, power is then inverted to AC supply. These are Blue powerpoints, and should only supply critical equipment that canot tolerate ANY disruption (ie, ICU ventilators), and only have limited supply.
- battery back-ups in certain machinery (ie, anaesthesia machines)
- Beige power points for cleaning equipment which may otherwise trip safety switches.

b)
* General
- detailed emergency manual on back-up supplies and lines of communication within hospital, and to outside engineers.
- should be a coordinator who is in charge of each section (ie, theatre).
- emergency torch, radios, batteries should be available
- all unnecessary equipment unplugged (to increase UPS supply or back-up)
- back-up means of getting into electronically locked rooms

*Theatre
- Accessible checklist for OT protocol
- (detailing battery life and equipment powered by machine)
- ?stop surgery?
- convert to spont vent technique if possible, may need to run battery powered infusion pumps, and use manual ventilation
- non-electrical monitoring should be available (ie, BP cuff, stethoscope, torch
(2011-A) Question 14 (a) Describe the abnormality on this electrocardiogram. (30%) (b) What are the implications of this abnormality for anaesthesia? (70%)
(2011-A) Question 14 (a) Describe the abnormality on this electrocardiogram. (30%) (b) What are the implications of this abnormality for anaesthesia? (70%)
a). The ECG apparently showed a prolonged QT interval.
This may be calculated, or may appear as prolonged, broad T waves, or with a bifid appearance. Prolonged QT can be caused by genetic abnormalities in cardiac ion channels (K or Na), or by drugs. T wave alternans also.

b)
Implications are that these patients are at risk of sudden death, or ventricular arrythmias (esp TdP). In particular, many drugs (in anaesthesia and elsewhere) can prolong the QT interval which increases the risk of arrythmias. These drugs include
- sevoflurane (case reports in children)
- ketamine
- Thiopentone (controversial, as argued that it does not prolong the transmural dispersion of polarisation, which appears to be main risk)
- droperidol, ondansetron
- neostigmine
- adrenaline, noradrenaline, ephedrine, dobutamine, isoprenaline (in BJA article, although elsewhere mentioned as a treatment !)
- apparently atropine/glyco/aramine all ok
avoid sux.
- hypothermia prolongs QT interval
- IPPV MAY prolong (valsalva maneouvres do).
- attenuation of sympathetic drive is required

These patients therefore need proper preop ax, and mx before elective surgery, potentially beta blockade or PPM/ICD, and periop Mg infusion.

LQT1 & 2 are both K channel defects, which cause problems under sympathetic stimulation (therefore beta-blockade). However, LQT3 is Na channel defect, and arrests often happen while pt sleeping, therefore pacing is required at highish back-up rate.
(2011-A) Question 15 (a) How would you identify a patient with autonomic neuropathy associated with diabetes? (50%) (b) What are the anaesthetic implications from a cardiovascular perspective? (50%)
a)
-Identification of AN by Hx/Ex/Ix.
- may have other "systems" affected.
Hx
- poor diabetic control
- hx of other diabetic complications (ie, peripheral neuropathy, eye disease, renal disease)
- known "heart disease" without chest pain
- orthostatic syncope/dizziness
- nausea/vomiting (gastroparesis), diarrhoea
- erectile dysfunction
- abnormal thermoregulation (hypothermia?)

Ex
- increased resting HR
- decreased HR varability in respiratory cycle (mean difference should be > 15bpm from insp to exp)
- valsalva ratio < 1.2 (longest R-R interval / shortest R-R interval)
- postural drop in systolic BP > 10mmhg
- evidence of peripheral neuropathy

Ix
- includes above (ECG)

b) Implications
- possible underlying heart disease without symptoms
- risk of CVS instability, impaired response to fluid shifts and intubation, arrythmias with ischaemia
- may need to increase level of monitoring (ie, IAL, 5-lead ECG)
- fluid load to reduce changes in BP
- use of vasopressors, etc as required
- may need higher level of monitoring depending on surgery
(2011-B) Question 1 Compare and contrast oxygen delivery by nasal prongs, simple facemask and Venturi mask.
Oxygen Delivery Sx
- fixed vs variable performance devices
- patient "dependent" or "independent"

Variability comes from a patient's respiratory parameters.
- If peak inspiratory flow rate is greater than oxygen flow, then fiO2 will be reduced
- to be a "fixed" performance device, need ~ 3-4 x MV as flow rate.

Nasal Prongs
- low flow, 1-4L/min (can have high flow nasal prongs acting as fixed performance device ~ ICU)
- higher fiO2 if nasal breathing, usually about 0.24-0.28
Advantages - comfort, allows eating/talking, no rebreathing
Disadvantages - low fiO2, higher flows cause irritation, mouth breathing no benefit

Simple FM
- low flow, variable air entrainment (fiO2 0.3-0.5)
- flow rates < 5l/min may result in rebreathing
- reservoir bag (NRB) allows up to 0.8-9 fiO2
Advantages - easy/cheap, oral/nasal breathing, highish fiO2 (esp with reservoir bag)
Disadvantages - less comfortable, removed for eating/talking, rebreathing

Venturi Masks
- exploit venturi effect to cause KNOWN entrainment of air through aperture to deliver known fiO2
- may entrain extra iar if peak inspirtory flow very high
Advantages - accurate fiO2, good for children, COAD/CO2 retainers, neurosurgical pts
Disad - mask not comfy, not high fiO2 (60%), need to change fitting for different fiO2 setting
(2011-B) Question 2 A new randomised controlled trial suggests therapy A is better than therapy B in the treatment of condition X. How would you evaluate this trial before changing your clinical practice?
A randomised control trial is a prospective trial, where 2 groups are studied, where the area of interest or intervention is (supposedly) the only difference between the 2.
Evaluation
- Conduct of trial
hypothesis? multicentre vs single centre? NUmber of patients, exclusion criteria / selection (applicability), randomisation, possible bias, blinding (ie, patient, observer, studier)
Statistics - appropriate data, statistical tests, p-values and confidence intervals, published sample size/power analysis
- clinically significant?
- is drug B the gold standard or should be compared to something else?
- adverse effects
(2011-B) Question 3 What are the benefits and limitations of red blood cell salvage? (50%) How would you justify its introduction into your institution? (50%)
see 102.

Justify by
- number of procedures that require transfusion > 10% of time, or anticipated to produce > 20% BV loss
- more it is used, the cheaper it becomes
- cochrane review has demonstrated reduced transfusion rate and cost savings in cardiac, major joint, some vascular, paed (scoliosis, cardiac) surgeries.
(2011-B) Question 5 Insertion of a central venous line may result in cardiac tamponade. a. How would you recognise this complication? (50%) b. How could you minimise the risk of this complication? (50%)
Cardiac tamponade
- Hx
dyspnoea, orthopnoea, PND
- Ex
hypoxia, tachypnoea, tachycardia, muffled HS
raised JVP, kussmaul's sign, pulsus paradoxus
LVF, leg oedema
- Ix
CXR (large globular heart, LVF)
Echo (pericardial effusion, RA collapse)
ECG (l
Cardiac tamponade
- Hx
dyspnoea, orthopnoea, PND
- Ex
hypoxia, tachypnoea, tachycardia, muffled HS
raised JVP, kussmaul's sign, pulsus paradoxus
LVF, leg oedema
- Ix
CXR (large globular heart, LVF)
Echo (pericardial effusion, RA collapse)
ECG (low voltages, electrical alternans)

Minimising risk
- care on insertion (wire)
- positioning of CVC (check on CXR)
*ideally above RA, level of RMB on CXR
* RIJ probably better as no angles (ie, LIJ often either abuts SVC wall or goes into RA)
- Mx of CVC
leave in for minimum time
surveillance

see zone B (for RIJ) and zone A (for LIJ - compromise).
(2011-B) Question 6 A 50-year-old patient presents for urgent craniotomy and decompression of a subdural haematoma. Two days ago he was well, but now has a Glasgow Coma Scale score of 11. He is combative and has pulled out his intravenous line. On inspection there are no obvious veins for cannulation. a. List the options available for induction and intubation. (30%) b. Describe and justify your preferred approach. (70%)
.
(2011-B) Question 7 Four hours after multi-level laminectomy with instrumentation, your patient complains of unilateral visual loss. a. What is your differential diagnosis? (40%) b. How can you minimise the risk of visual complications in the prone patient? (60%)
Key components of a response to this question included: Differential diagnosis considering pathology in or damage to
- - - - -
the globe and/or its parts the optic nerve vasculature (arteries or veins) the brain

- damage to globe from direct pressure
- central retinal artery occlusion (again from external compression most common)
- Ischaemic Optic neuropathy (posterior most common in spinal surgery), potentially caused by decreased arterial pressures, increased venous pressures, anaemia, increased IOP from positioning
- corneal abrasion
- cortical injuries
- hysterical illness (psychiatric)


Minimising the risk of visual complications:
- -
-
preoperative risk stratification
*patient Fx - atherosclerosis, hyperlipidemia, diabetes, elderly, smoker
*surgical/anaesthetic factors - prone, spinal, extensive duration, anaemia/blood loss, crystalloid resuscitation

*intraoperative positioning/surveillance (positioning particularly important to avoid direct pressure but also increased venous pressures)
*haemodynamic monitoring and acceptable limits
*fluid management including transfusion

*postoperative positioning, monitoring and surveillance, including rapid assessment by Opthalmology if suspected as poor prognosis otherwise
(2011-B) Question 8 A child with active upper respiratory tract infection presents for general anaesthesia. a. Outline the factors that increase the rate of adverse respiratory events during anaesthesia. (50%) b. How can you reduce the risk of an adverse event occurring? (50%)
Adverse resp events are ill-defined, but probably include, coughing, breath-holding, laryngospasm, bronchospasm, hypoxaemia, negative-pressure pulmonary oedema, and atelectasis/pneumonia.

a)
Patient Fx
- Age
<6yo, esp < 1yo
- Active URTI
purulent nasal discharge, moist cough / LRTI, wheeze, fever, malaise, looking unwell, parents smokers, parents confirming they have a "cold"
- Co-morbidities
Asthma, chronic lung disease, ex-prem, CF

Surgical Fx
- Airway, ENT, Eye surgery
- Major upper abdo, cardiac surgery

Anaesthetic Fx
- experience of anaesthetist
- airway instrumentation (ETT>LMA>FM)
- Propofol<Sevo<Des

b) Reduction of ARE's are by modifying factors above.
If the child looks UNWELL, or has signs of LRTI (or < 1), they should be postponed for at least 2 weeks. Reactivity probabaly occurs for up to 6/52, but postponing may not change outcome. This should also occur if parent is particularly worried.
- change anaesthetic technique if possible, prob least invasive airway possible, IV propofol ideal, no des
- IV for induction (at least for control of ARE)
- pretreatment with bronchodilators if possibly effective
- topical lignocaine also controversial
- avoid intubation/extubation in light planes of anaesthesia
- appropriate monitoring and observation

Should not postpone child who is otherwise well, as there is no evidence this improves outcome (As long as specialist there to manage potential ARE), and children have many URTI's in a year, so difficult to find a "window" to operate.
(2011-B) Question 9 You are called to anaesthetise a 70-year-old man with a perforated bowel for laparotomy, three days after colonoscopy. Outline the measures you will take to reduce the likelihood of this patient developing acute lung injury.
Key components of a response to this question included:

PREOPERATIVE -
- recognition of acute lung injury risk
- plan to reduce incidence of sepsis/ aspiration

INTRAOPERATIVE
- management of haemodynamics (fluids/use of blood products/inotropes, minimising fluid overload)
- ventilation strategy: mechanics, FiO2 (ie, lung protective ventilation, with TV 6ml/kg, Plateau pressure < 30, use of PEEP, low fiO2
- place of antibiotics/ steroids (none)

POSTOPERATIVE -
ventilation strategy
aspiration prophylaxis
(2011-B) Question 10 A 70-year-old patient wearing a transdermal buprenorphine slow release patch (Norspan®) (5μg/h) presents for knee arthroscopy. a. Describe the mechanism of action and pharmacokinetic profile of this patch. (50%) b. What are the implications for perioperative pain management? (50%)
Buprenorphine is a synthetic opioid with agonist activity at mu receptors (arguments over partial or full activity), and antagonist effects at kappa receptors (good for anti-hyperalgesic effects). It has high affinity for mu receptors, so may displace other full agonists from them (causing withdrawal symptoms).

It can be given IV, S/L (usually high dose maintenance therapy in addicts) or transdermally (low dose chronic pain patients).
Bioavailability is poor (approx 30% by Sl route), but close to 100% by TD route.
mainly hepatic excretion (faecal unchanged), to some active metabolite, but no dose adjustment needed in renal failure.

Other points
*patches come in 5, 10, and 20mg, corresponding to 5, 10 and 20 mcg/hr (high dose in Sl maintenance is 8-32mg daily)
*equivalent opioid dose is approx 10, 20, 30bd oxycodone, or 10, 20, 40 mg IV morphine in 24 hrs.
* there is a ceiling effect for analgesia and respiratory depression (safe!) - doses above 16mg may just increase elimination half-life (useful in daily dosing for drug addicts)
* analgesic effects not as long as anti-withdrawal effects (like methadone)
* respiratory depression can occur with IV form, but also in misue of patch, or adjuvant benzos

Implications
- need to determine opioid use (high or low), and operation pain amount (high or low).
- implications in low dose is not much, may be partially opioid-tolerant, and need to prevent withdrawal, but likely not enough to completely block other mu agonists
- principles are to continue patch, and provide a multi-modal approach to analgesia, including opioids as needed.
(for high dose, this is more complicated, and may need opioid conversion preop, d/w D&A, and multi-modal approach, as well as titration of potent opioid in HDU setting as risk of RD).
(2011-B) Question 11 a. Define quality assurance. (30%) b. How would you design and implement a Quality Improvement programme to assess patient satisfaction with the preoperative visit? (70%)
.
(2011-B) Question 12 a. Which peripheral nerve/s need to be blocked for complete analgesia following repair of a tibial plateau fracture? (30%) b. Describe your technique for blockade of these nerve/s (EXCLUDING central neuraxial blockade). (70%)
(2011-B) Question 12 a. Which peripheral nerve/s need to be blocked for complete analgesia following repair of a tibial plateau fracture? (30%) b. Describe your technique for blockade of these nerve/s (EXCLUDING central neuraxial blockade). (70%)
- likely femoral and sciatic nerve blocks.
need to block articular branches from femoral, saphenous, tibial and common peroneal nerves.
- Technique
consent
MADE, sedation
Femoral
- blokc at inguinal crease, midway point of inguinal ligament, lateral
- likely femoral and sciatic nerve blocks.
need to block articular branches from femoral, saphenous, tibial and common peroneal nerves.
- Technique
consent
MADE, sedation
Femoral
- blokc at inguinal crease, midway point of inguinal ligament, lateral to vessels, with ultrasound (NS shows patellar contraction)


Sciatic
- proximal?
- approx 6cm above popliteal fossa, lateral approach in this pt, ultrasound guided, look for block prior to nerve bifircation, needle from lateral through biceps femoris, and NS ankle plantarflexion/dorsiflexion
- total dose of approx 200mg ropivacaine (ie, in 20-30ml each block) single shot, or via nerve sheath catheter and continuous infusion.
(2011-B) Question 13 A 50-year-old man presents with confusion and the following electrolyte profile: Na+ 155 mmol/l K+ 4 mmol/l HCO3− 15 mmol/l Creatinine 120 μmol/l Hb 200 g/l a. What are the possible causes of this abnormality? (30%) b. How can they be distinguished? (70%)
Bloods show Hypernatremia, raised creatinine, a metabolic acidosis, and high Hb.

- There is major imbalance of Na/H20, which could be voluntary/iatrogenic/pathologic.
- potentially diabetes insipidus (central/nephrogenic)
- severe dehydration, HONK, renal failure, etc

Hx
- drinking/thirst
- fluid loss, trauma
- comoridities (ie, diabetes, renal disease, cardiac disease, medications/diuretics, endocrine disease, sx suggestive of neurogenic problems ~ headache, focal deficit, other endo problems), psych (lithium)

Ex
- volume status / dehydration
- neurological (ie, pituitary - visual fields, raised ICP)
- urinalysis

Ix
- plasma/urine osmolality, electrolytes, BSL
- anion gap
- Imaging including CT brain (pituitary infarction/mass)
response to ADH?
(2011-B) Question 14 A 70-year-old man has undergone radical prostatectomy under general anaesthesia. On emergence he has crushing central chest pain, is restless, and has cold, clammy skin. His blood pressure is 90/50 mm Hg, pulse rate 110/minute and SpO2 is 95% on oxygen via a Hudson mask. A twelve-lead ECG shows widespread ST segment elevation across the anterior chest leads. a. Describe your immediate management. (50%) b. What are the treatment priorities for this patient? (50%)
.
(2011-B) Question 15 Explain your approach to thromboprophylaxis in the patient undergoing total knee replacement.
This important clinical question has been asked in previous papers.
Key components of a response to this question included: Consideration of
- non-pharmacological techniques
- pharmacological agents
- consideration of patient factors that may influence clinical decision- making

and application of the evidence for thromboprophylaxis in this setting
(2012-A) Question 2 A 65-year-old man is on your list for an arthroscopic acromioplasty that is to be performed in the beachchair position. a. List the complications associated with this position (30%) b. Describe how the risk of these complications can be minimised (70%)
Beach-chair position is a semi-sitting position.

a) Hypoperfusion and cerebral/opthalmic/spinal cord/coronary ischaemia
may have risk of VAE but not usually a risk as irrigation fluid is used.
- arterial supply or venous drainage may be impaired
- pressure areas, to soft tissue/nerves, especially brachial plexus (need good support of arms) / ulnar, auricular, occiptal nerves
- poor access to patient (especially airway)

b)
slow positioning to minimise sudden changes in preload
the "least" head up possible for surgical access
invasive monitoring may be required (for BP, but also check Hb)
fluid loading / use of vasopressor for BP support, legs slightly elevated to increase preload
padding of sacral areas, eyes, arms
securing of airway (ETT, tapes/ties)
surveillance intraoperatively
plan on what to do in emergency
(2012-A) Question 3 A 60-year-old man is admitted to the high dependency unit following laparotomy for relief of a large bowel obstruction. He has a urinary catheter in situ. Three hours later he remains oliguric. a. Define oliguria (10%) b. What are the potential causes of oliguria in this patient? (40%) c. How would you differentiate between these causes? (50%)
a) <0.5ml/kg/hr urine output

b)
Pre-renal
- hypotension (cardiogenic shock, septic shock, hypovolemia, high IAP)

Renal
- pre-existing CRF, renal ischaemia / ATN, nephrotoxins, NORMAL stress response (increased ADH secretion)

Post-Renal
- obstruction in ureters (ie, mass, sutures)
- Blocked IDC
- raised IAP

c)
Differentiation by Hx/Ex/Ix
Hx - pre-existing impairments, fluid balance, anaesthetic chart
Ex - signs of hypovolemia, sepsis/cardiogenic shock, mucous membranes/skin turgor, etc
Ix - response to fluid bolus / monitoring of Cardiac output by some means / BP. Renal function, unblock/replace catheter, renal ultrasound, measure IAP.
(2012-A) Question 4 An elderly patient has previously declined an above knee amputation for a gangrenous leg. She becomes acutely unwell, confused and no longer competent to make decisions. At the request of the family, the surgeon has approached you to discuss whether to proceed with surgery or not. She is likely to die without the surgery. Outline the ethical considerations you would discuss with the surgeon
- Autonomy
Patient's right to self-determination about medical treatment, and earlier wish (was she competent then? Was she fully informed of results of her decision?)

- Beneficence
Depends on pre-morbid state, comorbidities, quality of life, and whether palliation is seen as doing good or bad

- non-malificence
as above. Harm may be done to the patient by proceeding, and having protracted recovery or death (also construed as assault given earlier wishes).

- Paternalism
Family / Medical staff making decision that overrides patient's - is this valid?

- Basis of family request?

- Justice
Use of resources in someone where treatment may or may not be futile, with previously expressed wish of no treatment

- Relief of suffering may bring about death to the patient.
(2012-A) Question 5 A healthy 28-year-old primigravida is scheduled for elective lower segment caesarean section for breech presentation at 39 weeks gestation. You have performed a spinal anaesthetic using 0.5% bupivacaine 2.2 ml and fentanyl 15 μg (total volume 2.5 ml). a. Describe the issues in assessing adequacy of the block for the planned surgery (50%) b. Describe the options for managing an inadequate block recognised prior to commencement of surgery (50%)
a) BLock to light touch or ice on the skin may not relate to adequate blockade of intraperitoneal pain fibres, and so may still have pain. motor block is reassuring, but again, this is not a sign of abdominal nerve blockade.
Issues with assessing block -
- level of block required (controversy in literature, and depends on what is used to test block, ie, ice, light touch, pin prick, ethyl chloride spray - usual practice is ice/pinprick, and accept T5-6).
- block to light touch better?
- timing of testing, if tested straight away, may not appear adequate, and therefore may also feed into mother's anxiety
- maternal expectations of block

b)
This is an elective procedure, and Mx will depend on the exact nature of the block. The benefits of the regional anaesthetic still outweigh risks of GA
- No block?
- patchy or low block?
- technical difficulties with spinal, loss of fluid on injection, drug error
- wait more time, position table
Options include Spinal, Epidural, CSE, GA.
If No block, and no technical difficulties - I would presume not in IT space, and redo spinal
If partial block, a CSE/Epidural technique may be safer (although time consuming), and prevent high block occurring.
GA possibly, if technically very difficult, and no obvious contraindications.
(2012-A) Question 6 A 25-year-old man with recurrent pneumothorax and persistent air leak is scheduled for video-assisted thoracoscopic pleurodesis. a. Outline the considerations involved in induction of anaesthesia in a patient with a persistent air leak (50%) b. Outline the management of an intraoperative deterioration of oxygen saturation in this patient (50%)
A)
The main risk to the patient is inability to positive pressure ventilate adequately (due to loss of tidal volume through leak, which means "good lung" i not ventilated - depends on size of leak), or tension pneumothorax if no ICC or clamped ICC wit PPV before lung isolation
- need to assess size of leak and clinical condition of patient (ie, hypoxic, oxygen-requiring, distressed)
- anaesthetic experience (not for the occasional thoracic anaesthetist), and assistance
- means of induction (ie, AFOI, Gas induction which allows for SV and no IPPV until lung isolation = difficult)
- if GA/ETT, use of RSI with no ventilation, difficult airway? Needs assessment if previous op.
- need to have a rapid means of assessing placement / lung isolation (FOB)

b)
Depends on severity and rapidity of fall in saturation (may need immediate resumption of 2LV, which may not improve things given BPF).
Apart from problems with OLV (and mainly SHUNT), need to exclude other causes (ie, machine, circuit, tube).
100% oxygen, hand-ventilation for compliance, check monitors (CO2/O2 delivery), communication with surgeon +/- calling for assistance
Check position with FOB (patency too)
Ensure adequate CO (BP/HR, fluid bolus, vasopressor, anaesthetic depth)
If atelectasis in dependent lung = PEEP
upper lung CPAP
Recruitment
intermittent 2LV
(2012-A) Question 7 a. List the methods of assessing intracranial pressure (ICP) (30%) b. Evaluate the role of ICP monitoring in the setting of traumatic brain injury (70%)
a)
Clinical (Hx - headaches, blurred vision, vomiting), Ex (papilloedema, cranial N palsies)
Non-invasive = CT
Invasive
- EVD
- intraparenchymal monitors (strain-guage = camino Bolt, fibreoptic = Codman's)
Indirect = TCD, Jugular bulb sats, NIRS, etc

b)
ICP monitoring can be used as "goal-directed" tool for CPP-based treatment (CPP = MAP - ICP)
Evidence not great
BTF recommendations
- severe TBI, with abnormal CT
- Severe TBI with normal CT and 2 0r more
*Age>40, motor posturing, SBP<90 on presentation
- other occasions if patient requires GA/Sedation in ICU for another reason (as unable to fully assess neuro status)

ICP monitors may also be therapeutic (ie, CSF drainage in high ICP with EVD)

Disadvantages
- no great evidence to back monitoring of ICP/CPP
- ICP > 20 historically poor prognosis but not always
- some trials (non-randomised) demonstrated better outcome with no ICP monitoring (and just for target MAP). Significant problems with these studies though (non-randomised, centre vs centre trial, did not include early deaths).
- CPP mx ass w higher ARDS (with CPP >70)
- costly
(2012-A) Question 8 A 35-year-old female is booked for thyroidectomy. Her blood results are as follows. Thyroid stimulating hormone (TSH, thyrotropin) 0.1 (N 0.3 – 3 mIU/l) Total Thyroxine (Total T4) 20 (N 4 – 11 μg/dl) Free Thyroxine (Free T4) 4 (N 0.7 – 1.8 ng/dl) Free Tri-iodothyronine (Free T3) 120 (N 60 – 175 ng/dl) a. Interpret the thyroid function tests (10%) b. Justify when you would proceed to thyroidectomy in this patient (50%) c. What is the management of an intraoperative thyrotoxic crisis? (40%)
a) Thyrotoxicosis with high free T4, and negative feedback low TSH from pituitary.

b)
Need to aim for euthyroid state prior to surgery to reduce risk of thyroid storm. This often takes 6-8 weeks for anti-thyroid drugs to take full effect.
Needs to be weighed up against possibility of airway obstruction from large goitre or retrosternal goitre, and/or aggressive malignancy.
Ie, when no longer symptomatic, and blood tests have normalised (T4 particularly in this case).
- drugs used carbimazole, propylthiouracil, beta-blockers, steroids (reduce peripheral conversion to T3)

c)
Vigilance, and low index of suspicion
DDx: MH, NMS, Sepsis
Characterised by haemodynamic instability, hyperthermia.
Need to call for assistance, and notify surgeon (?stop surgery).

Supportive Rx
- IV fluids
- BP support (up or down)
- beta-blockers for tachycardia
- cooling

Specific
- IV propylthiouracil
- IV hydrocortisone
- Lugol's iodine

Need to plan disposition of patient (usually ICU).
May need invasive monitoring if not already done.
(2012-A) Question 9 A 3-year-old presents to the emergency department with a recent onset of stridor. a. List the differential diagnoses (30%) b. How do you differentiate between the potential causes of this stridor? (70%)
a)
Could be supraglottic, glottic or subglottic.
Causes could be
- foreign body
- upper airway infection (retropharyngeal abscess, epiglottitis, croup, other)
- trauma to upper airway
- anaphylaxis

b)
Hx
- previous hx (ie, prolonged intubation, Down's - risk of subglottic stenosis), onset (rapid may be inhaled FB, slow may be croup/infection), well/unwell?, cough, fevers, hx of trauma
Ex
- looks toxic? febrile? drooling, posture. other injuries.
- angiooedema
Ix
- may not be appropriate in severely distressed child, may just start treatment with steroids (pred 1mg/kg or dex 0.15mg/kg) /ab's/nebulised adrenaline (0.5ml/kg of 1:1000).
- may need OT for rigid bronchoscopy or intubation
- xray? potential signs of soft tissue swelling or FB
(2012-A) Question 10 An adult patient from the intensive care unit with severe adult respiratory distress syndrome (ARDS) requires a laparotomy for an acute abdomen. a. What are the features of ARDS? (30%) b. Explain your perioperative ventilation strategy (70%)
ARDS
a) non-cardiogenic pulmonary oedema resulting in hypoxic respiratory failure.
- defined by
*bilateral infiltrates on CXR
*Normal LA pressure (or not explained by cardiac failure), PCWP<18
*PaO2/FiO2 ratio <200 (ALI is < 300), ie, A-a gradient due to SHUNT.

lung injury caused by multiple possible processes (ie, sepsis, SIRS, burns, CPB), causing alveolar exudate, collapse, and inflammatory response.

b)
Need "lung protective ventilation" as per ARDSnet.
- TV ~6ml/kg IBW
- Plateau pressure < 30
- permissive hypercapnia if required (hopefully pH>7.2)
- use of high PEEP to splint alveoli (ie, up to 15cmH2O), this is defined by the lowest PEEP above the "inflection point" on a compliance curve
- use of alveolar recruitment maneouvres as necessary

May need other strategies (note - the question says ventilation strategies, but the examiner's report does go outside this - READ THE QUESTION EXAMINERS!)
*NO, iloprost, restrictive fluids
No strong evidence for any of these, some have shown improved oxygenation without an overall improvement in outcome.

The only thing that has significantly improved outcome is the Lung Protective Ventilation strategy.
(2012-A) Question 11 You are asked to initiate an opioid patient-controlled analgesia service in your hospital. a. How would you ensure patient safety? (70%) b. What are the key components to include in designing an order form? (30%)
a)
- Staff Education
competency-based learning for nurses, medical staff
- Protocols
standardised orders to reduce risk of drug error
naloxone prescription required
No sedatives/opioids
APS control
standardised nursing observations
patient selection (ie, language, cognitive dysfunction, etc)
- Nursing Obs
regular sats/RR/Pain scores/Sedation scores 1/24 for 12 hrs, and 2/24 after depending on pt state
- Equipment
lockable, programmable pumps
anti-reflux valves
appropriate alarms and maintenance
- daily review by medical staff
- QA / incident monitoring

b)
Patient details
standardised prescription data
naloxone protocol
oxygen prescription
only to be filled in by anaesthetists
(2012-A) Question 12 a. Describe the anatomy of the epidural space (50%) b. What are the clinical implications of the anatomical differences between thoracic and lumbar epidural spaces in the placement and management of epidural analgesia? (50%)
(2012-A) Question 12 a. Describe the anatomy of the epidural space (50%) b. What are the clinical implications of the anatomical differences between thoracic and lumbar epidural spaces in the placement and management of epidural analgesia? (50%)
.
a) The epidural space is the space surrounding the dura (generally referring to the vertebral canal).

- Boundaries
*Superior = fusion of spinal/perisoteal layers of dura at foramen magnum
*Inferior = sacrococcygeal membrane
*Anterior = Posterior longitudinal ligament, vertebral bodies and IV discs
*Posteriorly = ligamentum flavum, facet joints and laminae (bone next to spinous process)
*Laterally = Pedicles, intervertabral foraminae, and continuous with paravertebral space

-Contains
- dural sac, ending at lower border S2, then filum terminale
- Fat
- spinal nerves
- epidural venous plexus of Batson (mainly anterior space)
- spinal segemental arteries

b) Anatomical differences
- thoracic region more problematic
* more horizontal spinous processes, so more difficult access, more use of paramedian approach
*smaller distance from ligamentum flavum to dura in thoracic region (ie, T6 = 2-3mm, L2 = 5-6mm), so higher risk of dural puncture, also due to decreased volume, higher dermatomal spread for given volume of LA (therefore lower bolus doses and infusion rates required).
- more commonly caudal flow of LA in thoracic region, versus cranial with lumbar
(2012-A) Question 13 a. List the risk factors for perioperative stroke (50%) b. Describe how you would minimise the risk in a high-risk patient having major orthopaedic surgery (50%)
Stroke is the result of either ischaemia (thrombosis/embolism) or harmorrhage (15%), and is a syndrome resulting from infarction of brain tissue (by definition sx lasting > 24 hrs).

Risk Fx
- Patient Fx
Age > 70, Male, previous/recent Stroke/TIA, diabetes, HT, Smoker, AF, CCF, thrombophilia
- Surgical Fx
Carotid, Cardiac, Neuro surgeries high risk
Other vascular, orthopaedic
Duration of surgery
Emergency surgery
inflammatory response
surgical technique (ie, risk of "kinking" of vessels in aneurysm clipping, etc)
- Anaesthetic Fx
use of high dose metoprolol SR preop (POISE) increased risk
Hypotension, hypoxia
Choice of GA/LA in carotid surgery makes no difference.

Minimising Risk
- Preop
continuing antiplatelets/anticoagulation where possible (thrombotic), or minimise time off
delaying surgery after sx/stroke
titration of beta-blockers carefully, and continue statins
manage other risk fx (ie, DM control, HT control, cessation smoking)

Intraop
- Care with BP/CO2 mx, maintenance of MAP/CPP
- care with positioning (ie, not sitting)
- control arrythmias
- neurological monitoring if regional technqiue

Postop
- BP control
- continue usual meds and re-start antiplatelets/anticoagulants when possible
(2012-A) Question 14 a. What do the terms decontamination, disinfection and sterilisation mean? (30%) b. What measures should be in place to minimise the risk of transmission of infection to the respiratory tract of patients via anaesthetic equipment? (70%)
see 2009
(2012-A) Question 15 With regard to oxygen therapy for patients in a general postoperative ward a. Describe the options available (30%) b. What are the justifications for your choice for a particular patient? (70%)
a) Variable performance devices (nasal prongs, face mask), and Fixed performance devices (ie, Venturi or high-flow Oxygen devices - not generally available on general ward).
Depends on patient's MV (particularly peak inspiratory flow) and oxygen flow, to determine degree of air entrained and "dilution" of FiO2.
(2011-A) Q7.
A patient is scheduled for posterior fossa surgery in the sitting position.
(a) Outline the precautions you would take to minimise the risk of venous air embolism. (70%)
(b) How would you recognise an air embolism intraoperatively? (30%)
a)
Preop.
Screening for at risk patients (particularly for paradoxical embolism - ie, PFO, VA shunt).
Discussion with surgeon re: planning - semi-sitting or another position.
Discussion with OT staff re: drill for VAE.

Intra-op
- Position with head as close to level of heart as possible to decrease gradient for air entrainment
- fluid loading, wrapping legs, and raising legs to increase central blood volume, and raise CVP
- Monitoring. Many types, but TOE, praecordial doppler, oesophageal stethescope, ETCO2 monitoring, PAC all have different strengths. CVC (special) in RA to aspirate air if does occur.
- Surgical
* use of bone wax, and careful monitoring of incisions/wounds, ready to flood surgical field with saline

b)
- TOE most sensitive, but need training and difficult in this position
- praecordial doppler/ ETCO2 monitoring probably best balance of sensitivity/specificity and ease of monitoring
- oesophageal stethescope / doppler by themselves not as specific or sensitivie
- PAC, invasive, will have some advantages, but will likely only see large changes
- hypotension, tachycardia will occur with large VAE
2010b - Question 5
A 6-month-old boy presents with an acute abdomen. He is diagnosed with intussusception and booked for laparotomy after a failed attempt at reduction. His heart rate is 160bpm and BP is 75/45 mmHg.
His electrolyte profile is as shown:
• Na⁺ 132 mmol/l (135 – 145)
• K⁺ 2.7 mmol/l (3.5 – 5.5)
• Cl⁻ 106 mmol/l (95 – 110)
• Urea 3.3 mmol/l (3.5 – 8.5)
• Creatinine 86 μmol/l (60 – 110)
• Lactate 4.5 mmol/l (1.0 – 1.8)
(a) How would you determine his degree of dehydration and how severe is it likely to be? (40%)
(b) Describe your perioperative fluid management. (40%)
(c) When would you proceed to surgery and why? (20%)
Intussusception is an emergency, and is caused by small bowel "telescoping" with potential for bowel obstruction, ischaemia, and fluid shifts causing severe shock.

a) I would assess degree of dehydration by Hx/Ex/Ix.
Hx - decreased oral intake, decreased wet nappies, lethargy. The hx of failed radiological intervention is significant, because it means there has probably been under resuscitation of the baby, and may have been a prolonged attempt.
Ex - vitals (HR/BP/RR), colour (pale, grey/mottled), general appearance (vigorous, or lethargic/flat), mucous membranes, cap refill, urine output (<0.5ml/kg/hr bad).
Ix- Bloods/electrolytes, venous gas (acidemia), lactate

Given the results given, it is difficult to know, but probably at least moderate dehydration if not severe - some evidence of shock with decreased perfusion (lactate). BP is potentially within normal range, while HR probably high. at least 10% dehydrated.

b)
Preop- needs fluid resuscitation with colloid (4% albumin) or crystalloid (Nacl or CSL) in 10ml/kg boluses until signs of underperfusion improve. (ie, HR slower, cap refill improved, colour improved, conscious state improved - could also take repeat VBG looking at lactate and electrolytes). ideally a 2nd cannula for maintenace fluid and potassium replacement could be given (ie, "increased" maintenace rates of 6ml/kg/hr of 1/2 NS & 5% dextrose w 20mmol/L KCL).

Intraop - continue maintenace fluids above as described. Replace obvious losses (ie, blood) by measuring packs, urine output, HR/BP. Some abdominal surgery will need 5-10ml/kg/hr of crystalloid. Re-checking of VBG intraop would be useful for electolytes, and adequacy of fluids, lactate, Hb.
Postop- depends on clinical ax and Ix. may need further volume replacement and electrolytes. Check Hb - ?need for transfusion.
Otherwise continue maintenance rates of 4ml/kg/hr (likely under 10kg) of above solution (take out KCL when this corrected).
c)
The decision to proceed to surgery should be made in conjunction with the surgeon. The laparotomy is urgent due to risk of bowel infarction, but the risk of anaesthetising an under-resuscitated child is also significant, and may result in cvs collapse.
The child should have at least initial fluid boluses to try and correct shock, and to be stable enough for induction of anaesthesia. Resuscitation is then ongoing throughout surgery - with constant checking of outcoem measures - Hr/BP/Urine output/cap refill, VBG/lactate, etc. I personally would only proceed after significant volume resuscitation has occurred, while being mindful of the need for urgent surgery.
(May-2006 Q5) A fifty year old man taking corticosteroid and pyridostigmine for myasthenia gravis is to have an elective right hemicolectomy under general anaesthesia. Discuss your management of his myasthenia pre and post operatively.
Myasthenia Gravis is an autoimmune condition affecting the neuromuscular junction, which causes muscle weakness and fatiguability, most commonly around they eye (diplopia, ptosis), however can also affect bulbar function, respiratory muscles.

Preop Mx
- Need to determine patient's risk, severity and stability of disease, particular predicting risk of postoperative need for ventilation

Hx/Ex
- duration of illness, stability, recent symptoms, bulbar dysfunction, medications and response (including recent worsening of weakness - cholinergic/myasthenic crisis, cushing's secondary pred use), previous probs under GA
- any associated comorbidities (cardiomyopathy, heart block, hyperthyroidism, etc)
- problems associated with the PATHOLOGY (ie, cancer? = 4 M's)
- signs of bulbar dysfn, respiratory disease, cardiac disease, cushing's
- Ix, including RFT's, ECG, Bloods, ?Tensilon test

Predictors of Postop Ventilation
- duration of disease > 6 yrs
- daily dose pyridostigmine > 750mg
- VC < 2.9L
- Co-existing resp disease (ie, COAD)
- bulbar dysfunction
- intrathoracic/intraperitoneal surgery

Consultation/optimisation
- Neurologist re: stability/optimisation, respiratory, ICU
- optimisation may involve change to medication, or even preop plasmapheresis
- medications need to be continued periop, so consideration on advice re: IV treatment, and steroid cover
- consideration of aspiration prophylaxis if bulbar dysfunction

Postop
- depending on surgery, complications
- need to assess neuromuscular blockade carefully (as only need 1/10th intubating dose or none at all, may be worsened by residual volatile, aminoglycoside, etc)
- needs to reach adequate tidal volumes to extubate, but may worsen later postop (need HDU)
- consider epidural so that systemic opioids can be avoided